88

Click here to load reader

Revista (format .pdf, 1.6 MB)

  • Upload
    ngothuy

  • View
    402

  • Download
    85

Embed Size (px)

Citation preview

Page 1: Revista (format .pdf, 1.6 MB)

Anul IV, Nr. 2 Iulie – Decembrie 2002

RECREAŢII MATEMATICE

REVISTĂ DE MATEMATICĂ PENTRU ELEVI ŞI PROFESORI

E d i t u r a “ C r e n g u ţ a G â l d ă u ”

I AŞ I , 2 0 0 2

e iπ = −1

Page 2: Revista (format .pdf, 1.6 MB)

Semnificaţia formulei de pe copertă:

Într-o formă concisă, formula 1−=πie leagă cele patru ramuri fundamentale ale matematicii: ARITMETICA reprezentată de 1 GEOMETRIA reprezentată de π ALGEBRA reprezentată de i ANALIZA MATEMATICĂ reprezentată de e

Redacţia revistei : Petru ASAFTEI , Temistocle BÎRSAN, Dan BRÂNZEI, Constantin CHIRILĂ, Eugenia COHAL, Adrian CORDUNEANU, Paraschiva GALIA, Mihai GÂRTAN, Paul GEORGESCU, Dumitru GHERMAN (Paşcani), Gheorghe IUREA, Lucian Georges LĂDUNCĂ, Gabriel MÎRŞANU, Gabriel POPA, Dan POPESCU (Suceava), Florin POPOVICI (Braşov), Maria RACU, Petru RĂDUCANU

Coordonatorul numărului : Temistocle BÎRSAN Adresa redacţiei:

Catedra de Matematică – Universitatea Tehnică “Gh. Asachi” Iaşi Bd. Carol I, nr.11, 6600, Iaşi

Tel. 032 – 213737 / int. 123 E-mail: [email protected]

©EDITURA CRENGUŢA GÂLDĂU Toate drepturile rezervate ISSN 1582 - 1765

Bd. N. Iorga, Bl. K2, ap. 4 Tel. / Fax: 032 - 230598

IAŞI, 6600

Page 3: Revista (format .pdf, 1.6 MB)

Anul IV, Nr. 2 Iulie – Decembrie 2002

RECREAŢ I I MATEMATICE

REVISTĂ DE MATEMATICĂ PENTRU ELEVI ŞI PROFESORI

Apare cu sprijinul

FILIALEI IAŞI a SOCIETĂŢII de ŞTIINŢE MATEMATICE

IAŞI, 2002

eiπ = −1

Page 4: Revista (format .pdf, 1.6 MB)
Page 5: Revista (format .pdf, 1.6 MB)

FractaliStefan Frunza 1

Ce este un fractal? Benoit B. Mandelbrot a inventat termenul în 1975. Acestcuvânt deriva din fractus care în limba latina înseamna fracturat (derivat, la rîndulsau, din verbul frangere - a frânge, a rupe). El vrea sa sugereze o multime careeste mult mai ”neregulata” decât multimile considerate în geometria clasica; cu câtaceasta este marita, tot mai multe neregularitati devin vizibile. În lucrarea sa ”TheFractural Geometry of Nature” (1982),Mandelbrot argumenteaza ca asemenea ab-stractiuni geometrice se potrivesc adesea cu lumea fizica mai bine decât curbele sisuprafetele netede. De exemplu, o linie de coasta neregulata (cum ar fi, de exem-plu, coasta estica a Angliei) arata destul de neteda daca o privim din avion, de lao înaltime mare, dar, pe masura ce ne apropiem, tot mai multe neregularitati devinvizibile. Aceste neregularitati creeaza probleme si în calcularea lungimei liniei decoasta sau a frontierei a doua tari vecine. Fizicianul englez L.F.Richardson stu-dia, în 1961, variatiile lungimilor aproximative L (ε) ale diverselor coaste, masuratecu compasul, pe harta, ca functie de etalonul ε si constata ca, pentru un larg domeniude valori ale lui ε, lungimea varia dupa o putere a lui ε,

L (ε) = N (ε) ε ∝ ε−ρ, ρ > o

(N (ε) este numarul de pasi de lungime ε cuprinsi cu compasul în linia respectivasi a (ε) ∝ b (ε) înseamna ca a (ε) si b (ε) variaza la fel când ε → 0+, mai precisa (ε) /b (ε) tinde la o constanta nenula când ε→ 0+).Se vede ca pentru o curba neteda cum este cercul, lungimea devine constanta

(ρ = 0) când etalonul este suficient de mic în raport cu raza de curbura. Dimen-siunea cercului este D = 1 (si corespunde lui ρ = 0). Alte curbe însa prezinta unexponent ρ > 0, astfel încât lungimea lor creste nedefinit pe masura ce etalonul semicsoreaza; este imposibil sa le atribuim o lungime finita, se spune ca aceste curbesunt nerectificabile.Exponentul 1+ρ al lui

1

N (ε), definit mai sus, este în fapt o ”dimensiune fractala”.

Aceasta determinare a masurii fractale, prin acoperirea curbei cu discuri de raza ε,este exact cea utilizata de Pontriaghin si Schnirelman, în 1932, pentru a definidimensiunea de acoperire.O varianta idealizata a unei asemenea situatii, care este si un model matematic

de fractal, este curba lui Helge von Koch (1904). Se porneste cu un segment unitate(initiatorul) care se împarte în 3 segmente congruente si se înlocuieste segmentul dinmijloc cu cele doua laturi ale unui triunghi echilateral, avându-l ca baza si situatdeasupra lui (generatorul).

. . .

În continuare se repeta aceeasi procedura cu fiecare din cele 4 segmente obtinute;constructia are un caracter recursiv. Curba poligonala obtinuta în fiecare stadiu

1 Conf. dr., Facultatea de matematica, Univ.”Al. I. Cuza”, Iasi

1

Page 6: Revista (format .pdf, 1.6 MB)

este, dupa Mandelbrot, o structura prefractala. Curba lui Koch este figura obtinutala limita.În aceasta situatie se poate urmari explicit cum depinde lungimea de etalonul de

masura.În stadiul 0 avem ε1 = 1, N (ε1) = 1 si L (ε1) = 1. Pentru ε2 = 1

3avemN (ε2) = 4

si L (ε2) = N (ε2) ε2 =4

3; pentru ε3 =

1

32avem N (ε3) = 4

2 si L (ε3) = N (ε3) ε3 =

= 42/32.

Se demonstreaza prin inductie ca, pentru εn =1

3n−1, N (εn) = 4n−1 si L (εn) =

= N (εn) εn = 4n−1/3n−1. Se observa deci ca L (εn)→∞ pentru εn → 0 (n→∞).

De aici rezulta si dimensiunea fractala1

N (εn)=

1

4n−1=

1

3(n−1) log3 4= εlog3 4n ,

deci D = 1 + ρ = log3 4 =ln 4

ln 3= 1, 2618... .

O varianta a curbei lui Koch se obtine daca triunghiul echilateral se înlocuieste cu

un triunghi isoscel cu unghiul la vârf α si bazal

2sin

α

2(l-lungimea laturii segmentului

initial).Dimensiunea fractalului corespunzator este D = log 4/ log [2 + 2 sin (α/2)].Cazul α = 0 conduce la D = 2, adica o curba care umple un triunghi. Aceasta

nu e o curba simpla deoarece are multe puncte duble.

. . .

Dimensiunea D = 2 se poate obtine pentru o curba simpla (curba lui Peano) încare generatorul este o usoara adaptare a situatiei

. . .

pentru a elimina punctele duble:

. . .

Aceasta este o curba simpla densa într-un patrat; faptul ca nu are puncte multiplese traduce prin faptul ca e o curba simpla. Deci are dimensiunea topologica 1.Trei copii ale curbei lui Koch construite, în exterior, pe cele trei laturi ale unui

triunghi echilateral formeaza o curba simpla închisa, numita adesea insula lui Kochsau curba fulgului de zapada (snowflake).Un fractal cu lacune, construit pe dreapta este multimea (sau praful) lui Cantor.

Ca si curba lui Peano, acesta multime a fost imaginata din motive pur matematice.Se porneste cu un segment care se împarte, în stadiul 1, în trei segmente congruentesi se elimina treimea din mijloc.

. . .

Cu fiecare din segmentele ramase se procedeaza analog.

2

Page 7: Revista (format .pdf, 1.6 MB)

Ea este un fractal cu dimensiunea (fractala) D = log 2/ log 3 = 0, 6309... si di-mensiunea topologica zero.Dimensiunea fractala nu caracterizeaza ea singura obiectul. Exista multimi de tip

Cantor având aceeasi dimensiune fractala, dar o structura spatiala diferita.Se împarte un segment (initiatorul) în 27 segmente congruente si se retin, în prima

etapa 4 segmente de lungime 1/9, egal departate, începând cu primul segment dinstânga pâna la ultimul segment din dreapta. În continuare se procedeaza la fel cufiecare din segmentele retinute.

. . .

Cele doua multimi de tip Cantor au aceeasi dimensiune fractala, dar difera prinlacunaritatea lor, o alta caracteristica ce se cere nu numai intuita ci si clar definita.Un alt fractal lacunar, cu o descriere similara, este triunghiul (sau sita) lui Sier-

pinski. Se porneste cu un triunghi echilateral care se împarte prin mijloacele laturilorsale în 4 triunghiuri congruente si se elimina triunghiul din mijloc. Cu fiecare dintriunghiurile ramase se procedeaza analog.

. . .

Multimea ramasa este sita lui Sierpinski. Acesta este un fractal cu dimensiuneaD = log 3/ log 2 = 1, 585...Adesea se considera numai laturile si se obtine un fractal cu aceeasi dimensiune.

Se poate arata ca ambele structuri prefractale converg la aceeasi structura fractala însensul unei distante naturale introduse de matematicianul român Dimitrie Pom-peiu si de matematicianul german Felix Hausdorff.O alta varianta este covorul (carpeta) lui Sierpinski.Se porneste cu un patrat care se împarte în 32 patrate congruente si se elimina

patratul din mijloc. Cu fiecare din patratele ramase se procedeaza la fel.

. . .

Exemplele de structuri fractale deterministe construite dupa cele doua modele sepot multiplica la infinit. Aceste structuri se pot dovedi foarte interesante pentrua modela unele probleme de transport în medii poroase si permit calcule analiticeexacte pentru diverse proprietati fizice (conductanta, vibratii, electrozi fractali etc.).Se pot imagina si analoage tridimensionale: sita tridimensionala a lui Sierpinski siburetele lui Menger, care au dimensiunile fractale D = log 4/ log 2 = 2, respectivD = log 20/ log 3 = 2, 73 . . .O alta varianta posibila consta în prezenta simultana a mai multor scari de di-

latare. Figura urmatoare ilustreaza iterarea determinata de factori 1/4 si 1/2.

3

Page 8: Revista (format .pdf, 1.6 MB)

. . .

Dimensiunea sa fractala este D = log¡1 +√17¢/ log 2 − 1. Am dat pîna acum

exemple de fractali deterministi (exacti), dar se pot defini cu usurinta si structurifractale statistice. Cei mai simpli sunt cei omogeni, când aria, volumul (sau masa)structurii sunt repartizate uniform la fiecare nivel al ierarhiei, adica diversii generatoriconserva raportul de masa de la un nivel la urmatorul.Astfel, se poate porni de la recurenta cu caracter statistic

. . .

cu raportul de masa (arie) 2/4 = 1/2 si se poate construi fractalul statistic ce începeprin

. . .

Raportul de masa poate varia de la un pas la cel urmator (fractali eterogeni).Un alt exemplu de fractal statistic poate modela distributia craterelor Lunii. Ea

poate fi privita ca o distributie de discuri ale caror centre urmeaza de exemplu odistributie Poisson si ale caror raze sunt aleatoare cu o densitate de probabilitate detipul P (R > r) = Qr−α. Numeroase asemenea exemple sunt descrise de Mandelbrotîn [3].Sa ne reîntoarcem acum la conceptul de dimensiune. O tentativa naturala de

a masura dimensiunea unui obiect E consta în pavajul obiectului prin ”paveuri”(apartinând spatiului în care obiectul este scufundat) de masura µ = εd(E), unded (E) este dimensiunea obiectului. Dar cum d (E) este apriori necunoscuta, o solutieconsta în a face încercari luând unitati de masura µ = εα cu un exponent α nedeter-minat. Sa consideram de exemplu un patrat (d = 2) de latura L si sa-l acoperim cupaveuri (patrate) de latura ε. Masura sa va fi data de M = Nµ, unde N e numarulde paveuri, adica N = (L/ε)

d. Astfel

M = Nεα =

µL

ε

¶dεα = L2εα−2.

Daca se încearca α = 1 se obtine M → ∞ când ε → 0+; lungimea unui patrat esteinfinita. Daca se încearca α = 3, se gaseste ca M → 0 când ε→ 0+; ”volumul” unuipatrat este nul. Aria patratului se obtine doar pentru α = 2 si dimensiunea sa estecea a unei suprafete α = d = 2.Faptul ca aceasta metoda se poate aplica pentru un α real oarecare permite

generalizarea la dimensiuni care nu sunt întregi. Se poate formaliza putin aceasta

4

Page 9: Revista (format .pdf, 1.6 MB)

masura. Mai întâi, având de-a face cu un obiect de forma oarecare, nu este în generalposibil sa-l acoperim cu paveuri identice de latura ε. Se poate însa acoperi obiectulprin bile Vi de diametru d (Vi) ≤ ε. Aceasta ofera un plus de suplete, dar impune saluam limita inferioara a sumei masurilor elementare µ = d (Vi)

α. Se obtine astfel ceeace se numeste α-masura de acoperire (Hausdorff 1919, Besicovici 1935) definitaprin

mα (E) = limε→0infnX

d (Vi)α : E ⊂ ∪Vi, d (Vi) ≤ ε

o.

Se defineste dimensiunea Hausdorff-Besicovici prin

d (E) = inf α : mα (E) = 0 = sup α : mα (E) =∞ .Deci dimensiunea Hausdorff-Besicovici este valoarea lui α pentru care mα face unsalt de la zero la infinit. Aceasta masura pentru α = d (E) poate fi orice numar întrezero si infinit.Aceste consideratii sugereaza, de exemplu, ca lungimea unei curbe nerectificabile

trebuie înlocuita cu masura Hausdorff corespunzatoare.Exista si alte definitii ale dimensiunii unei multimi, unele echivalente, altele

diferite de dimensiunea Hausdorff-Besicovici. Se poate defini, de asemenea, o di-mensiune topologica; o multime formata dintr-un punct are dimensiunea topologicazero, o multime ce poate fi pusa în corespondenta bijectiva si bicontinua (homeomor-fism) cu o curba neteda are dimensiunea topologica 1 s.a.m.d.O definitie-tentativa a fractalilor, propusa de Mandelbrot, consta în a cere ca

dimensiunea Hausdorff-Besicovici sa fie strict mai mare ca dimensiunea topologica.Aceasta se refera desigur la fractalii ideali, obtinuti din prefractali prin trecere lalimita în raport cu distanta Pompeiu-Hausdorff.Trebuie precizat ca fractalii au fost introdusi si utilizati în matematica înca la

sfârsitul secolului XIX-lea de Cantor, Peano, Sierpinski, Menger s.a. Aceste exempleau fost considerate multa vreme ca situate împotriva naturii. Este meritul incon-testabil al lui B.Mandelbrot de a fi demonstrat ca multimile fractale modeleaza oîntreaga varietate de fenomene stiintifice, de la cele moleculare la cele astronomice:miscarea browniana a particulelor, turbulenta în fluide, cresterea plantelor, studiulrocilor, materiale compozite, polimeri si geluri, peisaje, munti, linii de coasta ge-ografice, distributia galaxiilor în univers si chiar fluctuatiile de preturi pe pietele deschimb. Ultimul eseu al lui Mandelbrot se refera la aceste fluctuatii.Multimile fractale joaca un rol important în unele ramuri ale matematicii, cum

ar fi teoria numerelor si ecuatiile diferentiale neliniare (stabilitate si haos).Astfel, ceea ce a parut la început un concept de matematica pura a gasit nu-

meroase aplicatii în stiinte. Daca aceste aplicatii vor continua sa se diversifice sisa se adânceasca, s-ar putea ca studiul fractalilor sa devina o parte obligatorie acursurilor universitare. Geometria fractala este completarea care lipsea geometrieieuclidiene si simetriei cristaline (sau cvasicristaline).

Bibliografie1. K. Devlin - Vârsta de aur a matematicii, Theta, Bucuresti, 2000.2. J. -F. Gornyet - Physique et structures fractales, Masson, Paris, 1992.3. B. Mandelbrot - The Fractal Geometry of Nature, W.H.Freeman, 1982.4. B. Mandelbrot - Obiectele fractale, Editura Nemira, 1998.

5

Page 10: Revista (format .pdf, 1.6 MB)

Dreapta lui Euler privita ca loc geometricGabriel POPA1, Paul GEORGESCU 2

Vom arata în cele ce urmeaza ca dreapta lui Euler a unui triunghi poate fi gân-dita ca locul geometric al punctelor de concurenta a trei ceviene variabile asociatetriunghiului.Fie O, G, H punctele remarcabile (în no-

tatii uzuale) ale triunghiului ABC, A1, B1, C1punctele de pe cercul circumscris diametral opusevârfurilor notate corespunzator, iar SA, SB, SCpuncte pe dreptele A1M , B1N respectiv C1P(undeM , N , P sunt mijloacele laturilor) care îm-part segmentele orientate A1M , B1N , C1P într-

un acelasi raport k ∈ R\½1,4

3

¾.

Teorema. Dreptele ASA, BSB si CSC suntconcurente într-un punct situat pe dreapta luiEuler a triunghiului ABC.Demonstratie. Raportam planul la un reper cu originea în centrul cercului

circumscris O; vom nota cu rX vectorul de pozitie al punctului X. Atunci

rSA =1

1− k(rA1 − krM ) =

1

k − 1·rA +

k

2(rB + rC)

¸.

Fie Q un punct pe dreapta ASA care împarte segmentul orientat ASA în raportulQA

QSA= l; avem

rQ =1

1− l(rA − lrSA) =

1

1− lrA − l

(1− l) (k − 1)·rA +

k

2(rB + rC)

¸=

=1 + l − k

(1− l) (1− k)rA +

lk

2 (1− l) (1− k)rB +

lk

2 (1− l) (1− k)rC .

Vom încerca sa determinam l ∈ R\ 1 astfel încât rQ sa aiba o scriere simetrica înraport cu rA, rB, rC . Pentru aceasta,

2 (1 + l − k) = lk ⇔ l (k − 2) = 2− 2k ⇔ l =2− 2kk − 2 (1)

unde l = 1 daca si numai daca k =4

3. Pentru valoarea lui l data de (1), obtinem

rQ =k

3k − 4 (rA + rB + rC) (2)

Considerând acum punctele Q0 pe BSB si Q00 pe CSC care împart segmenteleorientate corespunzatoare în acelasi raport l dat de (1), va rezulta ca rQ0 = rQ00 = rQ,deci cele trei puncte coincid. Prin urmare, exista un punct comun dreptelor ASA,BSB si CSC .

1 Profesor, Liceul Teoretic ”Garabet Ibraileanu”, Iasi2 Profesor, Liceul de Informatica ”Grigore C. Moisil”, Iasi

6

Page 11: Revista (format .pdf, 1.6 MB)

Deoarece rO = 0 si rG =1

3(rA + rB + rC), obtinem ca

−→OG =

1

3(rA + rB + rC).

Atunci punctul Q dat de (2) se afla pe dreapta OG, care este dreapta lui Euler atruinghiului ABC.

Observatia 1. Pentru k =4

3, dreptele ASA, BSB si CSC nu sunt concurente,

întrucât sunt toate paralele cu dreapta lui Euler. Într-adevar, în acest caz SA ar fisimetricul lui A1 fata de H, deoarece punctele H, M si A1 sunt coliniare si M estemijlocul segmentului [HA1]. Rezulta de aici ca HO este linie mijlocie în triunghiulA1SAA, deci HOkASA.Cazul k = 1 trebuie evident exclus, întrucât el corespunde punctelor ”de la infinit”

de pe dreptele A1M , B1N , respectiv C1P .

Observatia 2. Câteva cazuri particulare remarcabile:• k = 0 conduce la obtinerea punctului O drept punct de concurenta a diametrilor

AA1, BB1, CC1;• k = 2 corespunde situatiei în care SA = SB = SC = H; evident atunci ca

punctul de concurenta Q dat de (2) este chiar H-ortocentrul triunghiului;• k = −2 corespunde situatiei în care SA = GA este centrul de greutate al

triunghiului A1BC etc. În acest caz, punctul de concurenta Q are vectorul de pozitie

rQ =1

5(rA + rB + rC) deci

QG

QO=2

3. Dealtfel, articolul de fata a pornit tocmai de

la acest caz, autorii demonstrând ca OGAkAΩ, unde Ω este centrul cercului celornoua puncte;• k = 4 corespunde situatiei în care SA este simetricul lui GA în raport cu M si

analoagele; în acest caz, punctul de concurenta Q este Ω, centrul cercului celor nouapuncte.

Observatia 3. Orice punct de pe dreapta lui Euler, cu exceptia centrului degreutate G, poate fi gândit ca un punct de concurenta Q obtinut pentru un anumitraport k. Mai precis, pe dreapta cu orientarea OH avem:• k ∈ (0, 1), deci SA se afla pe semidreapta opusa lui (A1M si analoagele; atuncik

3k − 4 ∈ (−1, 0) si se obtin astfel punctele de pe dreapta lui Euler dintre O si

simetricul lui H fata de O;

• k ∈ (−∞, 0], deci SA apartine segmentului (MA1] si analoagele; atuncik

3k − 4 ∈

∈·0,1

3

¶si se obtin astfel punctele de pe segmentul (GO];

• k ∈µ1,4

3

¶∪µ4

3,∞¶, deci SA se afla pe semidreapta opusa lui (MA1 si

analoagele; atuncik

3k − 4 ∈ (−∞,−1) ∪µ1

3,∞¶si se obtin punctele semidreptei

(GH pentru k ∈µ4

3,∞¶si cele ale semidreptei ramase pentru k ∈

µ1,4

3

¶.

7

Page 12: Revista (format .pdf, 1.6 MB)

Un mod de calcul al numarului lexicografic de ordineasociat unei permutari

Iuliana GEORGESCU 1, Paul GEORGESCU 2

În cele ce urmeaza vom studia câteva proprietati ale relatiei de ordine lexicograficape multimea Sn a permutarilor de ordinul n si vom preciza un mod de calcul alnumarului lexicografic de ordine asociat.Dupa cum este cunoscut, relatia de ordine lexicografica pe multimea Sn se de-

fineste în modul urmator.

Definitie. Date σ si σ0 ∈ Sn, vom spune ca σ precede lexicografic σ0 si vom notaσ ≺ σ0 daca ∃ i0 ∈ 1, n astfel încât σ (i) = σ0 (i), ∀i ∈ 1, i0 − 1, iar σ (i0) < σ0 (i0).În raport cu aceasta relatie de ordine, fiecarei permutari din Sn i se va putea

asocia un numar de ordine cuprins între 1 si n!, care va fi numit în cele ce urmeazanumarul lexicografic de ordine al acelei permutari. De asemenea, daca σ ∈ Sn,notam cu mσ (i) numarul de inversiuni corespunzatoare pozitiei i în permutarea σsi cu sσ (i) = σ (i+ 1) , σ (i+ 2) , . . . σ (n) multimea elementelor care-l succed peσ (i) în permutarea σ.

Lema 1. Daca σ, σ0 ∈ Sn, atunci are loc echivalenta σ ≺ σ0 ⇔ ∃i0 ∈ 1, n în asafel încât mσ (i) = mσ0 (i), ∀i ∈ 1, i0 − 1, iar mσ (i0) < mσ0 (i0).Demonstratie. „⇒” Fie i0 ∈ 1, n în asa fel încât σ (i) = σ0 (i), ∀i ∈ 1, i0 − 1,

iar σ (i0) < σ0 (i0). În mod evident, mσ (i) = mσ0 (i), ∀i ∈ 1, i0 − 1, pentru casσ (i) = sσ0 (i), ∀i ∈ 1, i0 − 1, iar mσ (i0) < mσ0 (i0) (prin trecerea pe pozitia i la unelement mai mare se „câstiga” inversiuni).„⇐” Fie i0 ∈ 1, n în asa fel încât mσ (i) = mσ0 (i), ∀i ∈ 1, i0 − 1, iar mσ (i0) <

< mσ0 (i0). Deoarecemσ (1) = mσ0 (1), se obtine ca σ (1) = σ0 (1), stiind camσ (1) == n−σ (1), mσ0 (1) = n−σ0 (1) . Cu un rationament asemanator se obtine ca σ (i) == σ0 (i), ∀i ∈ 1, i0 − 1, si deci sσ (i0 − 1) = sσ0 (i0 − 1). Deoarece mσ (i0) < mσ0 (i0),rezulta evident ca σ (i0) < σ0 (i0).Observatia 1. Evident, mσ (i) ≤ n− i. De asemenea, are loc inegalitatea

nPi=k

mσ (i) (n− i)! ≤nPi=k

(n− i) (n− i)! si de aici

nPi=k

mσ (i) (n− i)! ≤nPi=k

[(n− i+ 1)!− (n− i)!] = (n− k + 1)!− 1,∀k ∈ 1, n.

Notam N (σ) = 1+nPi=1

mσ (i) (n− i)!. Conform observatiei anterioare,

1 ≤ N (σ) ≤ n! .

Lema 2. Are loc echivalenta σ ≺ σ0 ⇔ N (σ) < N (σ0).Demonstratie. „⇒” Fie i0 ∈ 1, n în asa fel încât σ (i) = σ0 (i), ∀i ∈ 1, i0 − 1,

iar σ (i0) < σ0 (i0). Atunci N (σ0)−N (σ) = [mσ0 (i0)−mσ (i0)] (n− i0)! +

+nP

i=i0+1[mσ0 (i)−mσ (i)] (n− i)! ≥ (n− i0)! −

nPi=i0+1

mσ0 (i) (n− i)!.

Conform Observatiei 1, N (σ0)−N (σ) ≥ 1.1 Profesoara, Liceul cu Program Sportiv, Iasi2 Profesor, Liceul de Informatica ”Grigore Moisil”, Iasi

8

Page 13: Revista (format .pdf, 1.6 MB)

„⇐” Presupunem prin reducere la absurd ca σ0 ¹ σ. Daca σ0 = σ, atunci N (σ) == N (σ0), absurd. Daca σ0 ≺ σ, atunci N (σ) < N (σ0), conform celor demonstrateanterior, ceea ce este absurd.Fie acum F1 : Sn → 1, n!, F1 (σ) = N (σ). Conform Lemei 2, F1 este injectiva.

Deoarece card Sn = n! =card 1, n!, F1 este bijectiva, si deoarece F1 pastreaza or-dinea lexicografica (Lema 2 ), N (σ) este chiar numarul lexicografic de ordine asociatpermutarii σ. Se obtine deci urmatoareaTeorema. Numarul lexicografic de ordine asociat unei permutari σ este

N (σ) = 1 +nPi=1

mσ (i) (n− i)! .

Vom propune în continuare câteva aplicatii ale acestui rezultat.Problema 1. Fie σ, σ0 ∈ Sn în asa fel incât mσ (i) = mσ0 (i), ∀i ∈ 1, n.

Demonstrati ca σ = σ0.Solutie. Se observa ca N (σ) = N (σ0), deci σ = σ0.

Fie Mn = 0, n− 1× 0, n− 2× . . .× 0.Corolar. Functia F2 : Sn → Mn, F2 (σ) = (mσ (1) ,mσ (2) , . . . ,mσ (n)) este

injectiva.Demonstratie. Evident, cardSn = cardMn = n!, iar F2 este injectiva conform

Lemei 2, deci în fapt este bijectiva.Problema 2. Demonstrati ca pentru orice n-upla (k1, k2, . . . , kn) ∈ Nn astfel

încât 0 ≤ ki ≤ n− i, ∃σ ∈ Sn satisfacând mσ (i) = ki, ∀i ∈ 1, n.Solutie. Rezulta imediat din surjectivitatea lui F2.Fie acum n ∈ N∗, n ≥ 2, si k ∈ N, 1 ≤ k ≤ n. Notam cu Ak

n multimeatuturor aranjamentelor de câte n elemente luate câte k. Cu aceasta notatie, fiecaruiaranjament A ∈ Ak

n, privit ca o functie de la multimea 1, k la multimea 1, n, i sepoate asocia în mod natural multimea M (A) ce contine toate permutarile σ ∈ Sncu proprietatea ca σ (i) = A (i), ∀i ∈ 1, k, iar σ (i) ∈ 1, n\A (1) , A (2) , . . . , A (k),∀i ∈ k + 1, n. Evident, M (A) contine (n− k)! elemente cu numere lexicografice deordine asociate consecutive.Pentru i ∈ 1, k, notam cu nA (i) numarul elementelor din multimea 1, n\A (1) ,

A (2) , . . . , A (i) care sunt mai mari decât i. Evident, pentru n = k, nA (i) = mA (i).Relatia de ordine lexicografica definita pe multimea Sn se poate extinde în mod

evident si la multimea Akn. Aplicând rationamentul descris anterior, obtinem ca

numarul lexicografic de ordine asociat unui aranjament A este

NA = 1+kPi=1

mσ (i) (n− i)!/ (n− k)! .

Observatia 2. Împreuna cu schita unei alte demonstratii, Teorema este mentio-nata în [1] si preluata în [2], fara demonstratie si fara indicarea sursei originale.

Bibliografie.1. P. Georgescu - Asupra relatiei de ordine lexicografica pe multimea permutarilorde ordinul n, Revista „Joc Secund”, Liceul de Informatica „G.Moisil”, Iasi, ianuarie2001.

2. D. Hrinciuc-Logofatu - C++. Algoritmi si probleme rezolvate, Editura Polirom,Iasi, 2001.

9

Page 14: Revista (format .pdf, 1.6 MB)

Generalizari ale teoremei lui Ceva si aplicatiiTemistocle BÎRSAN 1

Teorema lui Ceva si reciproca sa caracterizeaza concurenta a trei ceviene cu aju-torul rapoartelor în care picioarele acestora împart laturile triunghiului. În loculcevienelor vom considera trei drepte oarecare si vom exprima concurenta lor în ace-lasi mod: cu ajutorul rapoartelor în care punctele de intersectie a dreptelor cu laturiletriunghiului împart aceste laturi. Diversitatea pozitiilor celor trei drepte în raport cutriunghiul dat face posibile mai multe generalizari ale teoremei lui Ceva. Utilizareasegmentelor orientate este avantajoasa în gruparea cazurilor ce pot aparea. Ca si în[2], se exclud tacit unele pozitii triviale ale celor trei drepte.Vom începe cu doua rezultate pregatitoare.Lema 1. Fie ABC un triunghi oarecare si punctele U ∈ AB, V ∈ AC, X ∈ BC

si Y ∈ AC. Notam u =UB

UA, v =

V C

V A, x =

XB

XC, y =

Y A

Y Csi T = UV ∩ XY .

Atunci, avem:TU

TV=(1− v) (x− uy)

(1− u) (1− vy).

Demonstratie. Fie X 0 = AB ∩ XY (fig. 1). Teorema lui Menelaus aplicatatriunghiului AUV si secantei XY conduce la relatia

TU

TV=

Y A

Y V· X

0UX 0A

. (1)

Fig. 1 Fig. 2 Fig. 3

Cum y =Y A

Y C⇒ Y A =

y

1− yAC si v =

V C

V A⇒ AV =

1

1− vAC, urmeaza ca

Y V = Y A+AV =1− vy

(1− v) (1− y)AC si, deci,

Y A

Y V=

y (1− v)

1− vy(2).

Pozitia punctului X 0 pe latura AB este precizata de raportulX 0AX 0B

=y

x(teorema

lui Menelaus relativ la 4ABC si secanta XY ). Deducem ca X 0A =y

x− yAB si cum

u =UB

UA⇒ AU =

1

1− uAB, obtinem: X 0U = X 0A+AU =

x− uy

(1− u) (x− y)AB. Ca

urmare,X 0UX 0A

=x− uy

y (1− u)(3).

Combinând (1), (2) si (3), obtinem relatia de demonstrat.

1 Prof. dr., Catedra de matematica, Univ. Tehnica ”Gh. Asachi”, Iasi

10

Page 15: Revista (format .pdf, 1.6 MB)

Lema 2. Fie ABC un triunghi si punctele U ∈ AB, V ∈ AC, X ∈ BC si

y ∈ AB. Notând u =UB

UA, v =

V C

V A, x =

XC

XB, y =

Y A

Y Bsi T = UV ∩ XY

(fig. 2), avem:TU

TV=(1− v) (1− uy)

(1− u) (x− vy).

Observatia 1. Relatia din Lema 1 a fost demonstrata în cazul particular dinfig. 1: punctele U,V,X, Y sunt interioare laturilor pe care se afla, dar ramâne valabila(cu aceeasi demonstratie) si în cazurile în care o parte dintre aceste puncte sau toatear fi situate pe prelungirile laturilor respective. Doar cazul XY kAB (în care x = y)

necesita o demonstratie diferita; se ajunge la formulaTU

TV=

x (1− v)

1− vxcare se obtine,

de altfel, din cea prezenta în Lema 1 daca se ia în aceasta x = y. Aceleasi observatiise pot face si privitor la Lema 2.

Propozitia 1. Fie dat un triunghi ABC si dreptele d1, d2, d3. Presupunem cad1 intersecteaza AB si AC în punctele M si respectiv N , d2 intersecteaza BC siBA în P si respectiv Q, iar d3 intersecteaza CA si CB în R si respectiv S si notam

m =MB

MA, n =

NC

NA, p =

PC

PB, q =

QA

QB, r =

RA

RC, s =

SB

SC. Atunci d1, d2, d3 sunt

concurente daca si numai daca este îndeplinita conditia

1 +mpr + nqs = mq + ps+ rn, i.e.

¯¯ 1 q rm 1 sn p 1

¯¯ = 0.

Demonstratie. Fie T =MN ∩ SR si T 0 =MN ∩PQ (fig. 3). În conformi-tate cu Lemele 1 si 2, putem scrie:

TM

TN=(1− n) (s−mr)

(1−m) (1− nr),

T 0MT 0N

=(1− n) (1−mq)

(1−m) (p− nq).

Atunci, d1, d2, d3 sunt concurente ⇔ T coincide cu T 0 ⇔ (1− n) (s−mr)

(1−m) (1− nr)=

=(1− n) (1−mq)

(1−m) (p− nq)⇔ 1 +mpr + nqs = mq + ps+ rn, q.e.d.

Daca Q coincide cu A, S cu B, si N cu C, adica q = s = n = 0, obtinem:Corolarul 1 (Ceva). Cevienele AP , BR, CM (se exclude cazul APkBRkCM)

sunt concurente daca si numai daca 1 +mpr = 0 sau

¯¯ 1 0 rm 1 00 p 1

¯¯ = 0.

Daca S coincide cu B si P cu C, adica s = p = 0, vom obtine un alt rezultatcunoscut([4, Teorema 3], [3, Teorema 1], [2, Prop.2] etc.):Corolarul 2. Dreapta MN trece prin punctul de intersectie a cevienelor BR si

CQ daca si numai daca mq + nr = 1.

Aplicatia 1. Daca pentru k ∈ Z fixat avem: BS

ck=

SP

ak=

PC

bk,CN

ak=

NR

bk=

=RA

cksi

AQ

bk=

QM

ck=

MB

ak(fig.3), atunci MN , PQ, RS sunt concurente.

11

Page 16: Revista (format .pdf, 1.6 MB)

Demonstratie. Avem: m = n = − ak

bk + ck, p = q = − bk

ck + ak, r = s =

= − ck

ak + bk. Se verifica printr-un calcul simplu conditia din Propozitia 1.

Observatia 2. Pentru k luând valorile 0, 1 si 2 punctele de intersectie corespun-zatoare sunt G, I si respectiv K (Lemoine).Aplicatia 2 (Newton). Într-un patrulater circumscris unui cerc dreptele care

unesc punctele de contact ale laturilor opuse trec prin punctul de intersectie a diago-nalelor.Demonstratie. Sa aratam numai faptul ca XY , AC si BD sunt concurente

(fig. 4). Vom aplica Corolarul 2. Avem:

Fig. 4 Fig. 5 Fig. 6

m =XB

XE= −

µr ctg

B

2

¶/

µr tg

B + C

2

¶, n =

Y C

Y E= −

µr ctg

C

2

¶/

µr tg

B + C

2

¶,

q =AE

AB= − d sinD

a sin (B + C)si r =

DE

DC= − d sinD

c sin (B + C), unde a = AB, b = BC,

c = CD si d = DA. Cu acestea si tinând seama ca a = r

µctg

A

2+ ctg

B

2

¶etc. si ca

sinA+B

2= sin

C +D

2, cos

B +D

2= − cos A+ C

2, se verifica prin calcul conditia

mq + nr = 1.Propozitia 2. Daca pozitiile punctelor M , N , P , Q, R, S (fig. 5) sunt precizate

de rapoartele m =MB

MA, n =

NC

NA, p =

PB

PA, q =

QC

QA, r =

RB

RA, s =

SC

SB, atunci

dreptele MN , PQ, RS sunt concurente daca si numai daca avem

prs+mq + nr = mrs+ np+ rq, i.e.

¯¯ 1 1 1m p rn q rs

¯¯ = 0.

Demonstratie. Notam T = SR∩NM si T 0 = SR∩QP . Conform Lemei 1,avem:

TS

TR=(1− 1/r) (n−ms)

(1− s) (1−m/r)si

T 0ST 0R

=(1− 1/r) (q − ps)

(1− s) (1− p/r). Asadar, MN , PQ,

RS sunt concurente ⇔ TS

TR=

T 0ST 0R

⇔ (n−ms) (1− p/r) = (q − ps) (1−m/r) ⇔prs+mq + nr = mrs+ np+ rq.Procedând la fel, dar cu utilizarea Lemei 2, se dovedeste siPropozitia 20. Daca punctele M , N , P , Q, R, S (fig. 6) au pozitiile precizate

de rapoartele m =MB

MA, n =

NC

NA, p =

PB

PA, q =

QC

QA, r =

RB

RC, s =

SC

SA, atunci

12

Page 17: Revista (format .pdf, 1.6 MB)

dreptele MN , PQ, RS sunt concurente daca si numai daca avem

qrs+ms+ np = nrs+mq + ps, i.e.

¯¯ 1 1 1m p rsn q s

¯¯ = 0.

Corolarul 3. Ceviana AD, cu D ∈ BC precizat de raportul d =DC

DB, trece prin

punctul de intersectie a dreptelor MN si PQ daca si numai daca¯¯ 1 1 0m p 1n q d

¯¯ = 0, i.e. d =

n− q

m− p.

Demonstratie. În Propozitia 2 (sau 20) se face ca R sa coincida cu A (respectiv,S sa coincida cu A) si se ia s = d (respectiv, r = 1/d).

Aplicatia 3. Fie ABC un triunghi cu a ≤ b ≤ c. Fie D, E, F mijloacelelaturilor (BC), (CA) si respectiv (AB) si D0, E0, F 0 mijloacele liniilor frânte BAC,CBA si respectiv ACB (i.e. BD0 = CA + AD0, CB + BE0 = AE0 si AF 0 =BC + CF 0). Aratati ca dreptele DD0, EE0 si FF 0 sunt concurente (fig. 7).Demonstratie. Rezultatul se obtine aplicând Propozitia 2 cu n = p = s = −1,

m =E0BE0A

= −c− a

a+ c(caci 2AE0 = a+ c si BE0 = c−AE0 =

1

2(c− a)), q =

F 0CF 0A

=

= −b− a

a+ bsi r =

D0BD0A

= −b+ c

c− b.

Fig. 7 Fig. 8 Fig. 9Observatia 3. În [1] este data o demonstratie sintetica (se arata ca DD0, EE0,

FF 0 sunt bisectoare ale 4DEF ), iar în [5, Teorema 2] una analitica.

În legatura cu Problema 191 [6] (a se vedea si [2]) dam

Aplicatia 4. Fie ABC un triunghi ce nu este isoscel. Fie A0 punctul de in-tersectie a dreptei ce uneste picioarele bisectoarelor unghiurilor B si C cu dreaptace uneste punctele de contact al cercului înscris cu laturile (AB) si (AC) si B0, C0analoagele acestuia. Sa se arate ca dreptele AA0, BB0, CC0 sunt concurente.

Demonstratie. Cu notatiile din fig. 8, avem: m = −ab, n = −a

c, p = − p− b

p− a=

= −a+ c− b

b+ c− asi q = − p− c

p− a= −a+ b− c

b+ c− a. În conformitate cu Corolarul 3, obtinem:

d =DC

DB=

n− q

m− p= . . . =

b (a− c) (a− b+ c)

c (a− b) (a+ b− c). În acelasi mod obtinem si

e =EA

EC=

c (b− a) (a+ b− c)

a (b− c) (−a+ b+ c), f =

FB

FA=

a (c− b) (−a+ b+ c)

b (c− a) (a− b+ c). Ca urmare,

def = −1 si, deci, dreptele AA0, BB0, CC0 sunt concurente.13

Page 18: Revista (format .pdf, 1.6 MB)

Propozitia 3. Fie D,M,P ∈ BC, Q ∈ AB si N ∈ AC (fig. 9) cu pozitiile

precizate de d =DB

DC, m =

MB

MC, p =

PC

PB, q =

QB

QAsi n =

NC

NA. Dreapta AD trece

prin punctul de intersectie a dreptelor MN si PQ daca si numai daca¯¯ mn q d

n pq 11 1 0

¯¯ = 0, i.e. d =

mn− q

n− pq.

Demonstratie. Utilizam Propozitia 2 luând punctul N în vârful C si înlocuindnumerele m,n, p, q, r, s cu 1/d, 0, p, 1/q, 1/m, 1/n respectiv. Rezultatul dorit poate fiobtinut, în mod asemanator, si prin utilizarea Propozitiei 20.Observatie. Teorema lui Ceva este un caz particular al Propozitiilor 2, 20, 3.Aplicatia 5. Se noteaza cu A1 centrul patratului D1D2D3D4 înscris în tri-

unghiul ABC cu D1,D2 ∈ BC, D3 ∈ CA si D4 ∈ AB. Similar se introduc sipunctele B1, C1. Sa se arate ca dreptele AA1, BB1, CC1 sunt concurente (punctullui Van Vecten).Demonstratie. Aplicam Propozitia 3 luând drept MN si PQ diagonalele pa-

tratului D1D2D3D4. Notând cu l latura acestuia, avem: m =D1B

D1C= − l ctgB

l + l ctgC,

p =D2C

D2B= − l ctgC

l + l ctgBsi n = q. Conditia din propozitia amintita se scrie: d =

m− 11− p

= . . . = −1 + ctgB1 + ctgC

. Relativ la laturile (CA) si (AB) obtinem

e = −1 + ctgC1 + ctgA

, f = −1 + ctgA1 + ctgB

. Rezulta ca def = −1 si, deci, dreptele AA1,

BB1, CC1 sunt concurente.

Aplicatia 6. Fie Db, Dc punctele de contact al cercului A - exînscris triunghiuluiABC cu dreptele AB si respectiv AC, iar perechile de puncte Ec, Ea si Fa, Fb avândsemnificatii analoage. Daca notam A0 = EcEa ∩ FbFa, B0 = FaFb ∩ DcDb

si C 0 = DbDc ∩ EaEc, atunci dreptele AA0, BB0, CC0 sunt concurente în H-ortocentrul triunghiului ABC.Demonstratie. Daca D = AA0 ∩ BC, se aplica Propozitia 3 pentru a arata

ca punctul D este piciorul înaltimii din A a triunghiului ABC etc.

Bibliografie1. F. D. Beznosikov - Problema 914, Matematika v skole, 3/1971, p.75 (enunt) si1/1972, p.76 (solutie).

2. T. Bîrsan - Un criteriu de concurenta a dreptelor, Recreatii Matematice, 4(2002),nr.1, 23-24.

3. C. Chiser - Conditii necesare si suficiente ca o dreapta sa treaca prin puncte im-portante dintr-un triunghi, G.M.-9/2000, 336-343.

4. N. Oprea - Un punct si o dreapta remarcabile din planul unui triunghi, G.M.-11/1996, 532-540.

5. J. Tong - Perimeter bisectors of the triangle, G.M.-3/1998 (seria informare - per-fectionare), 162-176.

6. *** - Problema 191, Recreatii Stiintifice, 4(1886), p.48 (enunt) si p.118 (solutie).

14

Page 19: Revista (format .pdf, 1.6 MB)

O proprietate a sistemelor de generatoriale unor grupuriMarian TETIVA1

În Problema C:316 [1] se cere sa se demonstreze ca din orice sistem de generatori algrupului aditiv al numerelor reale se poate extrage orice element, multimea ramasafiind, în continuare, sistem de generatori. Problema 26 din [3, p.55] arata ca deaceeasi proprietate se bucura si sistemele de generatori ale grupului (Q,+). Amintimca, dat fiind un grup abelian (aditiv) G, o multime nevida S ⊆ G se numeste sistemde generatori al grupuluiG daca, pentru orice g ∈ G, exista n ∈ N∗, a1, a2, . . . , an ∈ Zsi s1, s2, . . . , sn ∈ S astfel încât g = a1s1 + a2s2 + . . . + ansn. Remarcam ca oricegrup G are macar un sistem de generatori (si anume pe S = G). E usor de vazutca, daca un element s ∈ S are o scriere de acest tip cu toate elementele si ∈ S\s,atunci S\s este, de asemenea, sistem de generatori pentru G.Ne propunem în cele ce urmeaza sa aratam ca ambele probleme pot fi rezolvate în

acelasi fel, tinând seama de o proprietate comuna a grupurilor aditive ale numerelorreale, respectiv rationale si de urmatorul rezultat, pe care îl demonstram în aceastanota.

Propozitie. Fie G un grup divizibil, S un sistem de generatori al lui G si s ∈ S.Atunci S\s este, înca, sistem de generatori pentru G.

Vom considera grupul G abelian si dat în notatie aditiva; în aceste conditii Gse numeste grup divizibil daca pentru orice g ∈ G si orice numar întreg n 6= 0exista x ∈ G astfel încât nx = g (vom mai spune ca ”ecuatia” nx = g are solutiein G). Evident, grupurile (R,+) si (Q,+), despre care a fost vorba în problemeleamintite mai sus, sunt grupuri divizibile. Un exemplu de grup care nu este divizibilîl reprezinta grupul (Z,+) al numerelor întregi (de exemplu, ecuatia 2x = 1 nu aresolutie în Z). În continuare dam doua demonstratii ale propozitiei enuntate, unaelementara, alta care face apel la o teorema de izomorfism.Demonstratia I. Sa consideram multimea H a ”combinatiilor liniare”, adica a

elementelor din G care pot fi scrise în forma

a1s1 + a2s2 + . . .+ ansn,

unde n ∈ N∗, a1, a2, . . . , an ∈ Z si s1, s2, . . . , sn ∈ S\s; de asemenea, fie K =ns | n ∈ Z. Se verifica imediat ca H si K sunt subgrupuri ale lui G (ele se numescsubgrupurile lui G generate de S\s, respectiv de s). Trebuie sa aratam ori cas ∈ H (asa vom proceda în aceasta prima demonstratie), ori ca H = G (ceea ce vomface în a doua demonstratie).Aratam, mai întâi, ca exista un ”multiplu” qs, q ∈ Z∗, al lui s care apartine lui H.

Într-adevar, conform ipotezei, exista x ∈ G astfel încât 2x = s; tot conform ipotezei,x poate fi exprimat ca o combinatie liniara a unor elemente din S, adica existaa, a1, . . . , ap ∈ Z si exista s1, . . . , sp ∈ S\s astfel încât x = as + a1s1 + . . . apsp.Înlocuim aici s = 2x si obtinem egalitatea (1− 2a)x = a1s1 + . . . + apsp care arataca (1− 2a)x ∈ H; evident , 1− 2a este un numar întreg nenul (este impar).1 Profesor, Colegiul National ”Gheorghe Rosca Codreanu”, Bârlad

15

Page 20: Revista (format .pdf, 1.6 MB)

Deci exista q ∈ Z, q 6= 0 astfel încât qs ∈ H. Cum orice x ∈ G are o scriere deforma x = bs+ b1s1 + . . .+ brsr, cu b, b1, . . . , br ∈ Z si s1, s2, . . . , sr ∈ S\s, rezultaca, pentru orice x ∈ G, avem

qx = b (qs) + (qb1) s1 + . . .+ (qbr) sr ∈ H;

asadar avem qx ∈ H pentru orice x ∈ G. Dar, grupul G fiind divizibil si q fiindun întreg nenul, exista x0 ∈ G astfel încât qx0 = s; atunci s = qx0 ∈ H (ceea ceînseamna ca s este o combinatie liniara a unor elemente din S\s), de unde concluziaca si S\s este sistem de generatori pentru G este imediata.Demonstratia a II-a. Se bazeaza, în fond, pe aceleasi idei, dar, dupa cum am

spus, este formulata într-un limbaj mai abstract (v.[3]).Se observa ca avem G = H +K (din ipoteza ca S este un sistem de generatori

al lui G); conform unei teoreme de izomorfism rezulta ca G/H = (H +K) /H esteizomorf cu K/ (H ∩K).Nu e greu de vazut ca proprietatea unui grup de a fi divizibil se transmite prin

morfisme surjective (adica, daca G este un grup divizibil si f : G → G0 un morfismsurjectiv de grupuri, atunci si G0 este un grup divizibil), cu atât mai mult prin izomor-fisme. De asemenea, orice grup factor al unui grup divizibil este divizibil (deoareceîntre un grup si un grup factor al sau exista întotdeauna un morfism surjectiv -surjectia canonica).Revenind la problema, din observatiile de mai sus deducem ca grupul K/ (H ∩K)

este divizibil. Acest grup este însa ciclic (deoarece K este astfel, prin însasi definitialui, si orice grup factor al unui grup ciclic este tot ciclic) si, dintre grupurile ciclice,divizibil este numai grupul nul. Atunci rezulta ca G/H (izomorf cu K/ (H ∩K))este grupul nul, deci H = G (adica orice element din G este o combinatie liniara deelemente din S\s), ceea ce încheie demonstratia.(Un grup ciclic finit C, cu n ≥ 2 elemente, nu este divizibil deoarece nx = 0

pentru orice x ∈ C si în C mai exista si alte elemente în afara de 0; de altfel, aceeasiobservatie se poate face pentru un grup finit oarecare. În cazul în care este infinit,C este izomorf cu Z, despre care am aratat deja ca nu este divizibil.)

În încheiere mentionam ca o alta demonstratie elementara poate fi gasita în [2] lapagina 95, iar ca observatie finala sa mai spunem ca o consecinta imediata a propozi-tiei demonstrate este ca orice sistem de generatori al unui grup divizibil trebuie safie infinit (în particular, orice sistem de generatori al lui (Q,+) sau al lui (R,+) esteinfinit).

Bibliografie1. T. Albu - Problema C:316, G.M. - 6/1983, p.256.2. T. Albu, I. Ion - Itinerar elementar în algebra superioara, Editura All Educational,Bucuresti, 1997.

3. I. Ion, N Radu - Algebra, E.D.P., Bucuresti, 1991.

16

Page 21: Revista (format .pdf, 1.6 MB)

Siruri recurente de tipul xn+1 = f (n, xn)Dan POPESCU 1

În Recreatii Matematice, nr.1/2002, am abordat sirurile definite recurent prinxn+1 = f (xn), n ∈ N∗. Mai general, cu un instrument de lucru asemanator - înprincipal, teorema Stolz-Cesàro - pot fi studiate sirurile recurente date de xn+1 == f (n, xn), n ∈ N∗, x1 ∈ D, unde f : N∗ × D → D este o functie oarecare. Vomjustifica prin exemple aceasta afirmatie.1 (G.M.-12/1994, 23145, Gh. Marchitan). Fie sirul de numere reale pozitive

(xn)n≥1 dat de x1 > 0 si x2n+1 = n (xn − xn+1), n ∈ N∗. Sa se arate ca lim

n→∞nxn = e.

Solutie. Deoarece (xn)n≥1 este sir descrescator de numere pozitive, existalimn→∞xn = l ∈ [0, x1). Sumând egalitatile x2k+1 = k (xk − xk+1), k = 1, n− 1, mem-bru cu membru, obtinem relatia x22+x

23+...+x

2n

n−1 = x1+x2+...+xn−1n−1 −xn, n ≥ 2. Deoarece

xn →nl ⇒ x1+x2+...+xn−1

n−1 →nl si x2n →n l2 ⇒ x22+x

23+...+x

2n

n−1 →nl2, trecând la limita în

relatia precedenta obtinem l = 0.Cum lim

n→∞1

xn lnn= limn→∞

1/xn+1−1/xnln(1+1/n) = lim

n→∞xn+1/xn

n ln(1+1/n)= limn→∞

nxn+1+n

· 1ln(1+1/n)n =1,

rezulta ca limn→∞nxn = lim

n→∞ exn lnn = e.

2 (G.M.-2/1997, p.58, V. Nicula). Fie sirul (xn)n≥1 definit prin x1 > 0 sixn+1 =

nxnn+x2n

, n ∈ N∗. Sa se arate ca acest sir este convergent si sa se calculezelimita sa.Solutie. Observând ca (xn)n≥1 este descrescator si xn > 0, n ∈ N∗, exista

limn→∞xn = l ∈ [0, x1). Atunci l = lim

n→∞xn = limn→∞

nxnn = lim

n→∞ [(n+ 1)xn+1 − nxn] =

= limn→∞xn+1

£(n+ 1)− ¡n+ x2n

¢¤= lim

n→∞xn+1¡1− x2n

¢= l

¡1− l2

¢si, deci, l = 0.

3 (Recreatii Matematice - 1/2000, XI.9, Gh. Iurea). Consideram sirul (xn)n≥1definit prin x1 ∈ (0, 1) si xn+1 = xn−nx2n, n ≥ 1. Aratati ca lim

n→∞ (1 + nxn)n = e2.

Solutia este data în Recreatii Matematice - 1/2001, p.69.Propunem cititorului câteva exercitii de felul celor prezentate mai sus.4 (G.M.-10,11,12/1990, 22216, D. M. Batinetu - Giurgiu). Fie sirul (xn)n≥1

definit prin x1 > 0 si xn+1 = xn1+anx2n

, n ∈ N∗, unde a > 0 este fixat. Sa se arate casirurile (xn)n≥1 si (nxn)n≥1 sunt convergente si sa se determine limitele lor.5 (G.M.-4/1991, 22344, D. Popescu). Daca x1 ∈ (0, 1) si xn+1 = xn

(1+xn)xn+1 ,

n ∈ N∗, sa se calculeze limn→∞

n√xn.

6 (G.M.-9,10,11,12/1992, C:1330, D. Popescu). Daca sirul (xn)n≥1 este definit

prin x1 ∈ R si xn+1 = 1n

p1 + x2n, n ≥ 1, sa se determine lim

n→∞ (nxn)1/xn.

7 (G.M.-2/2000, C:2254, T. Tamâian). Fie sirul (xn)n≥1 dat prin x1 > 0 sixn+1 = xn +

axn, n ≥ 1, unde a > 0 este fixat. Sa se calculeze lim

n→∞¡xn −

√2an

¢.

8 (R.M.T.-1/2002, XI.108, R. Satnoianu). Fie a ∈ (0, 1], x0 ∈ (0, a] si xn+1 == axne

−xn , n ∈ N. Sa se arate ca sirurile (xn)n≥0 si (nxn)n≥0 au limite, care secer precizate.

1 Profesor, Colegiul National ”Stefan cel Mare”, Suceava

17

Page 22: Revista (format .pdf, 1.6 MB)

Proprietati ale punctului lui Nagelstabilite cu ajutorul numerelor complexe

Constantin ANTON 1

Vom prezenta câteva proprietati ale punctului si dreptei lui Nagel folosind cainstrument de lucru numerele complexe. O prezentare sintetica a acestor proprietatise poate gasi în [1]. Utilizarea numerelor complexe face ca demonstratiile date sa fiefoarte simple.Fie ABC un triunghi oarecare. Notam cu A0 mijlocul laturii (BC) si cu D,D0

punctele de contact ale acesteia cu cercurile înscris si A-exînscris triunghiului dat.Pe laturile (CA) si (AB) consideram punctele B0, E,E0 si respectiv C0, F, F 0 cusemnificatii analoage. Sunt cunoscute (sau se deduc usor) urmatoarele relatii:

BD0 = p− c,D0C = p− b; CE0 = p−a,E0A = p−c; AF 0 = p− b, F 0B = p−a. (1)Convenim ca afixul unui punct X oarecare sa fie notat cu zX .

Propozitia 1. Dreptele AD0, BE0, CF 0 sunt concurente într-un punct N cuafixul dat de

zN =1

p[(p− a) zA + (p− b) zB + (p− c) zC ] . (2)

Demonstratie. Tinând seama de (1), avem k =BD0

D0C=

p− c

p− b, deci zD0 =

=zB + kzC1 + k

=1

a[(p− b) zB + (p− c) zC ]; se obtin formule similare pentru afixele

punctelor E0 si F 0. Fie V un punct pe segmentul (AD0) determinat de raportul

v =AV

V D0 . Avem

zV =zA + vzD0

1 + v=

1

1 + v

·1

p− a(p− a) zA +

v

a(p− b) zB +

v

a(p− c) zC

¸.

Vom obtine o forma simetrica pentru paranteza patrata alegând v astfel încât1

p− a=

=v

a, adica v =

a

p− a. Atunci, punctul de pe AD0 corespunzator acestei valori a lui v,

punct pe care-l notam cuN , va avea afixul zN=1

p[(p− a)zA+ (p− b)zB+ (p− c)zC ].

Simetria acestei relatii face evident faptul ca punctul N este situat si pe dreptele BE0

si CF 0. Demonstratia este completa.

Punctul N pus în evidenta de Propozitia 1 se numeste punctul lui Nagel. Propri-etati remarcabile ale acestuia sunt date în propozitiile urmatoare.

Propozitia 2. Punctul lui Nagel are proprietatile:

a) G ∈ (IN) si NG = 2GI, b) NHkOI si NH = 2OI.

1 Elev, clasa a X-a, Colegiul National ”C. Negruzzi”, Iasi

18

Page 23: Revista (format .pdf, 1.6 MB)

Demonstratie. a) Pentru afixele punctelor G si I avem

zG =1

3(zA + zB + zC) , zI =

1

2p(azA + bzB + czC) (3)

(acestea pot fi deduse asa cum s-a procedat pentru zN în Propozitia 1).

Datorita relatiilor (2) si (3), vom avea:

zG − zIzN − zG

=

1

3(zA + zB + zC)− 1

2p(azA + bzB + czC)

1

p[(p− a) zA + (p− b) zB + (p− c) zC ]− 1

3(zA + zB + zC)

=

=1

2· 2p (zA + zB + zC)− 3 (azA + bzB + czC)

3 [(p− a) zA + (p− b) zB + (p− c) zC ]− p (zA + zB + zC)=1

2,

decizG − zIzN − zG

=1

2∈ R, de unde rezulta ca G ∈ (IN) si |zN − zG| = 2 |zG − zI |,

adica NG = 2GI.b)Alegem un sistem de axe cu originea înO, centrul cercului circumscris triunghiu-

lui ABC. În acest caz, stim ca zH = zA + zB + zC si vom avea:

zN − zHzI − zO

=

1

p[(p− a) zA + (p− b) zB + (p− c) zC ]− (zA + zB + zC)

1

2p(azA + bzB + czC)

= −2 ∈ R,

deci NHkOI si |zN − zH | = 2 |zI − zO| , adica NH = 2OI.

În geometria triunghiului, dreapta IN se numeste dreapta lui Nagel.

Propozitia 3. Cercul înscris în triunghiul median A0B0C0 are centrul în mijloculS al segmentului (IN).

Demonstratie. Evident, avem B0C 0 =a

2, C 0A0 =

b

2, A0B0 =

c

2si p0 =

1

2p,

p0 fiind semiperimetrul triunghiului median. Atunci, conform cu (3),

zI0 =1

2p0

µa

2zA0 +

b

2zB0 +

c

2zC0

¶=1

2p

·azB + zC2

+ bzC + zA2

+ czA + zB2

¸=1

4p[(b+ c) zA + (c+ a) zB + (a+ b) zC ] .

Pe de alta parte,

zS =1

2(zI + zN )=

1

2

·1

2p(azA + bzB + czC)+

1

p[(p− a) zA + (p− b) zB + (p− c) zC ]

19

Page 24: Revista (format .pdf, 1.6 MB)

=1

4p[(2p− a) zA+(2p− b) zB+(2p− c) zC ]=

1

4p[(b+ c) zA+(c+ a) zB+(a+ b) zC ].

Ca urmare, zI0 = zS si deci I 0 coincide cu S.

Punctul S, mijlocul segmentului (IN), se numeste punctul lui Spiecker, iar cercul

C³S,

r

2

´, înscris în triunghiul median, cercul lui Spiecker.

Propozitia 4. Centrul I al cercului înscris în triunghiul ABC este punctul luiNagel al triunghiului median A0B0C0.Demonstratie. Pentru punctul lui Nagel N 0 al triunghiului A0B0C 0 avem:

zN 0 =1

p0[(p0 − a0) zA0 + (p0 − b0) zB0 + (p0 − c0) zC0 ] =

=1

p

·(p− a)

zB + zC2

+ (p− b)zC + zA2

+ (p− c)zA + zB2

¸=

=1

2p(azA + bzB + czC) = zI ,

deci N 0 coincide cu I.Propozitia 5. Punctul lui Nagel N al triunghiului ABC este centrul cercului

înscris în triunghiul complementar XY Z (unde A ∈ Y ZkBC, B ∈ ZXkCA, C ∈∈ XY kAB).Demonstratie. Aplicam propozitia precedenta luând triunghiul complementar

în rolul triunghiului ABC. Se poate da si o demonstratie directa acestei propozitii(de tipul celei din Propozitia 4).

Bibliografie1. D. Brânzei, S. Anita, M. Chirciu - Geometrie, cl. a IX-a, Colectia Mate 2000,Editura ”Paralela 45”, Pitesti, 1998.

20

Page 25: Revista (format .pdf, 1.6 MB)

Aplicatii ale rotatiei în planul complexOana CÂRJA1

Exista o functie bijectiva între multimea numerelor complexe si multimea puncte-lor dintr-un plan. Multiplicarea cu i, adica functia z → iz, z ∈ C, este o rotatie aplanului cu centrul în origine si unghi de 90. Multiplicarea z → az, z ∈ C, unde a esteun numar complex fixat, este o rotatie în jurul originii cu un unghi egal cu argumentullui a, compusa cu o omotetie de raport |a|. Rotatia planului cu 90 în jurul unuipunct de afix b este data de z0 = b+ i (z − b) , z ∈ C. Rotatia planului cu un unghiϕ în jurul unui punct de afix b este data de z0 = b+ (cosϕ+ i sinϕ) (z − b) , z ∈ C.Scopul propus este de a pune în evidenta utilitatea acestei transformari, rotatia

de centru si unghi date, în rezolvarea unor tipuri de probleme de geometrie.

Problema 1. Demonstrati ca, daca se construiesc triunghiuri echilaterale înexterior (sau în interior) pe laturile unui triunghi oarecare, atunci centrele lor suntvârfurile unui triunghi echilateral.

Solutie. Fie ε = cosπ

3+ i sin

π

3; observam ca ε6 = 1 si

ε2 = ε−1. Cu notatiile din figura alaturata si cu conventiaca afixul unui punct P sa fie notat p, avem: segmentul(A0C) se obtine rotind (BC) cu un unghi de 60 în senstrigonometric, adica a0−c = ε (b− c) sau a0 = c+ε (b− c) .Analog, gasim: b0 = a + ε (c− a) si c0 = b + ε (a− b).Ca urmare, pentru centrele triunghiurilor echilaterale, vomavea:

a00 =1

3(a0 + b+ c) , b00 =

1

3(b0 + c+ a) , c00 =

1

3(c0 + a+ b)

sau

a00 =1

3[b+ 2c+ ε (b− c)] , b00 =

1

3[c+ 2a+ ε (c− a)] , c00 =

1

3[a+ 2b+ ε (a− b)] .

Atunci,

b00 − a00 =1

3[2a− b− c+ ε (2c− a− b)] , c00 − a00 =

1

3[a+ b− 2c+ ε (a− 2b+ c)]

si, înmultind prima egalitate cu ε si tinând seama ca ε2 = ε− 1, vom obtine:

ε (b00 − a00) =1

3[ε (2a− b− c) + (ε− 1) (2c− a− b)] =

=1

3[a+ b− 2c+ ε (a− 2b+ c)] = c00 − a00.

Asadar, |b00 − a00| = |c00 − a00|. Se obtine la fel ca |c00 − b00| = |a00 − b00|. Ultimeleegalitati arata ca triunghiul A00B00C00 este echilateral.Problema 2. Determinati locul geometric al centrelor de greutate ale triunghiu-

rilor echilaterale cu vârfurile pe laturile unui patrat.Solutie. Fie MNP un triunghi echilateral înscris în patratul ABCD cu pozitia

din figura. Punctele ce intervin au afixele indicate (sistemul de axe ales are laturapatratului ca unitate de masura pe acestea).

1 Eleva, clasa a X-a, Colegiul National ”C. Negruzzi”, Iasi

21

Page 26: Revista (format .pdf, 1.6 MB)

Segmentul (MN) se obtine rotind segmentul

(MP ) cu un unghi egal cuπ

3, deci

n+ i−mi =³cos

π

3+ i sin

π

3

´(p−mi)⇔

⇔ n+ i−mi =

Ã1

2+

√3

2i

!(p−mi)⇔

n =

1

2p+

√3

2m

1−m =

√3

2p− 1

2m

⇔½

m = 2−√3pn =√3− p

.

Cum 0 ≤ m ≤ 1, obtinem ca 0 ≤ 2 − √3p ≤ 1, adica 1√3≤ p ≤ 2√

3. Analog, din

0 ≤ n ≤ 1 obtinem √3− 1 ≤ p ≤ √3. Ca urmare, √3− 1 ≤ p ≤ 2√3

3.

Punctele M,N,P au afixele¡2−√3p¢ i, √3− p+ i si p, cu p ∈

"√3− 1, 2

√3

3

#.

Centrul de greutate G al triunghiului MNP are afixul g =1

3

£√3 +

¡3−√3p¢ i¤ =

=

√3

3+ αi, unde α = 1 −

√3

3p ∈

"1

3,

√3

3

#. Asadar, G este situat pe un segment

paralel cu Oy de lungime

√3− 13

.

Având în vedere si celelalte trei pozitii posibile ale triunghiului echilateral fata de

patrat, obtinem în final ca locul geometric cautat este un patrat de latura

√3− 13

,

cu acelasi centru ca al patratului dat si cu laturile paralele cu ale acestuia.

Problema 3. Fie ABC un triunghi si D un punct în interiorul sau astfel încât

m(\ADB) = m(\ACB) + 90 si AC ·BD = AD ·BC. Gasiti raportul AB · CDAC ·BD .

Solutie. Alegem un sistem de axe de coordonatecu originea în punctul D si fie A (a), B (b), C (c).Notam ω = cosα + i sinα, unde α = m(\CAD) ∈[0, π), si k =

AC

AD=

BC

BD.

Deoarece printr-o rotatie cu unghiul α si o omotetie

de raportAC

AD= k segmentul (AD) se transforma în

(AC), avem:a− c

AC= ω

a

ADsau a− c = kωa.

Din faptul ca m(\ADB) = m(\ACB) + 90 rezulta ca m(\CAD) +m(\CBD) = 90si obtinem ca mai sus relatia c− b = kiωb.Relatiile obtinute conduc la c = a (1− kω) = b (1 + ikω) si vom obtine (a− b) c =

= k (iωb+ ωa) c = kiωb · a (1− kω) + kωa · b (1 + ikω) = kabω (1 + i). Ca urmare,AB · CD = |a− b| · |c| = k |a| |b| |i+ 1| = k

√2AD · BD =

√2AC · BD si deci

22

Page 27: Revista (format .pdf, 1.6 MB)

AB · CDAC ·BD =

√2.

Problema 4. Pe laturile unui hexagon ce are un centru de simetrie se constru-iesc în exterior triunghiuri echilaterale. Vârfurile acestor triunghiuri, care nu suntvârfuri ale hexagonului dat, formeaza un nou hexagon. Aratati ca mijloacele laturiloracestuia din urma sunt vârfurile unui hexagon regulat.Solutie. Fie A1A2 . . . A6 hexagonul dat (orientat în sens invers acelor de cea-

sornic), fie A1B1A2, A2B2A3, . . . , A6B6A1 triunghiurile echilaterale construite pe la-turile acestuia si C1,C2, . . . , C6 mijloacele segmentelor (B1B2), (B2B3) , . . . , (B6B1).Alegem un sistem de axe cu originea în centrul de simetrie al hexagonului si convenim

ca afixul unui punct X sa fie notat x. Fie ε = cosπ

3+ i sin

π

3=1

2+ i

√3

2si deci

ε6 = 1, ε2 = ε− 1, ε+ ε = 1.Deoarece (A1B1) se obtine rotind (A1A2) cu

π

3în sens negativ, obtinem b1−a1 =

= ε (a2 − a1), adica b1 = (1− ε) a1+εa2 sau b1 = εa1+(1− ε) a2. Analog, se obtine

b2 = εa2 + (1− ε) a3. Vom avea: c1 =1

2(b1 + b2) =

1

2(εa1 + a2 + (1− ε) a3) si

relatii similare pentru c2, . . . , c6.Pe de alta parte, daca rotim C1 în jurul originii cu un unghi de

π

3, obtinem un

punct de afix1

2ε (εa1 + a2 + (1− ε) a3) =

1

2

¡ε2a1 + εa2 +

¡ε− ε2

¢a3¢= 1

2 (εa2 +

+a3 + (ε− 1) a1) = 12 (εa2 + a3 + (1− ε) a4) = c2 (s-a folosit faptul ca a4 = −a1, O

fiind centru de simetrie). Asadar, în urma rotatiei amintite C1 trece în C2. La fel

se arata ca punctul Ci+1 se obtine rotind Ci în jurul originii cuπ

3pentru i = 2, 6

(C7 fiind vârful C1). În concluzie, C1C2 . . . C6 este un hexagon regulat cu centrul înorigine.

Probleme propuse1. Pe laturile [AB], [BC], [CD], [DA] ale patrulaterului ABCD se construiesc în

exterior patratele de centru O1, O2, O3 si respectiv O4. Demonstrati ca O1O3⊥O2O4si [O1O3] ≡ [O2O4].2. Pe laturile unui triunghi ABC se construiesc în exterior triunghiurile echilate-

rale BA0C, CB0A, AC 0B. Demonstrati ca: a) [AA0] ≡ [BB0] ≡ [CC0]; b) centreletriunghiurilor echilaterale sunt vârfurile unui triunghi echilateral având acelasi centrude greutate ca si triunghiul ABC.

3. Fie ABCD un patrulater convex. Demonstrati ca, daca exista un punct Pîn interiorul sau astfel încât m(\PAB) = m\(PBC) = m(\PCD) = m(\PDA) = 45,atunci ABCD este un patrat. Generalizare.

Bibliografie1. M. Dinca, M. Chirita - Numere complexe în matematica din liceu, Editura AllEducational, 1996.

2. T. Andreescu, R. Gelca - Mathematical Olympiad Challenges, Birkhäuser, 1996.3. A. Engel - Problem-Solving Strategies, Springer, 1999.

23

Page 28: Revista (format .pdf, 1.6 MB)

Functii trigonometrice inverse - aplicatiiGheorghe IUREA1

În general, elevii au serioase dificultati în folosirea functiilor trigonometrice in-verse. De asemeni, în unele culegeri si chiar manuale se abuzeaza de formule cu astfelde functii sau sunt date rezolvari gresite.Vom prezenta, în aceasta nota, modalitati de abordare a unor probleme cu functii

trigonometrice inverse care sa apeleze la cât mai putine cunostinte din partea elevilor.Începem prin definirea acestor functii:

a) arcsin : [−1, 1] →h−π2,π

2

i, definita prin arcsinx = α ⇔ x ∈ [−1, 1], α ∈

∈h−π2,π

2

i, sinα = x;

b) arccos : [−1, 1] → [0, π], definita prin arccosx = α ⇔ x ∈ [−1, 1], α ∈ [0, π],cosα = x;

c) arctg : R →³−π2,π

2

´, definita prin arctg x = α ⇔ x ∈ R, α ∈

³−π2,π

2

´,

tgα = x;d) arcctg : R→ (0, π), definita prin arcctg x = α⇔ x ∈ R, α ∈ (0, π), ctgα = x.

Aplicatii.1. Au loc relatiile: a) arcsin (−x) = − arcsinx, x ∈ [−1, 1];b) arccos (−x) = π − arccosx, x ∈ [−1, 1];c) arctg (−x) = − arctg x, x ∈ R;d) arcctg (−x) = π − arcctg x, x ∈ R.Solutie. Vom dovedi numai egalitatile a) si b).

a) Fie arcsinx = α, deci x ∈ [−1, 1], α ∈h−π2,π

2

i, sinα = x. Cum −x ∈ [−1, 1],

−α ∈h−π2,π

2

isi sin (−α) = −x, rezulta ca arcsin (−x) = −α = − arcsinx.

b) Fie arccosx = α, deci x ∈ [−1, 1], α ∈ [0, π] si cosα = x. Cum −x ∈ [−1, 1],π − α ∈ [0, π] si cos (π − α) = −x, urmeaza ca arccos (−x) = π − α = π − arccosx.

Observatie. Formulele stabilite sunt utile în determinarea valorilor functiilor

trigonometrice inverse pentru valori negative. De exemplu, avem arcsin

µ−12

¶=

= − arcsin 12= −π

6; arccos

µ−12

¶= π − arccos 1

2= π − π

3=2π

3; arctg (−1) =

= − arctg 1 = −π4.

2. Determinati un interval de lungimeπ

12în care este situat arctg

√10

3.

Solutie. Fie arctg√10

3= α, deci α ∈

³−π2,π

2

´, tgα =

√10

3. Încadram

√10

3între doua numere care reprezinta tangentele unor unghiuri cunoscute. Cum 1 <

1 Profesor, Liceul ”D.Cantemir”, Iasi

24

Page 29: Revista (format .pdf, 1.6 MB)

<

√10

3<√3, deducem tg

π

4< tgα < tg

π

3. Folosind monotonia functiei tangenta,

rezulta α = arctg

√10

3∈³π4,π

3

´, iar acest interval are lungimea egala cu

π

12.

3. Calculati E = sinµ2 arctg

1

2+ arccos

4

5

¶.

Solutie. Fie arctg1

2= α, arccos

4

5= β. Atunci α, β ∈

³0,π

2

´, tgα =

1

2, cosβ =

=4

5si E = sin (2α+ β) = sin 2α cosβ + sinβ cos 2α. Cum sin 2α =

2 tgα

1 + tg2 α=4

5,

sinβ =p1− cos2 β = 3

5si cos 2α =

1− tg2 α1 + tg2 α

=3

5, gasim E = 1.

4. Aratati ca sinµ4 arctg

1

3

¶= cos

µ2 arctg

1

7

¶.

Solutie. Fie arctg1

3= α, arctg

1

7= β; avem : α, β ∈

³0,π

2

´si tgα =

1

3, tg b =

=1

7. Relatia de demonstrat devine sin 4α = cos 2β. Avem sin 4α = 2 sin 2α cos 2α =

= 2 · 2 tgα

1 + tg2 α· 1− tg

2 α

1 + tg2 α=24

25si cos 2β =

1− tg2 β1 + tg2 β

=24

25, deci sin 4α = cos 2β.

5. Determinati arcsin (sin 3).Solutie. Fie arcsin (sin 3) = α, deci α ∈

h−π2,π

2

i, sinα = sin 3. Cum 3 /∈

/∈h−π2,π

2

i, scriem relatia sub forma sinα = sin (π − 3), cu α, π − α ∈

h−π2,π

2

i.

Rezulta α = π − 3, deci arcsin (sin 3) = π − 3.

Observatii. 1. Din definitiile functiilor trigonometrice deducem: arcsin (sinα) == α, α ∈

h−π2,π

2

isi sin (arcsinx) = x, x ∈ [−1, 1].

2. O rezolvare de tipul : arcsin (sin 3) = 3 este evident gresita întrucât 3 /∈/∈h−π2,π

2

i.

6. Determinati arcsinµcos

7

2

¶.

Solutie. Notând arcsinµcos

7

2

¶= α, avem α ∈

h−π2,π

2

i, cos

7

2= sinα, de unde

cos7

2= cos

³π2− α

´, deci

π

2−α = ±7

2+2kπ sau α =

π

2± 72−2kπ, k ∈ Z. Cum α ∈

∈h−π2,π

2

i, gasim k = 1 si α =

π

2+7

2−2π = 7− 3π

2, deci arcsin

µcos

7

2

¶=7− 3π2

.

7. Aratati ca arcsin1√5+ arccos

3√10=

π

4.

Solutie. Fie arcsin1√5= α, arccos

3√10= β, deci α, β ∈

³0,π

2

´si sinα =

1√5,

25

Page 30: Revista (format .pdf, 1.6 MB)

cosβ =3√10. Atunci cos (α+ β) = cosα cosβ − sinα sinβ =

√2

2. Cum α + β ∈

∈ (0, π), urmeaza ca α+ β =π

4, adica ceea ce trebuia aratat.

Observatii. 1. Daca vom calcula sin (α+ β) gasim sin (α+ β) =

√2

2, α + β ∈

∈ (0, π), deci α+ β =π

4sau α+ β =

4. În aceasta situatie calculam si cos (α+ β)

pentru a determina în ce cadran este α + β sau aproximând mai bine α, β, gasim

α, β ∈³0,π

4

´si atunci α+ β ∈

³0,π

2

´; acum putem afirma ca α+ β =

π

4.

2. Evident, o solutie de tipul ”demonstram ca sinµarcsin

1√5+ arccos

3√10

¶=

= sinπ

4” este incompleta.

8. Aratati ca arcsin4

5+ arcsin

12

13+ arcsin

56

65= π.

Solutie. Fie α = arcsin4

5, β = arcsin

12

13; α, β ∈

h−π2,π

2

i, sinα =

4

5, sinβ =

12

13.

Gasim usor ca α ∈³π4,π

2

´, β ∈

³π4,π

2

´si sin (α+ β) =

56

65,π

2< α + β < π, deci

sin (π − (α+ β)) =56

65, π − (α+ β) ∈

³0,π

2

´, prin urmare π − (α+ β) = arcsin

56

65,

deci arcsin4

5+ arcsin

12

13+ arcsin

56

65= π.

9. Aratati ca 2 arctgr1− x

1 + x= arccosx, x ∈ (−1, 1].

Solutie. Fie α = arctgr1− x

1 + x, β = arccosx, deci tgα =

r1− x

1 + x, α ∈

³0,π

2

´,

cosβ = x, β ∈ (0, π). Gasim cos 2α = cosβ cu 2α, β ∈ (0, π), ceea ce impune ca2α = β, ceea ce trebuia aratat.

10. Calculati 2 arctg1

5+ arctg

1

4.

Solutie. Fie α = arctg1

5, β = arctg

1

4, deci tgα =

1

5, tg β =

1

4, α, β ∈

∈³−π2,π

2

´. Aproximând α, β, gasim α, β ∈

³0,π

6

´. Calculând tg (2α+ β), de-

ducem ca tg (2α+ β) =32

43; 0 < 2α+ β <

π

2, ceea ce conduce la 2α+ β = arctg

32

43,

care este valoarea cautata.

26

Page 31: Revista (format .pdf, 1.6 MB)

O metoda de calcul al distantei dintre doua drepteneparalele în spatiu

Romanta GHITA si Ioan GHITA1

În multe probleme de geometrie în spatiu se cere printre altele, aflarea distanteidintre doua drepte necoplanare. Aceasta problema este, în general, dificila daca seîncearca sa se puna în evidenta perpendiculara comuna a celor doua drepte.În aceasta nota vom da un mod de rezolvare a acestor chestiuni. În acest scop

vom utiliza urmatoarele propozitii [1]:1. Fiind date doua drepte necoplanare, exista si este unic un plan care contine o

dreapta si este paralel cu cealalta dreapta.

2. Distanta dintre doua drepte necoplanare este egala cu distanta de la o dreaptala planul paralel cu acea dreapta si care contine cealalta dreapta.

Prin urmare pentru a gasi distanta dintre doua drepte necoplanare, ducem printr-o dreapta planul paralel la cealalta dreapta si calculam distanta de la aceasta dreaptala plan.

Problema 1. Fie ABCD un tetraedru regulat, de muchie a. Sa se determinedistanta dintre doua mediane necoplanare a doua dintre fetele sale. (Olimpiada dematematica , Blaj, 1995).Solutie. Fie DE,AF medianele a doua fete (fig.1). Ducem FGkDE si atunci

d (DE,AF ) = d (DE, (AGF )). Consideram piramida AGEF . Daca d este distanta

dintre DE si AF avem VAGEF =1

3SAGF · d si cum VAGEF =

a3√2

96iar SAGF =

=a2√35

32, gasim d =

a√70

35.

Problema 2. Fie V ABCD o piramida patrulatera regulata, cu toate muchiileegale cu a. Sa se afle distanta dintre o mediana a unei fete laterale si o diagonala abazei. (Olimpiada de matematica, Blaj, 1996).Solutie. Sa presupunem ca mediana este dusa din V , fie acesta V E (fig.2); deter-

minam distanta d dintreAC si V E. FieEFkAC, deci d (V E,AC) = d (AC, (V EF )) =

= d (O, (V EF )). Avem VOVEF =1

3SV EF ·d si cum VOVEF =

a3√2

48, SV EF =

a2√5

8,

gasim d =a√10

10.

Presupunem acum ca mediana nu este dusa dusa din V , fie aceasta DE (fig.3).Daca EFkAC, F ∈ (V A), atunci d (AC,DE) = d (AC, (DEF )) = d (O, (DEF )).

Din VODEF =1

3SDEF · d, VODEF =

1

3OD · SEOF =

a3√2

48si SDEF =

a2√5

8,

obtinem d =a√10

10.

Problema 3. Sa se afle distanta dintre doua diagonale a doua fete laterale ce nuau un punct comun, ale unei prisme triunghiulare regulate drepte cu muchiile lateraleegale cu muchiile bazei.

1 Profesori, Colegiul National ”I.M.Clain”, Blaj

27

Page 32: Revista (format .pdf, 1.6 MB)

Solutie. Fie AB = a. Sa determinam distanta dintre C0B si A0C. Fie E ∈∈ (ABC) astfel încât ACBE sa fie paralelogram (fig.4); deducem ca A0EkC0B siatunci d (C0B,A0C) = d (C 0B, (A0CE)) = d (B, (A0CE)). Avem succesiv VBA0CE =

=1

3SA0CE · d, VBA0CE =

a3√3

12, SA0CE =

a2√15

4si atunci d =

a√5

5.

fig.1 fig.2

fig.3 fig.4

Probleme propuse.1. Sa se determine distanta dintre diagonalele a doua fete laterale alaturate ale

unui cub de muchie a, diagonalele neavând punct comun.2. Fie ABCDA0B0C 0D0 un cub de muchie a si M,N,P,Q mijloacele muchiilor

(AA0), (AB), (BB0) respectiv (BC). Sa se afle a) d (MN,PQ); b) d (MN,B0C).3. Fie ABCD un tetraedru regulat de muchie a siM,N,P,Q mijloacele muchiilor

(BC), (AC), (CD) si respectiv (BD). Sa se afle d (MN,PQ).4. Fie ABCDA0B0C0D0 un paraleliped dreptunghic de muchii AB = a, BC = b

si AA0 = c. Sa se afle d (A0B,B0C).5. Fie ABCD si ABEF patrate de latura a, cu unghiul planelor lor de 60.

Calculati d (AC,BF ).

Bibliografie.1. I. Cuculescu, C. Ottescu, O. Popescu - Manual de geometrie pentru clasa aVIII-a, Editura Didactica si Pedagogica, Bucuresti, 1996.

2. I. Dancila - Distante între elementele fundamentale ale spatiului euclidian, G.M.-9/1995, p.565.

3. V. Carbunaru, C. Carbunaru - Culegere de probleme de matematica, claseleV-VIII, Editura Conviocarb, Bucuresti, 1996.

28

Page 33: Revista (format .pdf, 1.6 MB)

Observatii metodice privind punctele de inflexiuneGabriel MÎRSANU 1

În manualul de cl. XI-a [2], punctul de inflexiune este introdus prin

Definitia 1. Fie f : I → R o functie continua definita pe intervalul I ⊆ R. Unpunct x0 ∈ I distinct de capetele lui I se numeste punct de inflexiune pentru f dacaexista puncte α < x0 < β în I astfel încât f sa fie convexa pe (α, x0] si concava pe[x0, β) sau invers.

Prin câteva contraexemple vom arata ca aceasta definitie este incompleta si con-duce la erori.

Contraexemplul 1. Fie functia f : R → R, f (x) =½

x2, x ≤ 0arctg x, x > 0

. Evi-

dent, f este continua pe R si avem:

f 0 (x) =½2x, x < 01/¡1 + x2

¢, x > 0

si f 00 (x) =½2, x < 0

−2x/ ¡1 + x2¢2, x > 0

.

Conform Corolarului Teoremei Lagrange obtinem ime-diat ca f 0s (0) = lim

x%0f 0 (x) = 0 si f 0d (0) = lim

x&0f 0 (x) = 1.

Ca urmare, functia f nu este derivabila în x0 = 0, este con-vexa la stânga lui x0 si concava la dreapta acestuia. Con-form definitiei de mai sus, x0 = 0 ar fi punct de inflexiune,ceea ce este fals, caci el este evident un punct unghiular.

Contraexemplul 2. Fie functia f : R → R, f (x) =½

x2, x ≤ 0√x, x > 0

. Aceasta

functie este continua pe R si avem:

f 0 (x) =

2x, x < 01

2√x, x > 0

, f 00 (x) =

2, x < 0

− 1

4x√x, x > 0

,

f 0s (0) = limx%0

f (x) = 0, f 0d (0) = limx&0

f 0 (x) = +∞. De-oarece f este continua în x0, convexa la stânga si concavala dreapta acestui punct, ar urma ca x0 = 0 este punct deinflexiune, ceea ce este fals, x0 = 0 fiind un punct unghiular(unghiul dintre cel doua semitangente în origine fiind de

π

2.

Aceste contraexemple ne-au determinat sa clarificam notiunea de punct de inflexi-une. Definitia punctului de inflexiune clara si corecta apare în [3, p. 339] si în [1,p. 17].

Definitia 2. Fie I ⊂ R, I interval, f : I → R si x0 ∈ int I. Se spune ca x0 estepunct de inflexiune al functiei f daca sunt îndeplinite urmatoarele conditii:(i) f este continua în x0;(ii) f are derivata (finita sau infinita) în x0;(iii) f este convexa de o parte a lui x0 si concava de cealalta parte.

1 Profesor, Liceul de Informatica ”Gr.Moisil”, Iasi

29

Page 34: Revista (format .pdf, 1.6 MB)

Daca x0 este punct de inflexiune al functiei f , punctul I0 (x0, f (x0)) de pe graficulacesteia se numeste punct de inflexiune al graficului.Geometric, a spune ca I0 este punct de inflexiune al graficului înseamna ca graficul

admite tangenta în punctul I0 (care poate fi si verticala când exista f 0 (x0) = +∞sau −∞) si ca de o parte a lui I0 graficul este o curba convexa, iar de cealalta partea lui I0 acesta este o curba concava sau ca de o parte a lui I0 tangenta se afla subgrafic, iar de cealalta parte tangenta se afla deasupra graficului (în cazul tangenteiverticale, aceasta se afla la stânga sau la dreapta graficului). Evident, tangenta dusaîntr-un punct de inflexiune al graficului traverseaza graficul. În acest sens, toatepunctele unei drepte sunt puncte de inflexiune.

Exemplu. Sa se determine punctele de inflexiune ale functiei f : R→ R,f (x) = 3

√x− 1 + 3

√x+ 1 [2, p.180].

Evident, functia f este continua peR si avem f 0(x) =1

3

1

3

q(x− 1)2

+1

3

q(x+ 1)

2

,x ∈ R\ −1, 1 si f 00(x) = −2

9

1

3

q(x− 1)5

+1

3

q(x+ 1)

5

, x ∈ R\ −1, 1, f 0 (1) == lim

x→1f 0 (x) = +∞, f 0 (−1) = lim

x→−1f 0 (x) = +∞ si f 00 (x) = 0 ⇔ x = 0. Formam

tabelul:

x −∞ −1 0 1 +∞f 0 (x) + +∞ + +∞ +

f (x) −∞ − 3√2 0 3

√2 +∞

f 00 (x) + − 0 + −i i i

În concluzie, functia are trei puncte de inflexiune: −1, 0, 1. Indicatia din manual:”se pune mai întâi conditia necesara f 00 (x) = 0 si apoi se studiaza semnul în jurulacestor puncte” este gresita - conduce la un singur punct de inflexiune: x0 = 0.(Aceasta greseala a fost constatata si în tezele candidatilor la admitere la Facultateade Matematica din Iasi, cu ani în urma.)

Bibliografie.1. M. Ganga - Elemente de analiza matematica pentru cl. a XI-a, partea a II-a, Ed.Mathpress, 1997.

2. Gh. Gussi, O. Stanasila, T. Stoica - Elemente de analiza matematica, cl. XI-a,E.D.P., Bucuresti, 1984.

3. M. Nicolescu, N. Dinculeanu, S. Marcus -Manual de analiza matematica, vol. I,ed. a 4-a, E.D.P., Bucuresti, 1971.

30

Page 35: Revista (format .pdf, 1.6 MB)

Un procedeu de rezolvare a unei ecuatii diofanticede gradul al doileaMihai GÂRTAN 1

Este vorba, în cele ce urmeaza, de ecuatia diofantica

ax2 + bxy + cy = d, a, b, c, d ∈ Z, b 6= 0 (1)

Pentru rezolvarea acesteia în Z × Z, o înmultim cu b2 si apoi scadem ac2 din ambiimembri ai rezultatutului obtinut:

a¡b2x2 − c2

¢+b2y (bx+ c) = b2d−ac2 ⇔ (bx+ c)

¡abx+ b2y − ac

¢= b2d−ac2. (2)

Solutiile ecuatiei (2), deci si ale ecuatiei date, sunt solutiile întregi ale sistemelor dedoua ecuatii urmatoare:

bx+ c = p, abx+ b2y − ac = q, (3)

unde p, q sunt divizori oarecare ai lui b2d− ac2 supusi restrictiei pq = b2d− ac2.

Exercitiul 1. Rezolvati ecuatia diofantica x2 − 5xy + y = 3.Solutie. Urmând procedeul de mai sus, înmultim ecuatia cu 25 si apoi scadem 1

si obtinem:¡25x2 − 1¢− 125xy + 25y = 74 sau (5x− 1) (5x− 25y + 1) = 74.

Avem de rezolvat în Z× Z opt sisteme:½5x− 1 = ±15x− 25y + 1 = ±74,

½5x− 1 = ±25x− 25y + 1 = ±37,

½5x− 1 = ±375x− 25y + 1 = ±2,

½5x− 1 = ±745x− 25y + 1 = ±1

(în fiecare sistem + corespunde cu + si − cu −). Se constata ca sistemul formatdin ecuatiile 5x− 1 = −1 si 5x− 25y + 1 = −74 admite solutia (0, 3) si ca sistemelecelelalte nu au solutii în Z× Z. În concluzie, ecuatia data are o solutie: (0, 3).Exercitiul 2. Determinati multimea A =

nx ∈ Z; 3x2−12x+5 ∈ Z

o.

Solutie. Notând 3x2−12x+5 = y, suntem condusi la ecuatia 3x2 − 2xy − 5y = 1.

Observam ca A este multimea acelor x ∈ Z pentru care perechea (x, y) este solutiea acestei ecuatii diofantice. Înmultind ecuatia cu 4 si scazând apoi 75, obtinem¡12x2 − 75¢ − 8xy − 20y = −71, de unde (2x+ 5) (6x− 4y − 15) = −71. Sistemeleasociate sunt:½2x+ 5 = 16x− 4y − 15 = −71,

½2x+ 5 = −16x− 4y − 15 = 71,

½2x+ 5 = 716x− 4y − 15 = −1,

½2x+ 5 = −716x− 4y − 15 = 1 .

Ele au solutiile: (−2,−11), (−3,−26), (33, 46) si (−38,−61). Ca urmare,A = −38,−3,−2, 33.Exercitii propuse.1) Rezolvati urmatoarele ecuatii diofantice:

a) x2 − xy + 3y = 2, b) 2x+ 3xy − 4y2 = 5, c) 2x2 − 3xy − y = −22.2) Determinati multimile:

a) A =nx∈Z; x2+12x+3 ∈Z

o, b) B =

nx∈Z; 3x2−12x+1 ∈Z

o, c) C =

nx∈Z; 3x2+12x+1 ∈Z

o.

1 Profesor, Colegiul National ”C.Negruzzi”, Iasi

31

Page 36: Revista (format .pdf, 1.6 MB)

Numarul divizorilor si suma divizorilorunui numar naturalMihai CRACIUN 1

Am închis usile ca sa nu intregreseala. Atunci adevarul m-aîntrebat: pe unde voi intra eu?

Rabindranath Tagore

Fie n un numar natural mai mare ca 1. Vom nota cu d (n) numarul divizorilor luin, iar cu S (n) suma acestor divizori. Vom prezenta doua formule simple, accesibileelevilor din clasele gimnaziale si care au aplicatii interesante.

Amintim mai întâi teorema fundamentala a aritmeticii:Orice numar natural n > 1 se reprezinta în mod unic, abstractie facând de ordinea

factorilor, ca un produs de numere prime.

În consecinta, orice numar natural n > 1 se descompune în mod unic sub forman = pa11 pa22 · . . . · pakk , unde p1, p2, . . . , pk sunt numere prime, p1 < p2 < . . . < pk, cua1, a2, . . . , ak ∈ N∗ (descompunerea canonica a lui n).Vom utiliza urmatoarea regula de baza a analizei combinatorii:Daca obiectele x1, x2, . . . , xk pot fi alese, în ordinea scrisa, în m1,m2, . . . ,mk

moduri, atunci k-uplul (x1, x2, . . . , xk) poate fi ales în m1m2 · . . . ·mk moduri (regulaprodusului).

Teorema. Daca n > 1 are descompunerea n = pa11 pa22 · . . . · pakk , atuncid (n) = (a1 + 1) (a2 + 1) · . . . · (ak + 1) , (1)

S (n) =pa1+11 − 1p1 − 1 · p

a2+12 − 1p2 − 1 · . . . · p

ak+1k − 1pk − 1 . (2)

Demonstratie. Un divizor al lui n are forma pb11 pb22 · . . . · pbkk , unde bi ∈

∈ 0, 1, . . . , ai, i = 1, k. Ca urmare, b1 poate fi ales în a1 + 1 moduri, b2 îna2 + 1 moduri, . . . , bk în ak + 1 moduri. Deci, (b1, b2, . . . , bk) poate fi ales în(a1 + 1) (a2 + 1) · . . . · (ak + 1) moduri si relatia (1) este dovedita.Suma divizorilor, în mod evident, coincide cu produsul¡1 + p1 + p21 + . . .+ pa11

¢ ¡1 + p2 + p22 + . . .+ pa22

¢ · . . . · ¡1 + pk + p2k + . . .+ pakk¢.

Cum acesta este egal cu

pa1+11 − 1p1 − 1 · p

a2+12 − 1p2 − 1 · . . . · p

ak+1k − 1pk − 1 ,

rezulta ca pentru S (n) are loc formula (2).

Exemplu. Pentru 72 = 23 · 32 avem d (72) = (3 + 1) (2 + 1) = 12 si

S (72) =24 − 12− 1 ·

33 − 13− 1 = 195.

1 Profesor, Grupul Scolar ”Unirea”, Pascani

32

Page 37: Revista (format .pdf, 1.6 MB)

Problema 1. Aflati cel mai mic numar natural care are exact 105 divizori.Solutie. Deoarece 105 = 3 · 5 · 7, rezulta ca numarul cautat are descompunerea

canonica de forma pa11 pa22 pa33 , cu (a1 + 1) (a2 + 1) (a3 + 1) = 3 · 5 · 7. Cel mai micnumar se obtine pentru p1 = 2, p2 = 3, p3 = 5 si a1 + 1 = 7, a2 + 1 = 5, a3 + 1 = 3.Acest numar este 26 · 34 · 52 = 129600.Problema 2. Aratati ca numarul divizorilor lui na+1 care se divid cu na este

egal cu d (n).Solutie. Este suficient sa observam ca orice divizor al lui na+1 ce se divide cu

na este de forma m · na, m fiind un divizor al lui n.

Problema 3. Un numar natural este de forma n = pa ·qb, unde p, q sunt numereprime. Stiind ca puterea a treia are 133 divizori, sa se gaseasca ce putere a lui n are645 divizori.

Solutie. Avem d¡n3¢= (3a+ 1) (3b+ 1) = 133 = 7 · 19 si, deci,

½3a+ 1 = 73b+ 1 = 19

sau½3a+ 1 = 193b+ 1 = 7

. Obtinem a = 2, b = 6 sau a = 6, b = 2. Cum d (nx) = (ax+ 1)×× (bx+ 1) = abx2+(a+ b)x+1, vom avea 12x2+8x+1 = 645 si, ca urmare x = 7.

Problema 4. Determinati un numar par stiind ca numarul divizorilor sai este15 si suma lor este 5467.Solutie. Numarul n cautat fiind par, are un factor prim egal cu 2. Deoarece

d (n) = 15 = 3 · 5, rezulta ca n are cel mult doi factori primi: 2 si p si acestia suntla puterile 2 si 4. Deci n = 24 · p2 sau n = 22 · p4 si cum S (n) = 5467, avemcorespunzator

25 − 12− 1 ·

p3 − 1p− 1 = 5467 sau

23 − 12− 1 ·

p5 − 1p− 1 = 5467.

Numai a doua ecuatie are solutia p = 5 ∈ N. Deci, n = 22 · 54 = 2500.Problema 5. Aratati ca numarul divizorilor unei puteri naturale a unui numar

dat este prim cu exponentul acelei puteri.Solutie. Daca n = pa11 pa22 · . . . · pakk , atunci nm = pma1

1 pma22 · . . . · pmak

k si d (n) == (ma1 + 1) (ma2 + 1) · . . . · (mak + 1) = mb+1, b ∈ N. Rezulta ca (d (nm) ,m) = 1.

Probleme propuse.1. Gasiti cel mai mic numar natural care are 25 de divizori.2. Aflati numarul n de forma n = 3x ·5y ·17z stiind ca 17n are 56 divizori naturali

mai mult ca n, iar 125n are 189 divizori mai mult ca n.

3. Determinati un numar de forma n = px · qy, unde p, q sunt numere prime six, y ∈ N, stiind ca d (n) = 20 si S (n) = 12400.4. Gasiti un numar n = pa ·qb ·ra+b+1, cu p, q, r numere prime, stiind ca d (n) = 30

si S (n) = 6045.

33

Page 38: Revista (format .pdf, 1.6 MB)

Aplicatii ale criteriului de integrabilitateal lui Lebesgue

Constantin CHIRILA1

Scopul acestei note este de a prezenta teorema lui Lebesgue de caracterizare afunctiilor integrabile Riemann si de a o aplica la rezolvarea simpla a unor problemeîntâlnite în manualul de liceu [4], în care se cere studiul integrabilitatii pentru functiicare nu sunt neaparat continue sau monotone.

Definitia 1. O multime A se numeste numarabila daca exista o aplicatief : N→ A. O multime finita sau numarabila se numeste cel mult numarabila.

Exemplu. Multimile N, Z, Q sunt multimi numarabile.Definitia 2. O multime E ⊂ R, se numeste multime neglijabila (sau de masura

Lebesgue nula) daca ∀ε > 0, exista o familie cel mult numarabila de intervale deschise(ai, bi)i astfel încât E ⊂

Si(ai, bi) si

Pi(bi − ai) < ε.

Indicele i variaza de la 1 la p daca familia de intervale este finita. Daca familia de

intervale este numarabila, i ∈ N si prinPi(bi−ai) întelegem suma seriei

∞Pi=1(bi−ai).

Exemple. a) Orice multime cel mult numarabila este neglijabila. Într-adevar,daca E = (yn)n∈N ⊂ R, atunci pentru ε > 0 sa consideram sirurile an = yn− ε

2n+1,

n ∈ N si bn = yn +ε

2n+1, n ∈ N. Avem E ⊂

∞Sn=1

(an, bn) si∞Pn=1

(bn − an) ≤ ε.

Prin urmare multimea E este neglijabila. Mai mult, se poate demonstra ca o sub-multime a unei multimi neglijabile este de asemenea neglijabila, iar o reuniune celmult numarabila de multimi neglijabile este neglijabila.b) Orice multime finita este neglijabila. Multimile N, Z, Q fiind numarabile, sunt

neglijabile, daca tinem seama de observatia anterioara.c) Exista multimi neglijabile care nu sunt numarabile. Un exemplu în acest sens

îl reprezinta multimea lui Cantor [1], [2].

Observatie. O proprietate punctuala definita pentru punctele unei multimiE ⊂ R spunem ca are loc aproape peste tot (a.p.t.) pe E daca multimea punctelordin E în care aceasta nu are loc este neglijabila. De exemplu, vom spune ca o functieeste continua a.p.t. daca multimea punctelor sale de discontinuitate este neglijabila.În cele ce urmeaza enuntam criteriul lui Lebesgue de integrabilitate Riemann; o

demonstratie a sa accesibila elevilor poate fi gasita în [1]:

Teorema (Lebesgue). O functie f : [a, b] → R marginita este integrabilaRiemann daca si numai daca este continua a.p.t. pe intervalul [a, b].

Ca o consecinta imediata a acestei teoreme se poate obtine ca functiile continue sicele monotone pe un compact sunt integrabile. Functiile continue pe un compact suntmarginite (Weierstrass) si au multimea discontinuitatilor vida. Functiile monotonepe [a, b] au imaginea inclusa în [f (a) , f (b)] sau [f (b) , f (a)] si au, dupa cum se stie,un numar cel mult numarabil de puncte de discontinuitate. Integrabilitatea acestor

1 Profesor, Liceul Teoretic ”G. Ibraileanu”, Iasi

34

Page 39: Revista (format .pdf, 1.6 MB)

clase de functii este demonstrata în manualul [4] cu ajutorul criteriului lui Darboux,care se dovedeste a fi tehnic si mai putin elegant decât teorema lui Lebesgue.În calculul integralelor unor functii, urmatoarea proprietate se dovedeste utila:

Propozitie. Daca functiile f, g : [a, b] → R sunt marginite pe [a, b], continuea.p.t. pe [a, b] si f(x) = g(x), ∀x ∈ [a, b] \A, unde A este neglijabila, atunci functiilef , g sunt integrabile pe [a, b] si are loc egalitatea

R baf (x) dx =

R bag (x) dx.

Prin problemele urmatoare vom releva eficienta si eleganta criteriului de integra-bilitate Lebesgue.

Problema 1. Se considera functia f : [0, 1]→ R definita prin

f(x) =

1, x = 1/21/2, x = 1/22

. . . . . .1/n, x = 1/2n

. . . . . .x, x 6= 1/2n, ∀n ∈ N∗

.

Sa se arate ca functia f este integrabila pe [0, 1] si sa se calculeze integrala sa.Solutie. Observam ca Df = 1/2n, n ∈ N∗ este multimea punctelor de discon-

tinuitate ale lui f . Atunci multimea Df este numarabila si prin urmare are masuraLebesgue nula. Functia f este continua a.p.t. si marginita, deci este integrabila.Pentru calculul integralei vom utiliza proprietatea mentionata. Consideram functia

g : [0, 1]→ R, g(x) = x; vom aveaZ 1

0

f(t)dt =

Z 1

0

g(t)dt =

Z 1

0

tdt =1

2.

Problema 2. Sa se arate ca functia f : [0, 1]→ R, definita prin

f(x) =

ax, daca x 6= 1

n, n ∈ N∗

xa, daca x =1

n, n ∈ N∗

,

unde a ∈ (0, 1) ∪ (1,+∞), este integrabila si sa se calculeze integrala sa.Solutie. Functia f este marginita si continua a.p.t. deoarece multimea punctelor

de discontinuitate Df =

½1,1

2,1

3, ...,

1

n, ...

¾∪0 este neglijabila. Deci functia f este

integrabila pe [0, 1]. Functia f coincide cu g (x) = ax a.p.t., adicaZ 1

0

f(x)dx =

Z 1

0

axdx =a− 1ln a

.

Problema 3. Sa se arate ca functia f : [0, 2√3] → R, f(x) = x − [x] este

integrabila si sa se calculeze integrala sa.Solutie. Observam ca f (x) = x−k, ∀x ∈ [k, k + 1), k = 0, 1, 2, iar f (x) = x−3,

∀x ∈ £3, 2√3¢. Atunci multimea discontinuitatilor lui f esteDf = 1, 2, 3, neglijabi-la. Cum f este si marginita pe intervalul considerat, rezulta ca este integrabila siZ 2

√3

0

f(x)dx =

Z 1

0

xdx+

Z 2

1

(x− 1)dx+Z 3

2

(x− 2)dx+Z 2√3

3

(x− 3)dx = 12− 6√3.

35

Page 40: Revista (format .pdf, 1.6 MB)

Problema 4. Fie f : [0, 1] → R o functie integrabila. Sa se arate ca functiag : [0, 1]→ R, g(x) = f(x2) este integrabila.Solutie. Conform criteriului lui Lebesgue , functia f fiind integrabila, este

marginita si multimea punctelor sale de discontinuitate este neglijabila.Prin urmarefunctia g este marginita. Vom arata ca functia g are multimea punctelor de discon-tinuitate Dg neglijabila. Daca x0 este punct de continuitate pentru functia f, atunci√x0 este punct de continuitate pentru g si reciproc (din caracterizarea cu siruri a

continuitatii unei functii într-un punct). Prin urmare, Dg = √x/x ∈ Df .Deoarece

masura Lebesgue nu se modifica prin adaugarea unui punct, putem presupune ca Dg

este multimea discontinuitatilor din intervalul (0, 1] (daca se adauga 0 nu se modificamasura).

Notam cu Dkg multimea discontinuitatilor functiei g pe intervalul

·1

k, 1

¸, k ∈ N∗.

Avem caS

k∈N∗Dkg = Dg si este suficient sa demonstram ca pentru fiecare k ∈ N∗

multimeaDkg este neglijabila. MultimeaD

kf = Df∩

·1

k2, 1

¸este o multime neglijabila

pentru ca orice submultime a lui Df este neglijabila. Înseamna ca pentru ∀ε > 0,

exista un sir de intervale deschise¡Ikn¢n∈N∗ , I

kn =

¡akn, b

kn

¢ ⊂ ·1

k2, 1

¸astfel încât

Dkf ⊂

Sn∈N∗

Ikn si∞Pn=1

¡bkn − akn

¢<2ε

k.

Daca notam cu Jkn =³p

akn,pbkn

´, ∀n ∈ N∗, rezulta ca Dk

g ⊂S

n∈N∗Jkn si cum

akn ≥1

k2si bkn ≥

1

k2avem

X∞n=1

³pbkn −

pakn

´≤ k

2

X∞n=1

¡bkn − akn

¢<

k

2

k= ε.

Acest fapt demostreaza caDkg este neglijabila . Prin urmare functia g este integrabila.

În încheiere, propunem spre rezolvare urmatoarele probleme:

1. Sa se studieze integrabilitatea functiilor f, g, h : [0, 1]→ R, iar în caz afirmativsa se calculeze integrala lor:

f(x) =

1

2n, x =

1

n, n ∈ N∗

0, x 6= 1

n, n ∈ N∗

; g(x) =

1

x, x =

1

n, n ∈ N∗

x, x 6= 1

n, n ∈ N∗

; h(x) =

(cos

1

x, x 6= 0

α, x = 0, α ∈ R.

2. Sa se arate ca daca f : [0, 1]→R este integrabila, atunci si functia g : [0, 1]→R,g(x) = f(xp), p ∈ Q+ este integrabila.

Bibliografie.1. St. Frunza - Analiza matematica, partea a II-a, vol. I, Ed.Univ. ”Al. I. Cuza”, Iasi,1987.

2. A. Precupanu - Analiza matematica - functii reale, E.D.P.Bucuresti, 1976.3. C. Popa, M. Megan, V. Hiris - Introducere în analiza matematica prin exercitiisi probleme, Editura Facla, Timisoara, 1976.

4. N. Boboc, I. Colojoara - Matematica, manual pentru clasa a XII-a, Elemente deanaliza matematica, E.D.P. Bucuresti, 1981.

36

Page 41: Revista (format .pdf, 1.6 MB)

O provocare - problemele nonstandard

Ileana ROSCAN si Valerica BELDIMAN 1

Marele nostru matematician Gheorghe Titeica afirma: ”Studiul stiintific al feno-menelor naturii cauta sa ia forma matematica si acest studiu se socoteste deplin,când forma matematica a fost gasita”.Iata deci ca, pentru cunoasterea deplina a fenomenelor, se impune stabilirea unor

relatii numerice care sa redea adevarata masura a elementelor luate în conside-ratie. Acest lucru este prezent în toate manualele scolare, urmarindu-se aplicareaprocedeelor sau algoritmilor specifici fiecarei teme în scopul formarii deprinderilorcorecte de calcul, fara a se neglija dezvoltarea gândirii logice si a calitatilor acesteia:fluenta, flexibilitate, creativitate.Exista însa si un alt mod de abordare a matematicii care, atât prin formularea

enuntului cât si prin modul de rezolvare, îl aduce pe elev mai aproape de universullui, de ”problemele” vietii de fiecare zi: acestea sunt problemele nonstandard. Eleofera posibilitatea dezvoltarii gândirii elevilor pe un plan mai general, prin eliminareatiparelor, a sablonismului.Prin acest articol ne propunem sa le readucem în actualitate si va provocam la

rezolvarea si compunerea unor astfel de probleme. Efectele vor fi fantastice.Va sugeram sa încercati cu urmatoarele enunturi folosind solutiile numai în cazul

în care nu aveti alta cale de rezolvare.

1. Gasiti toate numerele de forma xyz ce au suma cifrelor 17, iar cifra suteloreste cu 4 mai mare decât cifra unitatilor.Solutie. Avem x + y + z = 17 si x = z + 4. Aceste relatii conduc la egalitatea

2z + y + 4 = 17⇔ 2z + y = 13 din care deducem 2 ≤ z ≤ 5 si y nu este numar par.y 1 3 5 7 9

2z + y 13 13 13 13 13z 6 5 4 3 2

Obtinem xyz ∈ 692, 773, 854, 935.2. Determinati numarul numerelor naturale de forma abc scrise cu cifre impare.Solutie.

Pentru fiecare cifra impara pusa pe locul al doilea, pe ultimul loc putem puneuna din cifrele 1,3,5,7,9, ceea ce înseamna ca, pentru orice cifra pusa pe primul loc,

1 Învatatoare, Scoala ”Ionel Teodoreanu”, Iasi

37

Page 42: Revista (format .pdf, 1.6 MB)

avem 5 · 5 = 25 numere cu cifre impare pe ultimele doua locuri. Deoarece pe primulloc putem aseza cinci cifre impare, concluzionam ca avem 5 · 25 = 125 numere.3. Se considera expresia 1¤ 2¤ 3¤ . . . ¤ 10. Sa se arate ca oricum am înlocui

casetele cu semnele grafice ”+” sau ”−”, astfel încât operatiile sa fie posibile, nuputem obtine rezultatul zero.Solutie. Daca am obtine rezultatul zero înseamna ca exercitiul s-ar transforma

într-o scriere de forma S1 − S2 = 0, unde S1 reprezinta suma numerelor care seaduna, iar S2 este suma numerelor care se scad. Deducem ca S1 = S2, iar S1+S2 == 1 + 2 + 3 + . . .+ 10 = 55, fals, deoarece S1 si S2 sunt egale.

4. Numarul 37 a fost scris cu ajutorul a cinci cifre de 3 în felul urmator:

37 = 33 + 3 +3

3. Gasiti un alt mijloc pentru a exprima numarul 37 cu ajutorul

a cinci cifre de 3.

Solutie. 37 =333

3× 3 .

5. Aratati ca fractia2002

n (n+ 1)nu poate fi echiunitara oricare ar fi numarul

natural n nenul.Solutie. Pentru ca fractia sa fie echiunitara este necesar sa avem 2002 = n (n+ 1),

ceea ce ne spune ca 2002 trebuie scris ca produsul a doua numere consecutive.Avem 44 · 45 = 1980, 45 · 46 = 2070 de unde deducem ca orice alt produs de

numere consecutive este mai mic decât 1980 sau mai mare decât 2070. Concluzionamca numarul 2002 nu poate fi scris ca un produs de numere consecutive, deci fractianu este echiunitara.

6. La o serbare vin 20 de tineri, baieti si fete. Unul din baieti a dansat cu 5 fete,un al doilea a dansat cu 6 fete, un al treilea cu 7 fete si asa mai departe. Ultimul adansat cu toate fetele. Câti baieti si câte fete au luat parte la serbare?Solutie. Faptul ca primul baiat a dansat cu 5 fete poate fi reprezentat prin:

1→ 1 + 4. Continuând în acelasi fel, avem:2→ 2 + 43→ 3 + 4. . . . . . . . . . . .n→ n+ 4unde n+4 reprezinta numarul fetelor, iar n reprezinta numarul baietilor, deci numarulfetelor este cu 4 mai mare decât numarul baietilor. Gasim

n = (20− 4) : 2 = 16 : 2 = 8.La serbare au luat parte 8 baieti si 12 fete.

Probleme propuse.1. Sa se determine numarul numerelor de forma abc scrise cu cifre pare.2. Scrieti numarul 100 cu ajutorul a sase cifre identice.3. Scrieti numarul 55 folosind cinci cifre de 4.4. Scrieti numarul 20 cu ajutorul a patru cifre de 9.

38

Page 43: Revista (format .pdf, 1.6 MB)

Concursul ”Adolf Haimovici”, editia a VI-apentru liceele economice, industriale si agricole

Nota. Toate subiectele sunt obligatorii. Timp de lucru: 2 ore.

Faza judeteana, 23 februarie 2002Clasa a IX-aI. Fie ecuatia mx2 + (3m− 1)x+ 2m− 1 = 0, m ∈ R∗.a) Determinati m ∈ R asa încât x1, x2 > 0.b) Determinati o relatie între solutii, independenta de m.

II.1. Rezolvati inecuatiax− 1x+ 1

≤ m− 1m+ 1

, m ∈ R\ −1. Discutie.

2. Aratati ca daca x+ y ≥ 1, atunci x2 + y2 ≥ 12.

3. Determinati p, q ∈ R si solutiile x1, x2 ale ecuatiei x2 + px + q = 0, stiind cax1 + 1, x2 + 1 sunt solutiile ecuatiei x2 − px+ q = 0.

III.1. Aflati partea întreaga a numarului1

5√2− 7 +

√3.

2. Sa se rezolve ecuatia·x− 23

¸=

x+ 2

4.

3. Determinati α asa încât solutiile ecuatiei¡1 + α2

¢x2+(1 + α)x+α (1− α) = 0,

sa verifice inegalitatea −1 ≤ 1

x1+1

x2+

1

x1x2≤ 0.

Clasa a X-aI.1. Rezolvati inecuatia

3 · 2x−13x − 2x > 1 +

µ2

3

¶x.

2. Rezolvati sistemul

µ3

2

¶x−y−µ2

3

¶x−y=65

36xy − x+ y = 118

.

II.1. Rezolvati inecuatia loga 2 + loga¡4x−2 + 9

¢ ≤ 1 + loga ¡2x−2 + 1¢.2. Sa se rezolve si sa se discute dupa m ∈ R ecuatia lg

µx+

5

x

¶− lg

µ1

x+2

m

¶=

= lgm.III.1. Determinati m ∈ R, asa încât ecuatiap2 |x|− 2x = 5m−x sa admita trei

solutii reale distincte.2. Fie (an)n∈N∗ o progresie aritmetica de ratie r, iar (bn)n∈N∗ o progresie geo-

metrica de ratie q. Sa se calculeze în functie de a1, r, b1, q suma S =a1b1+

a2b2+

+ . . .+anbn.

Clasa a XI-aI. Fie matricea A =

µ1 10 1

¶∈M2 (N).

a) Sa se determine toate matricele X ∈M2 (R), pentru care A2X = XA2.

39

Page 44: Revista (format .pdf, 1.6 MB)

b) Sa se determine (daca exista) matricele Y ∈M2 (R) cu proprietatea caA2Y − Y A2 = I2.

c) Calculati An, n ∈ N∗.II. Fie x1, x2, x3 solutiile ecuatiei x3 + x2 + ax+ b = 0, a, b ∈ R.

a) Calculati ∆ =

¯¯ x1 x2 x3x3 x1 x2x2 x3 x1

¯¯ în functie de a si b.

b) Aratati ca

¯¯ 1 1 1x1 x2 x3x21 x22 x23

¯¯ = (x3 − x1) (x3 − x2) (x2 − x1).

c) Pentru ∆ = 0, b = −37 sa se rezolve ecuatia data.III. Fie sirul (xn)n∈N definit de relatia: xn+1 =

1

2

µxn +

1

xn

¶, x0 > 1.

a) Aratati ca xn > 1.b) Studiati monotonia sirului (xn)n∈N.c) Determinati lim

n→∞xn în functie de x0.

Clasa a XII-aI.1. Fie a, b, c ∈ R si F (x) =

¡ax2 + bx+ c

¢√3− 2x o primitiva a functiei

f : D→ R (D ⊂ R), f (x) = x√3− 2x. Calculati numarul F (1)− F (0).

2. Sa se calculeze integralaZ

dx

x4 + 9.

II. Fie multimea G =½M (x) =

µ2− x x− 12 (1− x) 2x− 1

¶|x ∈ R∗

¾.

a) Aratati ca exista si sunt unice matricele A,B ∈M2 (R), ce nu contin x astfelîncât M (x) = A+ xB, ∀x ∈ R∗.

b) Demonstati ca G formeaza grup abelian în raport cu operatia de înmultire amatricelor.

c) Aratati ca¡R∗+, ·

¢ ' (G, ·).III. Fie functiile f, g,G : [1,∞)→ R, f (x) =

1

x (1 + lnx) (2 + lnx),

g (x) =1

x (a+ lnx), a > 0 si G (x) = ln (a+ lnx).

a) Sa se arate ca G este o primitiva a lui g.b) Daca F este o primitiva a lui f , calculati I (x) = F (x)− F (1).c) Calculati lim

x→∞ I (x).

Faza interjudeteana, 18 mai 2002

Clasa a IX-aI. Fie familia de functii fm (x) = mx2 − (2m− 1)x+m− 1, m ∈ R∗.a) Sa se determine m asa încât vârfurile parabolelor asociate acestor functii sa se

afle pe dreapta de ecuatie y = x.b) Sa se arate ca parabolele familiei trec printr-un punct fix.II. Sa se rezolve:a)px+ 3− 4√x− 1 +

px+ 8− 6√x− 1 = 1.

40

Page 45: Revista (format .pdf, 1.6 MB)

b)

½ ¯x2 − 3x+ 2¯+ 4y = 4|x− 2|+ |y − 1| = 1 .

III. Fie expresia: f (x) =x4 + 2ax3 + 3a2x2 + 2a3x+ a4 + 9

x2 + ax+ a2, unde |a| < 2.

a) Sa se calculeze¡x2 + ax+ a2

¢2.

b) Sa se afle x pentru care f (x) are un minim.

Clasa a X-aI. Sa se rezolve ecuatia

qlogx√3x · log3 x = −1.

II. a) Sa se demonstreze identitatea: C0n−C1n+C2n+ . . .+(−1)nCnn = 0, ∀n ∈ N∗.

b) Fie x1, x2, . . . , xn, xn+1, termeni consecutivi ai unei progresii aritmetice. Ara-tati ca: x1 − C1nx2 + C2nx3 − C3nx4 + . . .+ (−1)nCn

nxn+1 = 0, ∀n ∈ N, n ≥ 2.

III. Termenul al patrulea al dezvoltarii binomului

Ã5√a4

x√ax−1

+ ax+1√ax−1

!8este

56a11/2. Sa se determine x.

Clasa a XI-a

I. Din formula sinx+sin 2x+ . . .+sinnx =sin¡n+12 x

¢sin nx

2

sin x2

, sa se deduca suma

primelor n numere naturale nenule.II. ÎnM2 (R), multimea matricelor patratice de ordinul doi, se considera matricea

X (a) =

µ1 + 5a 10a−2a 1− 4a

¶, a ∈ R.

a) Sa se arate ca oricare ar fi a, b ∈ R are loc X (a) ·X (b) = X (ab+ a+ b).b) Sa se determine a ∈ R pentru care matricea X (a) este inversabila si sa se afle

inversa sa.c) Sa se demonstreze, utilizând metoda inductiei matematice, ca pentru oricare

n ∈ N∗ avem (X (1))n= X (2n − 1).

III. Sa se studieze continuitatea si derivabilitatea functiei f : R→ R,

f (x) =

½x2, daca x ∈ Q

2x3, daca x ∈ R\Q .

Clasa a XII-aI. Fie G = (−1, 2) si asocierea x ∗ y def

=xy + 4x+ 4y − 22xy − x− y + 5

, ∀x, y ∈ G.

a) Sa se arate ca (G, ∗) este grup abelian.b) Sa se arate ca functia f : G → R∗+, f (x) =

−x+ 2x+ 1

este izomorfism între

grupurile (G, ∗) si ¡R∗+, ·¢.c) Calculati x ∗ x ∗ . . . ∗ x| z

de 10 ori

.

II. Fie sirul (xn)n≥1, definit prin xn =

Z 3

2

1

|n− x|+ 2dx. Sa se studieze conver-genta sirului si în caz afirmativ sa se afle limita sa.

III. Fie F acea primitiva a functiei f : R→ R, f (x) =x2 − 1

x4 + x2 + 1, pentru care

F (0) = 0. Sa se calculeze F (1) + F (−1).

41

Page 46: Revista (format .pdf, 1.6 MB)

Concursul ”Florica T. Câmpan”, editia a II-aNota. Toate subiectele sunt obligatorii. Timp de lucru: 1.5 ore - cl. IV si 2 ore - cl. V-VIII.

Faza judeteana, 16-17 martie 2002

Clasa a IV-aI.1. Andrei este mai mare decât Mihai. Mihai este mare decât Mihaela. Ioana

este mai mare decât Andrei si mai mica decât Alexandra, care este mai mica decâtMaria. Care este copilul cel mai mare? Dar cel mai mic?2. În ce coloana si în care rânduri este nepotrivirea în aceste siruri de cifre?

Ia) 7 3 2 5b) 5 4 9 1c) 4 4 2 4

IId) 2 5 2 7e) 4 2 3 2f) 1 4 2 5

3. Separati 5 kg de faina din 16 kg prin doua cântariri, folosind o balanta si ogreutate de 2 kg.II.1. Patronul unei fabrici de sârma cumpara 1000 kg de fier vechi cu 1000 de lei

kilogramul. Dupa topire se pierde jumatate din cantitatea de fier cumparata. Dacapentru 10 m de sârma se folosesc 5 kg de fier topit, iar cheltuielile de fabricare seridica la 500 000 lei, cu cât trebuie sa vânda 1 metru de sârma ca sa obtina un profitde 1 000 000 lei?2. Fiecare din cei sapte pitici a trimis câte un cadou celorlalti. Câte cadouri s-au

trimis în total?III. Lucian-Georges a rezolvat în trei saptamâni un numar de probleme astfel

încât în prima saptamâna a rezolvat de doua ori mai mult decât în a doua, iar în atreia saptamâna cu 10 probleme mai mult decât de 4 ori cât a rezolvat în primeledoua saptamâni. Câte probleme a rezolvat în fiecare saptamâna daca în ultima arezolvat cu 100 de probleme mai mult decât în celelalte saptamâni?

Clasa a V-aI.1. Numarul natural n are numai doi divizori d1 si d2 în multimea N. Se stie ca

d1 + d2 = 29. Sa se afle numarul n. Aceeasi întrebare daca d1 + d2 = 30.

2.

•• • •

• • • • • •• • • • • • • • • •

. . .

În figura de mai sus sunt reprezentate prin bile numerele 1, 3, 6, 10, . . . . De câtebile este nevoie pentru a alcatui figura corespunzatoare pasului 2002?II. În 25 de cutii sunt bomboane de aceeasi forma si marime. Toate bomboanele

din 24 de cutii au masa de 16 g, iar cele din a 25-a cutie au masa de 17 g. Fiecarecutie contine exact 30 de bomboane. Avem la dispozitie un cântar gradat. Cumputem determina printr-o singura cântarire cutia cu bomboane mai grele?III. M-am trezit noaptea si m-am uitat la ceas. Arata ora 200. Apoi mi-am dat

seama ca ceasul era oprit. L-am întors si am adormit din nou. Când m-am trezit,la radio s-a anuntat ora 700, în timp ce ceasul meu indica ora 530. La ce ora m-amtrezit noaptea?

42

Page 47: Revista (format .pdf, 1.6 MB)

Clasa a VI-aI. În câte moduri se poate completa patratul alaturat cu numerele 1

sau −1, astfel încât produsul numerelor de pe fiecare linie si de pe fiecarecoloana sa fie −1?II. Un negustor de animale de casa a cumparat un anumit numar de soareci albi

si jumatate pe atâtea perechi de papagali. El a platit 2 $ pentru fiecare soarecesi 1 $ pentru fiecare papagal. Pretul de vânzare l-a fixat cu 10 % peste pretul deachizitie. Dupa ce a vândut toate animalele (soareci si papagali) - în afara de sapte -negustorul constata ca a încasat pe ele exact suma totala platita de el la cumparare.Câstigul negustorului este dat de banii pe care îi încaseaza în urma vânzarii ultimelor7 animale. Cât este acest câstig?III. Fie un triunghi ABC cu AB = AC = 3 si BC = 4. Consideram punctele

oarecare în plan D1, D2, D3, . . . , D1001. Sa se arate ca exista un punct M ∈[AB] ∪ [BC] ∪ [CA] astfel încât MD1 +MD2 + . . .+MD1001 ≥ 2002.

Clasa a VII-aI. Doi frati, Nicu si Mihai, însotiti de fiii lor, Razvan si Rares, cumpara carti.- Ce curios, constata unul dintre copii, fiecare dintre noi, am platit pe fiecare carte

cumparata un numar de euro egal cu numarul cartilor cumparate si fiecare tata acheltuit cu 15 euro mai mult decât fiul sau.- Si în plus Nicu a cumparat cu 3 carti mai putin decât Razvan, remarca Mihai.Cum se numeste tatal lui Rares?II. Sa se determine trei numere naturale nenule, diferite doua câte doua, ale caror

inverse, adunate, dau un numar natural.III.Din patratul din fig.1, prin rearanjarea partilor componente (1, 2, 3, 4) obtinem

triunghiul din fig.2. Aria patratului este 25 (u2), iar aria triunghiului este 24 (u2),adica 25 = 24. Unde este greseala?

Fig.1 Fig.2

Clasa a VIII-aI.1. Fie Gf = [AB]∪ (CD)∪ [EF ] graficul unei functii în plan, unde A (−4,−2);

B (−2, 0); C (−2, 1); D (2,−1); E (2, 0); F (4, 4). Sa se cerceteze daca punctele

M (5, 5), Nµ−1, 1

2

¶si P (−3,−1) apartin graficului functiei.

2. Sa se arate caµ1 +

13

7

¶6+

µ1 +

7

13

¶6> 27.

43

Page 48: Revista (format .pdf, 1.6 MB)

II. Patru pârtii de schi rectilinii au ca punct de plecare, în directii diferite, vârfulunui munte. Pe fiecare pârtie porneste un schior care se deplaseaza cu viteza con-stanta. Stiind ca la momentul t1 schiorii se afla în patru puncte coplanare, sa searate ca la momentul t2 punctele în care se afla sunt coplanare.III.1. În interiorul unui paraleliped dreptunghic cu dimensiunile a, b, c se conside-

ra n3+1 puncte distincte. Sa se arate ca exista cel putin doua puncte cu proprietatea

ca distanta dintre acestea nu este mai mare decât1

n

√a2 + b2 + c2.

2. Care sunt valorile întregi ale lui x pentru care suma ab + x2 este un patratperfect, daca a si b sunt numere prime mai mari decât 2?

Faza interjudeteana, 25 mai 2002

Clasa a IV-aI.1. Daniel si-a facut un joc de bile. Daca lovea o bila, aceasta lovea alte 4 bile,

care la rândul lor loveau câte 5 bile, iar acestea loveau câte 6 bile. Câte bile erau înjoc?2. Mutând doar 3 monede din desenul alaturat, rearanjeaza triun-

ghiul format din acestea astfel încât sa fie cu vârful în jos.

II.1. Puneti semnul + (plus) între cifrele numarului 987654321, astfel ca sumanumerelor care rezulta sa fie egala cu 99.2. Doi copii, Andrei si Bogdan, sunt buni prieteni. Bogdan poseda o bicicleta.

Daca Andrei îi da lui Bogdan 2 ciocolate, atunci Bogdan îi împrumuta bicicleta sapentru 3 ore. Daca Andrei îi da lui Bogdan 28 de caramele, el obtine bicicleta luiBogdan pentru 2 ore. Aflati pentru cât timp va primi Andrei bicicleta lui Bogdan încazul în care Andrei îi ofera lui Bogdan o ciocolata si 7 caramele.III. Ionel se joaca pe scara blocului formata din 15 trepte. El urca 3 trepte si

coboara apoi 2. Câti pasi face Ionel ca sa ajunga sus, cu ambele picioare, tinând contde faptul ca treptele se urca una câte una.

Clasa a V-aI. Un numar natural par are 2003 divizori. Aflati numarul.II. Într-un concurs de sah participa 32 de elevi. La fiecare etapa se formeaza

grupuri de câte 4 elevi în care fiecare joaca cu fiecare. Primii doi din fiecare grupaintra în etapa urmatoare. În final ramân 2 concurenti, dintre care unul va câstigaconcursul. Câte partide s-au jucat în total?III. Daca sapte capre si patru iezi manânca 100 de verze, câte verze manânca o

capra si un ied împreuna? (fiecare ied manânca acelasi numar de verze întregi, fiecarecapra manânca acelasi numar de verze întregi). Aratati ca problema are solutie unica.

Clasa a VI-aI. Se dau trei fisicuri de monede asezate vertical, asupra carora putem efectua

una dintre operatiile O1: ”luam cele doua monede de deasupra unui fisic si le asezampeste altul”, sau O2: ”luam cele doua monede de deasupra unui fisic si le asezamcâte una peste fiecare dintre celelalte doua fisicuri”.

44

Page 49: Revista (format .pdf, 1.6 MB)

a) Gasiti o conditie necesara pentru ca, dupa un numar de operatii, toate fisicurilesa contina la fel de multe monede;

b) Aratati ca aceasta conditie nu este suficienta daca este permisa o singura ope-ratie, însa este suficienta în cazul în care sunt permise amândoua.

G.Popa, Recreatii matematice - 1/2002II. Alegeti în fata fiecaruia dintre numerele 1, 2, 3, . . . , 2002 unul dintre semnele

”+” sau ”−” astfel încât numarul |±1± 2± 3 . . .± 2002| sa ia cea mai mica valoareposibila.III.1. Pe o masa de biliard cu dimensiunile 2m×6m se lanseaza din mijlocul

laturii mari o bila a carei traiectorie face la plecare un unghi de 45 cu latura. La a59-a ciocnire, la câti metri de punctul de plecare se afla bila?2. Platipus Pytagoricus (ornitorincul matematician) depune oua perfect sferice

cu diametrul de 3 cm. El face câte un singur ou si îl îngroapa cu precizie la 200 m desursa de apa cea mai apropiata si la 670 m de un eucalipt. Niciodata doua astfel deanimale nu-si îngroapa oul în acelasi loc. Câte oua pot spera sa gasesc în zona unuirâu rectiliniu si a unui eucalipt?

Clasa a VII-aI. Fie 4ABC oarecare si A1, B1, C1 picioarele înaltimilor din A, B si respectiv

C. Ce conditii trebuie impuse 4ABC pentru ca relatia 4ABC ∼ 4A1B1C1 sa aibaloc?

P.Bîrsan, Recreatii matematice - 2/2001II. 1. Fie A = 1, 2, 3, . . . , 2002 si B = 1, 2, 3, . . . , 145, 147, 148, . . . , 2002.

Comparati media aritmetica a numerelor din multimea A, cu media aritmetica anumerelor din multimea B.2. Într-o urna se gasesc n bile numerotate de la 1 la n. Un magician solicita

unui spectator sa extraga o bila si sa-i comunice media aritmetica a bilelor ramase înurna. Spectatorul comunica magicianului ca media solicitata este 51, 15 , dupa caremagicianul a indicat corect numarul bilei extrase. Voi stiti cum a procedat?

N.Miron si J.Grigoras, IasiIII. Se da paralelogramul ABCD. Sa se explice cum se poate determina mijlocul

M al segmentului [AB] si apoi cum se poate desena paralela prin M la AC, folosindnumai o rigla negradata.

M.Gavrilut, Roman

Clasa a VIII-aI. Fie functia f : R→ R cu proprietatea f (t) + f ([t]) · f (t) = t, ∀t ∈ R.a) Determinati functia f .b) Trasati graficul functiei f pe intervalul [1, 2].II. Pe un satelit sferic cu raza de 2 unitati sunt amplasate la întâmplare 9 statii

(puncte) de emisie-receptie. Demonstrati ca exista doua statii pe satelit aflate la odistanta ce nu depaseste π.III. a) Sa se arate ca pentru orice alegere a 12 numere naturale consecutive nu se

pot numerota muchiile unui cub astfel ca suma numerelor aflate pe 3 muchii care auun vârf comun sa fie aceeasi pentru toate vârfurile cubului (nu se numeroteaza douamuchii cu acelasi numar).

45

Page 50: Revista (format .pdf, 1.6 MB)

b) Sa se arate ca este posibila numerotarea descrisa daca se aleg convenabil 12numere dintre oricare 13 numere naturale consecutive.

C.Chirila, Recreatii Matematice - 1/2001

Premiile acordate:Etapa judeteanaClasa a IV-a: Constantinescu Andra (Sc. nr. 33 ”M. Kogalniceanu”) - premiul I; Munteanu Alexandru

(Sc. nr. 15 ”Stefan Barsanescu”) - premiul II; Hodea Cosmin (Lic. ”Vasile Alecsandri”) - premiul III.

Clasa a V-a: Pui Ariadna (C. N. ”Emil Racovita”) - premiul I; Gaftea Alexandru (Col. National) -

premiul II; Temneanu Bogdan (C. N. ”C. Negruzzi”) - premiul III.

Clasa a VI-a: Codrea Sorina (C. N. ”Emil Racovita”) - premiul I; Mihaila Laura (Col. National) -

premiul II; Frunza Tudor Cristian (C. N.”C. Negruzzi”) - premiul III.

Clasa a VII-a: Tiganas Vladimir (Sc. nr. 42 ”N. Iorga”) - premiul I; Iuga Andreea (Sc. nr. 15 ”Stefan

Bârsanescu”) - premiul II; Vasiliu Lucian Andrei (Sc. nr. 13 ”Alexandra cel Bun”) - premiul III.

Clasa a VIII-a: Ionescu Claudiu (C. N. ”C. Negruzzi”) - premiul I; Ceobanu Vlad (Lic. Informatica)

- premiul II; Bentea Eduard (C. N. ”Emil Racovita”) - premiul III.

Etapa interjudeteanaClasa a IV-a: Constantinescu Andra (Sc. nr. 33 ”M. Kogalniceanu”, Iasi) - premiul I; Tacuteanu -

Liucu Vlad (Sc. nr. 9, Iasi) - premiul II; Asandei Alexandra (Sc. nr. 15 ”Stefan Bârsanescu”, Iasi) - premiul

III.

Clasa a V-a: Hodea Victor (C. N. ”Ferdinand I”, Bacau) - premiul I; Gaftea Alexandru (Col. National,

Iasi) - premiul II; Pui Ariadna (C. N. ”Emil Racovita”, Iasi) - premiul III.

Clasa a VI-a: Rosu Eugenia (C. N. ”C. Negruzzi”, Iasi) - premiul I; Maftei Mihnea (Sc. nr. 1, Roman)

- premiul II; Crânganu Ioana (Lic. Pedagogic, Galati) - premiul III.

Clasa a VII-a. Stoica Catalina (Sc. nr. 10, Focsani) - premiul I; Balusescu Alina (Sc. nr. 1, Roman) -

premiul II; Barat Marius (Sc. nr. 10, Piatra-Neamt) - premiul III.

Clasa a VIII-a: Istrate Adriana (Lic. ”Stefan cel Mare”, Târgu-Neamt) - premiul I; Popovici Teodora

(C. N. ”Ferdinand I”, Bacau) - premiul II; Havârneanu Raluca (C. N. ”C. Negruzzi”, Iasi) - premiul III.

Concursul de matematica ”Traian Lalescu”Editia a III-a, Colegiul ”C.Negruzzi”, Iasi, 11 mai 2002

1. Determinati valoarea lui a din egalitatea 5− 2 · [(6 + a) : 3− 2] · 7+3 = 24.2. a) Sa se calculeze: 1 + 2 + 3 + . . .+ 100.b) Suma a 100 de numere naturale distincte, diferite de zero este 5051. Sa se afle

numerele.3. Într-o clasa 28 de elevi stau câte doi în 14 banci. La începutul fiecarei luni

învatatorul îi redistribuie astfel încât în fiecare banca sa stea doi elevi care nu au maistat pâna atunci împreuna.

a) Care este numarul maxim de luni în care învatatorul poate efectua acesteschimbari?

b) Cum îi poate aranja învatatorul pe elevi?4. Sa se arate ca într-un grup de 2002 persoane exista doua care au acelasi numar

de cunoscuti printre ceilalti. (Se admite ca daca persoana X cunoaste pe Y , atuncisi Y cunoaste pe X).

46

Page 51: Revista (format .pdf, 1.6 MB)

Olimpiada de matematica - faza judeteana, IasiBaraj, clasa a X-a, 21 februarie 2002

1. Fie Z [i] = a+ ib | a, b ∈ Z si z1, z2 ∈ Z [i]. Sa se demonstreze ca urmatoareleafirmatii sunt echivalente:(i) z2 = z1 sau z2 = z1; (ii) 2Re z1 + 2Im z1 = 2Re z2 + 2Im z2 .

Vlad Martinusi, Iasi

2. Se considera inegalitatea:

|cosnx| ≤ n · |cosx| , n ∈ N∗, x ∈ R (∗)a) Sa se determine n stiind ca (∗) este adevarata pentru orice x ∈ R;b) Sa se determine x stiind ca (∗) este adevarata pentru orice n ∈ N∗.

Gheorge Iurea, Iasi

3. Se considera 4ABC, M un punct oarecare în interiorul sau. Fie D, E,F mijloacele laturilor [BC], [CA], respectiv [AB] si fie punctele M1, M2, M3 pesemidreptele (MD, (ME, respectiv (MF , în exteriorul triunghiului, astfel încât:MD

DM1=

ME

EM2=

MF

FM3. Sa se demonstreze ca dreptele AM1, BM2 si CM3 sunt

concurente.Gabriel Popa si Paul Georgescu, Iasi

4. Demonstrati ca în orice interval (a, b) ⊂ (0, 1) nu exista numere rationale deforma lgn− [lgn], n ∈ N∗, dar exista o infinitate de numere reale de aceasta forma.

Gheorghe Iurea, Iasi

Olimpiada de matematica - clasele V - VIFaza judeteana, 18 mai 2002, Iasi

Clasa a V-a1. a) Un numar de trei cifre împartit la rasturnatul sau da câtul 2 si restul 100,

iar diferenta dintre cifra sutelor si cea a unitatilor numarului este 4. Aflati numarul.

b) Sa se rezolve în N3 ecuatia x · y = z + 2

z − 1 . Cristina Vâlcu, Iasi

2. a) Sa se arate ca1

12+1

22+ . . .+

1

20022< 2.

b) Fie d1, d2, . . . , dn toti divizorii naturali ai numarului a. Sa se arate ca(d1 · d2 · . . . · dn)2 = an.

3. Aflati cifrele a, b în baza 10 pentru care este adevarata egalitateaa

b= b, a.

Clasa a VI-a1. Fie numerele a1, a2, . . . , a10 ∈ −1, 1 si suma S = |a1 + a2| + |a2 + a3|+

+ |a3 + a4|+ . . .+ |a9 + a10|.a) Determinati cea mai mica si cea mai mare valoare pe care o poate lua S;b) Aratati ca S nu poate lua valoarea 15.

Ciprian Baghiu, Iasi

47

Page 52: Revista (format .pdf, 1.6 MB)

2. Fie numerele rationale nenule a1, a2, . . . , a2002 invers proportionale cu nu-

merele 1, 2, . . . , 2002. Sa se afle numarul rational x daca 2002 · a2002 = x³a12+

a23+

+ . . .+a20022003

´.

Sergiu Prisacariu, Iasi3. Fie [AB] un segment si C ∈ (AB). De aceeasi parte a dreptei AB se constru-

iesc triunghiurile echilaterale ACD si CBE. Daca M este mijlocul lui [AE], iar Nmijlocul lui [BD], aratati ca:

a) [AE] ≡ [BD];b) triunghiul CMN este echilateral;c) MNkAB daca si numai daca [AC] ≡ [CB].

Gheorge Iurea, Iasi

SolutiiClasa a V-a1. a) Din ipoteza, abc = 2cba+ 100 si atunci c este o cifra para. Cum a− c = 4,

a este o cifra para mai mare decât 4. Considerând succesiv cazurile a = 6 si a = 8,obtinem singura solutie acceptabila abc = 692.

b) Trebuie caz + 2

z − 1 ∈ N, de unde z − 1 | z + 2. Însa z − 1 | z − 1, deci z − 1 |(z + 2)− (z − 1), adica z − 1 | 3. Rezulta ca z ∈ 2, 4. Analizând cele doua cazuri,obtinem solutiile (1, 4, 2), (2, 2, 2), (4, 1, 2), (1, 2, 4), (2, 1, 4).

2. a) Evident ca1

12+1

22+ . . . +

1

20022< 1 +

1

1 · 2 + . . . +1

2001 · 2002 . Însa1

1 · 2 +1

2 · 3 + . . .+1

2001 · 2002 =1

1− 12+1

2− 13+ . . .+

1

2001− 1

2002= 1− 1

2002< 1,

de unde concluzia.b) Putem presupune d1 < d2 < . . . < dn; atunci d1 =

a

dn, d2 =

a

dn−1, dn =

a

d1.

Concluzia rezulta prin înmultirea acestor relatii membru cu membru.

3. Observam ca b 6= 0 si atuncia

bnu poate fi fractie subunitara, deci a ≥ b.

Obtinem solutia a = 0, b = 1 si putem presupune în continuare a > b ≥ 2. Deoarecea

beste fractie neperiodica, numitorul sau poate fi doar 2, 4, 5 sau 8. Analizând fiecare

situatie mai obtinem doar solutia a = 5, b = 2.

Clasa a VI-a1. a) Evident ca S ≥ 0 si deoarece pentru a1 = a3 = . . . = a9 = 1, a2 = a4 =

= . . . = a10 = −1 obtinem S = 0, rezulta ca minimul sumei este 0. Observam caS ≤ 18 si pentru a1 = a2 = . . . = a10 = 1 obtinem S = 18, deci maximul sumei este18.

b) În interiorul fiecarui modul avem unul dintre numerele −2, 0, sau 2, deci S esteun numar par, adica S 6= 15.2. Deoarece 1 · a1 = 2 · a2 = . . . = 2002 · a2002, obtinem ca a2 =

a12, a3 =

a13,

. . . , a2002 =a12002

. Înlocuind în ipoteza, a1 = x³a12+

a12 · 3 + . . .+

a12002 · 2003

´si

cum1

1 · 2 +1

2 · 3 + . . .+1

2002 · 2003 = 1−1

2003=2002

2003, rezulta ca x =

2003

2002.

48

Page 53: Revista (format .pdf, 1.6 MB)

3. a) Se observa imediat ca 4DCB ≡ 4ACE(LUL), de unde [DB] ≡ [AE].

b) Se arata ca 4ACM ≡ 4DCN (LUL),deci [CM ] ≡ [CN ] si \ACM ≡ \DCN . Atuncim(\MCN) = m(\ACD) − m(\ACM) + m(\DCN) ==60, adica 4CMN este isoscel cu un unghi de 60,de unde concluzia.

c) Daca MNkAB, atunci m(\NCB) = m(\CNM) = 60 si cum m(\CAD) = 60,rezulta ca NCkAD. Însa N este mijlocul lui [BD], deci NC va fi linie mijlocie în4BAD, adica C este mijlocul lui [AB]. Reciproca se demonstreaza usor.

Premiantii Olimpiadei de Matematica - Premiile M.E.C.Faza finala, Râmnicu Vâlcea, 2002

Clasa a VII-a: Timofte Diana (C. N. ”E. Racovita”), Vâlcu Ruxandra (Col. National) - mentiune.

Clasa a VIII-a: Pachitariu Marius (Col.National), Havârneanu Raluca (C.N. ”C.Negruzzi”) -mentiune.

Clasa a IX-a: Iacob Alin (C. N. ”C. Negruzzi”) - mentiune.

Clasa a X-a: Anton Constantin, Romanescu Razvan (ambii C. N. ”C. Negruzzi”) - mentiune.

Clasa a XI-a: Sucila Andrei (C. N. ”C. Negruzzi”) - mentiune.

Calificati în lotul largit pentru a 6-a O. B. M. J.: Pachitariu Marius, Mustata Irina, Vâlcu Ruxandra

(toti Col. National).

Premiantii Olimpiadei de MatematicaFaza judeteana, Iasi

Clasa a V-a: Gaftea Alexandru, Neagu Victor (ambii Col. National), Vieriu Antonela (C. N. ”C. Negru-

zzi”), Cozma Andrei (C. N. ”E. Racovita”) - premiul I; Bivol Mark (C. N. ”C. Negruzzi”), Co jocaru Alexan-

dra (Col. National), Lazar Laura (C. N. ”E. Racovita”), Pricop Mircea (Lic. Informatica), Sârbu Cristian,

Temneanu Bogdan, Daneliuc Ana, Costea Ramona, Ibanescu Tudor (toti C. N. ”C. Negruzzi”), Ungureanu

Dragos (Lic. ”G. Ibraileani”), Alexandru Ion (Lic. ”M. Eminescu”), Galearschi Ana (C. N. ”C. Negruzzi”),

Haures Sergiu (Col. National), Gavriluta Alina, Mustata Andrei (ambii C. N. ”C. Negruzzi”), Simon Teodo-

ra (C. N. ”E. Racovita”) - premiul II; Arhip Andra (Col. National), Petrov Robert (Lic. Informatica),

Airimitoaie Andrei, Luchian Alexandra, Tudori Ioana (toti C. N. ”C. Negruzzi”), Conea Alexandru, Flo-

rea Ioana (ambii C. N. ”E. Racovita”), Volintiru Ioana, Vasiliu Ana, Cucu Lucian, Zaharia Diana (toti

C. N. ”C. Negruzzi”), Mahu Adriana, Afloarei Elena, Târcoveanu Filip, Bandac Alexandra (toti C. N. ”E. Ra-

covita”), Ilas George (Col. National), Chitac Razvan, Stoian Vlad (ambii Lic. Informatica) - premiul III.

Clasa a VI-a: Raileanu Ioana, Rosu Eugenia, Gâlca Octavia, Igna Razvan (toti C. N. ”C. Negruzzi”),

Baibarac Arina (Sc. nr. 22), Codrea Sorina (C. N. ”E. Racovita”), Istrate Ciobanu Ioana (Lic. ”M. Sadovea-

nu”, Pascani) - premiul I; Panaghiu Gheorghe (Lic. ”M. Eminescu”), Airinei Adriana (C. N. ”C. Negruzzi”),

Mihaila Laura (Col. National), Anita Flavia (Lic. ”M. Eminescu”), Echimov Tania (Lic. ”I. Neculce, Târgu

Frumos), Alexa Ioana (Col. National), Furcoi Vladut, Poiata Raluca (ambii C. N. ”C. Negruzzi”), Chiriac

Andreea (C. N. ”E. Racovita”), Diaconu Daniel (Col. National) - premiul II; Dima Georgiana (C. N. ”E. Ra-

covita”), Loghin Alexandru (Sc. ”P. Rares”, Hârlau), Carare Sabina (Lic. Informatica), Andron Georgia-

na (Col. National), Florea Alexandru, Ratoi Mihaita (ambii C. N. ”C. Negruzzi”), Dolhascu Simona (Lic.

”M. Sadoveanu”, Pascani), Didilescu Madalina, Ianus Andrada (Col. National), Diaconescu Alexandra

(C. N. ”E. Racovita”), Rosca Ioana (Lic. ”G. Ibraileanu”), Dumitrescu Bogdan (Sc. nr. 22) - premiul III.

49

Page 54: Revista (format .pdf, 1.6 MB)

Clasa a VII-a: Vâlcu Ruxandra (Col. National) - premiul I; Timofte Diana (C. N. ”E. Racovita”)

- premiul II; Gaina Razvan, Hudici Stefan, Teodorescu Horia (toti C. N. ”C. Negruzzi”), Zanoschi Delia

(Col. National), Mihul Andrei (Lic. Informatica), Tiganas Vladimir (Sc. nr. 42) - premiul III.

Clasa a VIII-a: Pachitariu Marius (Col. National) - premiul I; Havârneanu Raluca, Popovici George

(ambii C.N. ”C.Negruzzi”), Comandar Dragos (C.N. ”E.Racovita”) - premiul II; Berdan Alexandra (C. N.

”E. Racovita”) - premiul III.

Clasa a IX-a: Ichim Andrei (C. N. ”E. Racovita”) - premiul I; Mustata Irina (Col. National), Iacob

Alin (C. N. ”C. Negruzzi”) - premiul II; Hopu Claudia (Lic. Informatica) - premiul III.

Clasa a X-a: Romanescu Razvan (C. N. ”C. Negruzzi”) - premiul I; Cârja Oana, Iosub Daniela (am-

bii C. N. ”C. Negruzzi”), Gîrlea Codruta (Lic. ”M. Sadoveanu”, Pascani) - premiul II; Anton Constantin

(C. N. ”C. Negruzzi”), Popa Alexandru (Lic. Informatica) - premiul III.

Clasa a XI-a: Munteanu Ionut (C. N. ”E. Racovita”) - premiul I; Sucila Andrei (C. N. ”C. Negruzzi”)

- premiul I; Dumitrascu Aurel, Boureanu Ioana, Cracan Arcadie (toti C. N. ”C. Negruzzi”) - premiul III.

Clasa a XII-a: Stan Dana (Lic. ”D. Cantemir”) - premiul I; Aldea Cristina (Lic. Informatica) - pre-

miul II; Sechelea Andrei, Târniceriu Adrian (toti C. N. ”E. Racovita”), Marangoci Oana (Lic. ”M. Sadovea-

nu”, Pascani) - premiul III.

Olimpiada Balcanica de Matematica pentru JunioriEditia a VI-a, Târgu - Mures, 22 -28 iunie 2002

A. Problemele de concurs - enunturi si solutii1. Fie ABC un triunghi isoscel (AC = BC) si P un punct de arcul AB al

cercului circumscris triunghiului, ce nu contine punctul C. Fie D proiectia lui C pePB. Demonstrati ca PA+ PB = 2PD.

Grecia2. Cercurile C1 si C2 de raze diferite se intersecteaza în A si B astfel încât AB

separa centrele O1 si O2. Fie B1, B2 diametral opuse lui B în C1, respectiv C2 siM mijlocul segmentului [B1B2]. Pe C1 si C2 consideram punctele M1, respectiv M2

astfel încât \AO1M1 ≡ \AO2M2, B1 este interior unghiului \AO1M1 si B este interiorunghiului \AO2M2. Demonstrati ca \MM1B ≡ \MM2B.

Cipru3. Gasiti numerele naturale N cu proprietatile• N are exact 16 divizori 1 = d1 < d2 < . . . < d16 = N ;• divizorul d5 este egal cu (d2 + d4) d6.

Bulgaria

4. Aratati ca1

b (a+ b)+

1

c (b+ c)+

1

a (a+ c)≥ 27

2 (a+ b+ c)2, ∀a, b, c ∈ (0,∞).

Grecia

1. Solutia I. Conform teoremei lui Ptolemeu, PA · BC + PB · AC = PC · ABsi cum AC = BC, avem ca PA + PB =

PC ·ABAC

=2AE · PC

AC, unde E este

piciorul inaltimii din C, deci mijlocul lui [AB]. Atunci problema revine la a arata ca

2PD =2AE · PC

AC, adica

PD

PC=

AE

AC, i.e. cos\CPB = cos\CAB, evident adevarat.

50

Page 55: Revista (format .pdf, 1.6 MB)

Solutia II. Prelungim [PB] cu segmentul [BM ] ≡ [AP ].Atunci 4BCM ≡ 4ACP , deci [CP ] ≡ [CM ], adica 4CPMeste isoscel, cu înaltimea CD. Urmeaza ca [PD] ≡ [DM ], deci2PD = PM = PB +BM = PB + PA.

2. Este imediat ca B1, A,B2 sunt puncte colini-are. Atunci MO1 este linie mijlocie în 4B1BB2, deci

MO1 =1

2BB2 = AO2 si analogMO2 = AO1. Urmeaza

ca 4MO1A ≡ 4AO2M , de unde \MO1A ≡ \MO2A

si atunci \MO1M1 ≡ \MO2M2. De aici, 4MO1M1 ≡≡ 4MO2M2 si rezulta ca [MM1] ≡ [MM2]. Pe de alta

parte,m(\ABM2)+m(\ABM1) =³180 − a

2

´+a

2= 180

(unde a = m( \AO1M1) = m( \AO2M2)), deci punctele M1, B,M2 sunt coliniare.Atunci unghiurile din concluzie sunt la baza unui triunghi isoscel, adica ele suntcongruente.3. Cum N are exact 16 divizori, el nu poate avea mai mult de 4 divizori primi

diferiti. În plus, N este par; altfel toti divizorii ar fi impari si a doua conditie arconduce la o absurditate. Rezulta ca d2 = 2.Observam ca 2 + d4 ≥ d5 ≥ 7, deci d4 ≥ 5. Cum d4 < d5 ≤ 2 + d4, urmeaza

fie ca d5 = d4 + 1, fie ca d5 = d4 + 2. În primul caz, d6 = 2 + d4 si atunci N are

trei divizoori consecutivi, deci N... 3, de unde d3 = 3. Deoarece N

... 6, avem succesiv

d4 = 6, d5 = 7, d6 = 8, adica N... 4 si ar trebui ca d4 = 4, contradictie.

Ramâne ca d5 = 2 + d4. Analizam situatiile:

• N ... 4; cum d4 ≥ 5, avem d3 = 4, deci N... 8. Însa d6 ≥ 8, deci 8 ∈ d4, d5, d6.

Daca d4 = 8, atunci d5 = 10, deci N... 5 si ar trebui ca d4 = 5, absurd. Daca d5 = 8,

avem d4 = 6, deci N... 3, adica d3 = 3, imposibil. Daca d6 = 8, atunci d5 = 7, d4 = 5

si prin urmare N... 10. Însa d7 = (2 + 5) 8 = 56 > 10, contradictie.

Urmeaza ca N nu este multiplu de 4, deci d3 prim.

• N ... 3; rezulta ca d3 = 3, de unde N... 6 si cum d4 ≥ 6, obtinem d4 = 6, d5 = 8,

adica N... 4, absurd. În concluzie, N nu se divide cu 3 si deci d3 ≥ 5, d4 ≥ 7.

Cum N si 2 + d4 nu se divid cu 4, rezulta ca d4 este impar. Deoarece 2 + d4 sid4 nu se divid cu 3, obtinem ca d4 = 3k+ 2, în fapt d4 = 6l+ 5, l ∈ Z. Dar d5 ≤ 16,adica 7 ≤ d4 ≤ 14. Singura posibilitate este ca d4 = 11, d5 = 13. Cum 2d3 estedivizor al lui N , 2d3 ≥ d4, deci d3 ≥ 6. În plus, d3 < 11 si d3 prim, adica d3 = 7.Obtinem în final ca N = 2 · 7 · 11 · 13 = 2002.4. Solutia I. Din inegalitatea mediilor,µ

1

b (a+ b)+

1

c (b+ c)+

1

a (a+ c)

¶3≥ 27

abc (a+ b) (b+ c) (c+ a).

51

Page 56: Revista (format .pdf, 1.6 MB)

Pe de alta parte, înmultind membru cu membru inegalitatileµa+ b+ c

3

¶3≥ abc siµ

2 (a+ b+ c)

3

¶3=

µ(a+ b) + (b+ c) + (c+ a)

3

¶3≥ (a+ b) (b+ c) (c+ a), obtinem

ca1

abc (a+ b) (b+ c) (c+ a)≥ 33 · 3323 (a+ b+ c)6

, ceea ce încheie demonstratia.

Solutia II. Conform inegalitatii Cauchy - Schwarz, [(a+ b) + (b+ c) + (c+ a)]××·

1

b (a+ b)+

1

c (b+ c)+

1

a (a+ c)

¸≥µ1√b+1√c+

1√a

¶2⇔ 1

b (a+ b)+

1

c (b+ c)+

+1

a (a+ c)≥µ1√b+1√c+

1√a

¶2· 1

2 (a+ b+ c). Ramâne sa demonstram caµ

1√b+1√c+

1√a

¶21

2 (a+ b+ c)≥ 27

2 (a+ b+ c)2 ⇔ (a+ b+ c)

µ1√b+1√c+

1√a

¶2≥

≥ 27. Însa a+b+c ≥³√

a+√b+√c´2

3si³√

a+√b+√c´µ 1√

b+1√c+

1√a

¶≥ 9,

ceea ce încheie demonstratia.

B. Probleme aflate în atentia juriului1. Un elev joaca pe calculator. Calculatorul îi prezinta 2002 numere pozitive

alese la întâmplare. Regulile jocului îi permit urmatoarele operatii:— sa ia doua dintre numerele date, sa-l dubleze pe primul, sa-l adauge pe al doilea

si sa pastreze suma;— sa ia alte doua numere dintre cele ramase, sa-l dubleze pe primul, sa-l adauge

pe al doilea, sa multiplice suma cu precedenta si sa pastreze rezultatul;— sa repete procedeul pâna utilizeaza toate cele 2002 numere.Elevul câstiga jocul daca produsul obtinut în final este maximul posibil. Gasiti

strategia câstigatoare si demonstrati-o.2. Numerele naturale pozitive sunt aranjate în forma:

1 3 6 10 15 . . .2 5 9 14 . . .4 8 13 . . .7 12 . . .11 . . .. . .

Gasiti numarul coloanei si al liniei pe care este pus numarul 2002.3. Fie a, b, c numere reale pozitive astfel încât abc = 2. Demonstrati inegalitatea

a3 + b3 + c3 > a√b+ c+ b

√c+ a+ c

√a+ b.

4. Fie a, b, c numere reale pozitive. Demonstrati inegalitateaa3

b2+

b3

c2+

c3

a2≥ a2

b+

b2

c+

c2

a.

5. Daca pentru numerele reale a1, a2, a3, a4, a5, a6 au loc: a1 6= 0, a1a6 + a3a4 == 2a2a5 si a1a3 ≥ a22, demonstrati ca a4a6 ≤ a25. Când are loc egalitatea?

52

Page 57: Revista (format .pdf, 1.6 MB)

6. Se dau 2002 întregi ai, i = 1, 2, . . . 2002 astfel ca a−31 +a−32 + . . .+a−32002 = 1/2.Demonstrati ca cel putin trei dintre ei sunt egali.7. Fie triunghiul ABC, centrul sau de greutate G si mijloacele A1, B1, C1 ale

laturilor BC, CA, AB. Paralela prin A1 la BB1 intersecteaza B1C1 în F . Demon-strati ca triunghiurile ABC, FA1A sunt asemenea daca si numai daca patrulaterulAB1GC1 este inscriptibil.8. Fie triunghiul ABC, ortocentrul H, centrul I al cercului înscris, centrul O

al cercului circumscris. CI retaie cercul circumscris în L. Se stie ca AB = IL siAH = OH. Gasiti masurile unghiurilor triunghiului ABC.9. Fie triunghiul ABC de arie S si D, E, F puncte arbitrare pe BC, CA, AB.

Perpendicularele în D, E, F pe BC, CA, AB taie cercul circumscris în perechile depuncte (D1,D2), (E1, E2), (F1, F2). Demonstrati

|D1B ·D1C −D2B ·D2C|+|E1C ·E1A−E2C ·E2A|+|F1A ·F1B − F2A ·F2B| > 4S.10. Fie ABCD un patrulater convex cu AB = AD si BC = CD. Se aleg punctele

K, L, L1, K1 pe AB, BC, CD, DA încât KLL1K1 sa fie dreptunghi. Se consideraapoi dreptunghiurile: MNPQ înscris în triunghiul BLK (M ∈ KB, N ∈ BL,P,Q ∈ LK) si M1N1P1Q1 înscris în triunghiul DK1L1 (M1 ∈ DK1, N1 ∈ DL1si P1, Q1 ∈ L1K1). Fie 2S, 2S1, S2 si S3 ariile lui ABCD, KLL1K1, MNPQ,M1N1P1Q1. Gasiti cea mai mare valoare posibila a expresiei: (2S1 + S2 + S3) /2S.11. Fie A1, A2, . . . A2002 puncte în plan. Demonstrati ca oricare ar fi cercul de

raza unitate în plan si oricare ar fi dreptunghiul înscris în el, exista trei vârfuri M ,N , P ale dreptunghiului încât MA1 +MA2 + . . . +MA2002 +NA1 +NA2 + . . .++NA2002 + PA1 + PA2 + . . .+ PA2002 ≥ 6006.

La aceasta olimpiada au participat 8 tari (Albania având probleme de nerezolvatcu deplasarea). În clasamentul neoficial pe natiuni România ocupa detasat locul I cu219 puncte din 240 posibile, urmeaza Bulgaria cu 170, Moldova cu 158. Componentiiechipei României au obtinut urmatoarele rezultate:

Problema 1 2 3 4 Total MedaliiKreindler Gabriel 10 10 10 10 40 aurPachitariu Marius 10 10 9 10 39 aurUngureanu Andrei 10 10 8 9 37 aurIbram Remzi 10 10 6 10 36 aurMihailescu Ioana 10 10 6 9 35 aurMichnea Dragos 10 9 8 5 32 argint

Echipa tarii a fost condusa de prof.Dan Brânzei asistat de prof.Dinu Serba-nescu.

53

Page 58: Revista (format .pdf, 1.6 MB)

Solutiile problemelor propuse în nr. 2 / 2001

Clasele primareP.14. Scrieti toate scaderile de tipul aa− bb = cc cu rezultatul mai mare ca 50.

(Clasa I ) Gabriela Lozovanu, eleva, IasiSolutie. Rezultatul cc poate fi: 55, 66, 77, 88. Distingem cazurile:1) bb = 11, când sunt posibile scaderile: 66− 11 = 55, 77− 11 = 66, 88− 11 = 77,

99− 11 = 88;2) bb = 22, când avem: 77− 22 = 55, 88− 22 = 66, 99− 22 = 77;3) bb = 33, când avem: 88− 33 = 55, 99− 33 = 66;4) bb = 44, când avem o singura scadere: 99− 44 = 55.P.15. Mama împarte noua mere la cei trei copii ai sai. Câte mere poate primi

fiecare daca unul din ei a primit mai mult de trei mere si mai putin de sase?(Clasa I ) Maria Mursa, eleva, IasiSolutie. Prin scrierea (a, b, c) vom întelege ca un copil a primit a mere, un altul b

mere, iar ultimul c mere. Primul copil poate primi 4 mere sau 5 mere. Avem cazurile:(4, 1, 4), (4, 2, 3), (5, 1, 3), (5, 2, 2).

P.16. Suma vârstelor a trei frati este cinci ani. Doi dintre ei sunt gemeni, iarcel mai mare are ochii negri. Câti ani are fiecare dintre frati?(Clasa a II-a) Teodora-Cerasela Grigoras, eleva, IasiSolutie. Deoarece fratii gemeni sunt mai mici decât fratele cu ochi negri, singura

solutie este 1 an, 1 an, 3 ani.

P.17. Despre numerele naturale a si b stim ca: 199 < a < 203 si 314 < b < 317.Sa se afle toate valorile diferentei b− a.(Clasa a II-a) Stefania Atodiresei, eleva, IasiSolutie. Valorile lui a sunt 200, 201 sau 202, iar valorile lui b sunt 315 sau

316. Avem posibilitatile: 315 − 200 = 115, 315 − 201 = 114, 315 − 202 = 113,316− 200 = 116, 316− 201 = 115 si 316− 202 = 114, de unde rezulta ca b− a poatelua valorile 113, 114, 115, 116.

P.18. Într-o urna sunt patru bile albe, sase bile rosii si zece bile galbene. Careeste cel mai mic numar de bile pe care trebuie sa-l extragem din urna astfel încâtprintre bilele extrase sa avem cel putin câte o bila de fiecare culoare?(Clasa a III-a) Roxana Tudorache, eleva, IasiSolutie. Cel mai mic numar de bile pe care trebuie sa-l extragem este 17. Acest

numar se poate afla astfel: observam ca avem 10 bile albe sau rosii, 14 bile albe saugalbene si 16 bile rosii sau galbene. Pentru a nu avea numai bile de doua culori,trebuie sa extragem un numar de bile mai mare decât fiecare din numerele 10, 14 si16. Cel mai mic numar cu aceasta proprietate este 17.

P.19. Ce ora arata ceasul meu acum, daca de la ora 12 a trecut jumatate dintimpul care a mai ramas pâna la sfârsitul zilei?(Clasa a III-a) Înv. Rodica Agrici, IasiSolutie. Timpul care a trecut de la ora 12 pâna în momentul citirii ceasului este

54

Page 59: Revista (format .pdf, 1.6 MB)

o treime din perioada dintre ora 12 si sfârsitul zilei, adica (24− 12) : 3 = 12 : 3 = 4ore. Ceasul arata ora 16.

P.20. Sa se determine numarul natural A = xyzt stiind ca sunt îndepliniteconditiile: a) suma cifrelor este 20, b) fiecare cifra este cu doi mai mare decât ceadin fata ei.(Clasa a III-a) Înv. Elena Marchitan, IasiSolutie. Deducem ca y = x + 2, z = x + 4, t = x + 6. Daca înlaturam din 20

suma (2 + 4 + 6) obtinem de 4 ori cifra x. Deci 4 ·x = 20− (2 + 4 + 6) = 20−12 = 8si x = 8 : 4 = 2. Numarul cautat este 2468.

P.21. În curtea bunicii sunt 81 de pasari: rate, gâste si gaini. Stiind ca ele potfi grupate astfel încât la o gâsca sa corespunda trei gaini, iar la doua gâste, o rata,aflati câte pasari de fiecare fel are bunica în curte.(Clasa a IV-a) Teodora-Cerasela Grigoras, eleva, IasiSolutie. Deoarece la doua gâste corespunde o rata, iar la fiecare gâsca trei gaini,

putem sa facem grupuri de câte o rata, 2 gâste si 6 gaini, ceea ce înseamna 9 pasaride curte. În curte sunt 81 : 9 = 9 (grupuri). Vom avea solutia 9 · 1 = 9 (rate),9 · 2 = 18 (gâste) si 9 · 6 = 54 (gaini).P.22. Lungimea unui dreptunghi reprezinta

3

8din perimetrul sau. Daca adunam

1

2din latime cu lungimea, obtinem 105 cm. Aflati dimensiunile si perimetrul

dreptunghiului.(Clasa a IV-a) Înv. Maria Racu, Iasi

Solutie. Suma lungimilor reprezinta6

8din perimetru, iar suma latimilor repre-

zinta2

8din perimetru. Înseamna ca o lungime reprezinta

3

8din perimetru si o latime

1

8din perimetru. Cum jumatate din latime reprezinta

1

16din perimetru, iar fractia

3

8este egala cu

6

16, deducem ca

7

16din perimetru reprezinta 105 cm, apoi

1

16din

perimetru reprezinta 105 : 7 = 15 (cm). Perimetrul este 16·15 = 240 (cm). Lungimeaeste 240 : 8 · 3 = 30 · 3 = 90 (cm), iar latimea este 240 : 8 = 30 (cm).P.23. Daca împartim la patru fiecare termen al unui sir de numere, obtinem mai

multe numere consecutive impare, cu suma ultimelor doua, adica 96, mai mare cu16 decât dublul sumei primelor doua numere. Sa se gaseasca al cincilea termen alsirului.(Clasa a IV-a) Înv. Mihai Agrici, IasiSolutie. Dublul sumei primelor doua numere impare consecutive este 96− 16 =

= 80. Suma primelor doua numere impare consecutive este 80 : 2 = 40. Primulnumar din sirul numerelor impare consecutive este (40− 2) : 2 = 38 : 2 = 19. Alcincilea numar din sirul numerelor impare consecutive este 19 + 2 + 2 + 2 + 2 = 27.Numarul cautat este 27 · 4 = 108.

Clasa a V-aV.21. Aratati ca numarul N = 2n+1 · 3n + 2n · 3n+1 + 6 · 6n este divizibil cu 11

55

Page 60: Revista (format .pdf, 1.6 MB)

pentru orice n ∈ N.Dorina Carapanu, Iasi

Solutie. Calculând, avem N = 2 ·2n ·3n+2n ·3n ·3+6 ·6n = 2 ·6n+3 ·6n+6 ·6n == 6n (2 + 3 + 6) = 6n · 11, deci N se divide cu 11 oricare ar fi n ∈ N.V.22. Determinati multimea A =

©n;n ∈ N, n2 + n+ 2 = 1 + 2 + 3 + . . .+ 2001

ª.

Maria Zalinescu, IasiSolutie. Deoarece n2+n+2 = n (n+ 1)+ 2 este numar par pentru orice n ∈ N,

iar 1 + 2 + . . .+ 2001 =2001 · 2002

2= 2001 · 1001 este impar, urmeaza ca A = ∅.

V.23. Sa se afle ultima cifra a numarului S = 1 + 2 + 22 + . . .+ 22001.Cristiana Artenie, eleva, Iasi

Solutia I. Se observa ca U(24k+1+24k+2+24k+3+24k+4) = U (2 + 4 + 8 + 6) == 0, oricare ar fi k ∈ N. Deoarece S = 1+¡21 + 22 + 23 + 24¢+¡25 + 26 + 27 + 28¢++ · · ·+¡21997 + . . .+ 22000

¢+22001, rezulta ca U (S) = U(1+0+0+ . . .+0+2) = 3.

Solutia II. Avem 2S = 2+22+ . . .+22002 = 22002−1+S si atunci S = 22002−1iar U (S) = 3.

V.24. Sa se determine numerele abc pentru care a+ b+ c = abc.Paraschiva Bîrsan, Iasi

Solutie. Mai general, vom determina numerele naturale a, b, c cu proprietateaa+ b+ c = abc (1).Daca cel putin unul din cele trei numere este nul, atunci abc = 0, deci a+b+c = 0

si atunci a = b = c = 0.Fie a, b, c ∈ N∗ si sa presupunem ca a ≤ b ≤ c. Impartind prin c, (1) se scrie sub

formaa

c+b

c+1 = ab si tinând seama de presupunerea facuta rezulta ca 1 < ab ≤ 3,

adica ab ∈ 2, 3. Daca ab = 2, atunci a = 1 si b = 2, iar din (1) avem ca 3+ c = 2c,deci c = 3. Daca ab = 3, atunci a = 1 si b = 3, iar din (1) obtinem 4 + c = 3c,adica c = 2 < b, care nu convine. Ramâne ca (a, b, c) = (1, 2, 3) în mod necesar si severifica imediat ca 1 + 2 + 3 = 1 · 2 · 3. Prin urmare, tripletele cautate sunt (1, 2, 3),(1, 3, 2), (2, 1, 3), (2, 3, 1), (3, 1, 2), (3, 2, 1).

V.25. Fie numarul a = 20012001. Notam cu b suma cifrelor numarului a, apoicu c suma cifrelor numarului b si continuam în acest mod pâna ajungem la un numarformat cu o singura cifra. Care este acest din urma numar?

***

Solutie. Deoarece 2001... 3 numarul a se divide cu 9. Aceasta este echivalent cu

faptul ca suma cifrelor lui a se divide cu 9, altfel spus b se divide cu 9. Urmeaza

c... 9 si, continuând rationamentul, obtinem ca numarul cu o singura cifra la care seajunge se divide cu 9. Cum acesta este evident nenul, el este egal cu 9.

Clasa a VI-aVI.21. Fie date numerele distincte a1, a2, a3, . . . , a2n+1 ∈ Z, n > 2, astfel încât

|a1|+|a2|+|a3|+. . .+|a2n+1| = n (n+ 1). Sa se calculeze |a1 + a2 + a3 + . . .+ a2n+1|.Cristiana Artenie, eleva, Iasi

Solutie. Considerând suma modulelor celor mai mici, în modul, 2n + 1 nu-mere întregi distincte, obtinem 0 + |1| + |−1| + |2| + |−2| + . . . + |n| + |−n| =

56

Page 61: Revista (format .pdf, 1.6 MB)

= 2n (n+ 1)

2= n (n+ 1). Rezulta atunci ca a1, a2, . . . , a2n+1 = 0,±1, . . . ,±n

si deci |a1 + a2 + . . .+ a2n+1| = 0. Rezultatul are loc pentru n ≥ 1.VI.22. Sa se afle numerele a, b ∈ N, stiind ca 5a− 3b = 72 si (a, b) = 24.

Mihai Gârtan, IasiSolutie (data de Boieriu Bianca, eleva, Brasov). Fie a = 24m, b = 24n, cu

m,n ∈ N, (m,n) = 1; atunci ecuatia devine 5m − 3n = 3 sau înca 5m = 3 (n+ 1).

Deducem ca m... 3, deci m = 3t si n = 5t − 1. Cum n ∈ N, gasim t ∈ N∗. Sa

determinam t astfel încât (m,n) = 1. Fie d | 3t si d | 5t − 1; rezulta ca d | 5 · 3t−−3 (5t− 1), deci d | 3, prin urmare m si n sunt prime între ele sau se divid la 3. Cum3 | 3t, din 3 | 5t− 1 gasim t = 3k + 2, k ∈ N. Astfel, (m,n) = 1 daca si numai dacat = 3k sau t = 3k + 1. Gasim solutiile a = 24 · 9k, b = 24 (15k − 1), k ∈ N∗ saua = 24 (9k + 3), b = 24 (15k + 4), k ∈ N.VI.23. a) Aflati a, b ∈ N∗ cu proprietatiile: ab = 24 si [a, b] + (a, b) = 14.b) Fie a, b ∈ N∗, a ≤ b, cu proprietatea ca [a, b] + (a, b) = a + b. Sa se arate ca

a | b.Cristiana Constanda, eleva, Iasi

Solutie. a) Deoarece [a, b] · (a, b) = a · b = 24 si [a, b] + (a, b) = 14, rezulta ca[a, b] = 12 si (a, b) = 2. Atunci, a = 2a0, b = 2b0, cu (a0, b0) = 1 si a0 · b0 = 6, adica(a0, b0) ∈ (1, 6) , (2, 3) , (3, 2) , (6, 1), deci (a, b) ∈ (2, 12) , (4, 6) , (6, 4) , (12, 2) siaceste perechi verifica ipotezele problemei.

b) Deoarece [a, b] + (a, b) = a + b si cum are loc si [a, b] · (a, b) = ab, rezulta ca[a, b] = a si (a, b) = b sau [a, b] = b si (a, b) = a. Conditia a ≤ b impune [a, b] = b si(a, b) = a. Din acestea, deducem usor ca a | b.VI. 24. a) Fie suma S =

1

39+1

40+ . . .+

1

50. Sa se arate ca

6

25< S <

4

13;

b) Daca S =a

b, cu a, b ∈ N∗, (a, b) = 1, sa se arate ca b este numar par, iar a

se divide cu 89.***

Solutie. a) Observam ca suma are 12 termeni, cel mai mic termen este1

50, iar

cel mai mare este1

39. Rezulta ca 12 · 1

50< S < 12 · 1

39, deci

6

25< S <

4

13.

b) Observam caa

b=

µ1

39+1

50

¶+

µ1

40+1

49

¶+ . . . +

µ1

44+1

45

¶=

89

39 · 50+

+89

40 · 49 + . . .+89

44 · 45 =89k

39 · 40 · . . . · 50 . Aceasta din urma fractie nu se simplifica

prin 89, deoarece acest numar este prim, si atunci a... 89.

Pe de alta parte,a

b=

µ1

39+ . . .+

1

47+1

49+1

50

¶+1

48=

m

8n+1

48cu n impar

(din conditia (a, b) = 1), decia

b=6m+ n

48n, numaratorul 6m+n fiind impar. Rezulta

ca fractia6m+ n

48nnu se simplifica prin 2 si atunci b este par (chiar multiplu de 8).

VI.25. Fie ABC un triunghi isoscel (AB = AC). Notam cu E proiectia lui Bpe AC si cu F proiectia lui E pe BC. Fie M mijlocul lui BE. Sa se arate ca

57

Page 62: Revista (format .pdf, 1.6 MB)

FM ⊥ AB.Constantin Cocea si Dumitru Neagu, Iasi

Solutie. Notam cu D proiectia lui A pe BC si fieP = FM∩AB. Atunci AD este bisectoarea unghiului bA:m(\DAB) = m(\DAC) = α. Cum FM este mediana cores-punzatoare ipotenuzei în triunghiul dreptunghic FEB,rezulta ca FM = MB, de unde \MBF = \MFB. Însam(\MBF ) = 90−m( bC) = m(\DAC) = α, decim(\MFB) =

α. În concluzie, m(\BPF ) = 180−hm( bB) +m(\MFB)

i=

180 −hm( bB) + α

i= 180 − 90 = 90, adica FP ⊥ AB.

Clasa a VII-aVII.21. Fie E (x) =

µ1

x+ 2− 3

x− 2¶· x (x+ 4) + 4

x2 − 16 , x ∈ R\ ±2,±4. DacaA = x ∈ N; x par si E (x) ∈ Z si B = x ∈ N; x impar si E (x) ∈ N, sa se arateca multimea

½x

y| x ∈ A si y ∈ B

¾are un singur element.

Dumitru-Dominic Bucescu, Iasi

Solutie. Calculând, obtinem ca E (x) =−2 (x+ 2)

(x− 2) (x− 4) . Fie x = 2k ∈ A; cum

E (x) =− (k + 1)

(k − 1) (k − 2) ∈ Z si k ∈ N, avem în mod necesar (k − 1) (k − 2) ≤ k + 1,

de unde k2−4k+1 ≤ 0, sau (k − 2)2 ≤ 3, i.e. |k − 2| ≤ √3, deci k ∈ 0, 1, 2, 3, ceeace implica x ∈ 0, 2, 4, 6. Însa x ∈ R\ ±2,±4, iar dintre numerele E (0) si E (6)este întreg numai al doilea; rezulta ca A = 6. Fie acum x = 2l + 1 ∈ B; deoarece

E (x) =−2 (2l + 3)

(2l − 1) (2l − 3) ∈ N, urmeaza ca½2l − 1 | 2l + 32l − 3 | 2l + 3 ⇔

½2l − 1 | 42l − 3 | 6 ⇔½

l ∈ 0, 1l ∈ 0, 1, 2, 3 . Obtinem ca posibile valori x ∈ 1, 3 si, prin verificare, B = 3.

Concluzia este acum imediata.

VII. 22. Sa se arate ca numarul A = 12+ 123 +125 + ...+122001 se divide prin1716.

Gabriel Popa, IasiSolutia I (data de autor). Deoarece 1716 = 11 · 12 · 13 cu factorii relativ primi

si este evident ca A... 12, ramâne sa aratam ca A

... 11 si A... 13. Observam ca A este o

suma cu 1001 termeni, iar 1001 = 7 · 11 · 13. Atunci:A− 1001 = (12− 1) + ¡123 − 1¢+ ¡125 − 1¢+ . . .+

¡122001 − 1¢ =M11

A+ 1001 = (12 + 1) +¡123 + 1

¢+¡125 + 1

¢+ . . .+

¡122001 + 1

¢=M13,

unde am folosit faptul ca 122k+1 − 1 = (12− 1) ¡122k + . . .+ 12 + 1¢= M11, iar

122k+1 + 1 = (12 + 1)¡122k − . . .− 12 + 1¢ =M13.

Solutia II (data de eleva Stefana Branisteanu). Observam ca 1716 = 12·143si 122 = 143 + 1. Avem:

A = 12¡1 + 122 + 124 + . . .+ 122000

¢= 12

h1 + (143 + 1) + (143 + 1)

2+

58

Page 63: Revista (format .pdf, 1.6 MB)

+ . . .+ (143 + 1)1000i= 12 [1 + (143 + 1) + (M143 + 1) + . . .+ (M143 + 1)] =

= 12 (1001 +M143) = 12 (7 · 143 +M143) = 12 ·M143 =M1716.

VII.23. Intr-un reper cartezian xOy se considera punctele A (−a, a), B (b, a),C (0, c) si D (0, a) cu a > 0, b > 0, c > 0. Sa se compare CA + CB + CO cuDA+DB +DC +DO.

Maria Zalinescu, IasiSolutie. Daca a > c, atunci DC + CO = DO. Deoarece DA + DC > CA si

DB + DC > CB, atunci DA + DB + 2DC > CA + CB, deci DA + DB + DC++(DC + CO) > CA+CB+CO, adicaDA+DB+DC+DO > CA+CB+CO. Dacaa < c, procedând analog, obtinem ca DA+DB+DC+DO < CA+CB+CO. Dacaa = c, cele doua sume sunt evident egale. Problema poate fi rezolvata si utilizândformula distantei între doua puncte.VII.24. Sa se arate ca pentru orice n ∈ N∗ are loc inegalitatea

(n+ 1)n−1 ≤ (n!)2 ≤ (n+ 1)n−1

µn+ 2

3

¶n(s-a notat n! = 1 · 2 · 3 · . . . · n).

Ünige Bencze, BrasovSolutie. Autoarea foloseste inegalitatea H ≤ G ≤ A între mediile armonica, geo-

metrica si aritmetica, luând ca numere ak = k (k + 1), k = 1, n, si obtine inegalitateadorita. Se pot obtine limitari mai ”strânse” pentru (n!)2.VII.25. Fie ABCD un dreptunghi, iar M ∈ (CD) un punct oarecare. Perpen-

diculara din D pe BM intersecteaza perpendiculara din C pe AM în punctul P . Sase arate ca PM ⊥ CD.

Constantin Cocea, IasiSolutia I. Fie P 0 punctul în care perpendiculara din D pe MB taie perpen-

diculara în M pe CD, iar P 00 punctul în care perpendiculara din C pe MA taieperpendiculara în M pe CD. Problema ar fi rezolvata daca am arata ca P 0 = P 00.Însa 4P 0MD ∼ 4MCB, fiind ambele dreptunghice, iar \P 0DM ≡ \MBC ca

unghiuri cu laturile respectiv perpendiculare; rezulta caP 0MMD

=MC

CB. Analog, din

4P 00MC ∼ 4MDA obtinemP 00MMC

=MD

DA. De aici, P 0M · CB = P 00M · DA si

fiindca CB = DA, avem P 0M = P 00M ; cum P 0, P 00 se afla în acelasi semiplan fatade CD, urmeaza ca P 0 = P 00.

Solutia II (data de Edgar Kern, Brasov). FieE = DP ∩BM si F = = CP ∩AM . Din asemanarileimediate 4DEM ∼ 4BCM si 4CFM ∼ 4ADM rezulta

caEM

CM=

DM

BM, respectiv

FM

DM=

CM

AM; urmeaza de aici ca

EM

AM=

FM

BMsi atunci 4EFM ∼ 4AMB. În consecinta

\MEF ≡\MAB si cum \MAB ≡\FMC (corespondente),

avem \MEF ≡\FMC. Pe de alta parte, patrulaterulMFPE este inscriptibil, de unde\PEF ≡\PMF . Obtinem astfel ca m(\PMF ) +m(\FMC) = m(\PEF ) +m(\MEF ) =90, deci PM ⊥ CD.

59

Page 64: Revista (format .pdf, 1.6 MB)

Clasa a VIII-aVIII.21. Daca f : 0, 1, 2, . . . , 10 → 0, 1, 2, . . . , 10 este data prin f (x) =

= ax+ b, a ∈ R∗, b ∈ R, atunci f (5) = 5.Gheorghe Iurea, Iasi

Solutie. Evident ca x1 6= x2 daca si numai daca f (x1) 6= f (x2) si deci f (0) ,f (1) , . . . , f (10) = 0, 1, . . . , 10. Atunci f (0)+f (1)+. . .+f (10) = 1+2+. . .+10,adica a (1 + 2 + . . .+ 10) + 11b = 55, de unde rezulta ca 5a+ b = 5, i.e. f (5) = 5.

VIII.22. Fie a > 0, b > 0. Aratati ca1

a4+

1

4a3b+

1

6a2b2+

1

4ab3+1

b4≥ 128

3 (a+ b)4

Lucian Tutescu, CraiovaSolutie. Avem ca¡

a4 + 4a3b+ 6a2b2 + 4ab3 + b4¢µ 1

a4+

1

4a3b+

1

6a2b2+

1

4ab3+1

b4

¶=

= 5 +

µ1

4+ 4 +

2

3+3

2

¶µa

b+

b

a

¶+

µ1

6+ 1 + 6

¶µa2

b2+

b2

a2

¶+

+

µ1

4+ 4

¶µa3

b3+

b3

a3

¶+

µa4

b4+

b4

a4

¶≥ 128

3,

deoarecex

y+

y

x≥ 2, ∀x, y > 0. Concluzia se obtine acum tinând seama de faptul ca

(a+ b)4= a4 + 4a3b+ 6a2b2 + 4ab3 + b4. Egalitatea este atinsa pentru a = b.

VIII.23. Sa se rezolve în multimea numerelor reale ecuatiax2

|x+ 1| =x− 4x+ 1

+ ||x|− 1| .Mihai Craciun, Pascani

Solutie. Explicitam modulele pe fiecare dintre intervalele (−∞,−1), (−1, 0),[0, 1), [1,∞) si rezolvam ecuatiile obtinute. În final, gasim doua solutii: −5 si 5.VIII.24. Aratati ca α ∈ (0, 1), stiind ca numarul real α este o solutie a ecuatiei

a1x2k1+1 + a2x

2k2+1 + . . .+ anx2kn+1 = b,

unde a1, a2, . . . , an, b ∈ (0,∞); k1, k2, . . . , kn ∈ N si a1+a2+. . .+an > b (generalizarea problemei VIII.17 din nr.1/2001).

Mihaela Negrea, BrasovSolutie. Daca α ≥ 1, atunci a1α2k1+1+ . . .+anα

2kn+1−b ≥ a1+ . . .+an−b > 0,deci α nu poate fi solutie a ecuatiei date. Daca α ≤ 0, atunci a1α2k1+1 + . . .++anα

2kn+1 − b ≤ −b < 0 si din nou obtinem ca α nu poate fi solutie. Rezulta caα ∈ (0, 1).VIII.25. Fie cubul ABCDA0B0C 0D0 si M ∈ [A0C] cu proprietatea ca

MA0

MC=3

2. Sa se determine un punct P ∈ [AA0] astfel încât suma PM + PC0

sa fie minima.Petru Asaftei, Iasi

Solutie. Notam cuM 0 simetricul luiM fata de AA0 si fie P =M 0C 0∩AA0. Înacest caz, avem PM+PC 0 = PM 0+PC0 =M 0C 0. Daca P1 ∈ [AA0], P1 6= P , atunciM 0C 0 < M 0P1 + P1C

0 =MP1 + P1C0, deci P este punctul cautat. Sa determinam

60

Page 65: Revista (format .pdf, 1.6 MB)

PA0; pentru aceasta, fie O = AA0 ∩MM 0

si M 00 = CC0 ∩ MM 0. În 4A0AC avemMOkAC si atunci, conform teoremei funda-

mentale a asemanarii,OM

AC=

A0OA0A

, deci

OM =3AC

5.

Analog, în 4M 0M 00C0 avem OPkC 0M 00,

deciOP

M 00C0=

M 0OM 0M 00 , adica OP ·M 0M 00 =

=M 00C0·M 0O, de unde OP ·µ3AC

5+AC

¶=3AA0

5· 3AC5, prin urmare OP =

9

40AA0.

În final, PA0 = OA0 −OP =3AA0

5− 9AA

0

40=3AA0

8.

Clasa a IX-aIX.21. Sa se stabileasca care dintre afirmatiile urmatoare:(i) ||sinx|− |sin y|| ≤ |sin (x− y)| , ∀x, y ∈ R;(ii) ||cosx|− |cos y|| ≤ |cos (x− y)| , ∀x, y ∈ R;(iii) |sin (x− y)| ≤ |sinx− sin y| , ∀x, y ∈ R;(iv) |cos (x− y)| ≤ |cosx− cos y| , ∀x, y ∈ R

este adevarata si care este falsa.Gheorghe Costovici, Iasi

Solutie. Deoarece sinx = sin (x− y + y) = sin (x− y) cos y + cos (x− y) sin y,urmeaza ca |sinx| ≤ |sin (x− y)| + |sin y|. Analog, |sin y| ≤ |sin (y − x)| + |sinx| sidin aceste doua relatii obtinem ca − |sin (x− y)| ≤ |sinx| − |sin y| ≤ |sin (x− y)|,ceea ce arata ca (i) este adevarata.

Afirmatia (ii) este falsa: pentru x =π

2si y = 0, obtinem contradictia 1 ≤ 0.

De asemenea, (iii) este falsa (putem lua x =π

2, y =

π

4si obtinem

√2

2≤ 1 −

√2

2,

absurd), iar (iv) este tot falsa (considerând x = y, am avea 1 ≤ 0).IX.22. Sa se arate camacar una dintre ecuatiile: ax2+2

√3bcx+bc (a+ b+ c) = 0,

bx2+2√3cax+ca (a+ b+ c) = 0, cx2+2

√3abx+ab (a+ b+ c) = 0, unde a, b, c ∈ R∗,

admite radacini reale.Mihail Bencze, Brasov

Solutie. Daca, prin absurd, nici una dintre ecuatii nu ar avea solutiile reale,atunci toate ar avea discriminantul negativ, deci ∆1+∆2+∆3 < 0. Însa ∆1+∆2+

+∆3 = 12£a2b2 + b2c2 + c2a2 − abc (a+ b+ c)

¤= 6

h(ab− ac)2 + (ab− bc)2+

+(bc− ac)2 ≥ 0

i, de unde concluzia.

IX.23. Fie a ∈ R si o functie f : R→ R ce satisface relatiaf (f (x+ y) + y) = x+ f (ay) , ∀x, y ∈ R.

1 Sa se arate ca f este injectiva.2 Sa se determine functiile f ce satisfac conditia de mai sus.

Dan Popescu, SuceavaSolutie. 1 Luând y = 0 în relatia din ipoteza, obtinem ca f (f (x)) = x+ f (0),

61

Page 66: Revista (format .pdf, 1.6 MB)

∀x ∈ R. Daca x1, x2 ∈ R sunt astfel încât f (x1) = f (x2), atunci f (f (x1)) == f (f (x2)), deci x1 + f (0) = x2 + f (0), adica x1 = x2; rezulta ca f este injectiva.2 Deoarece f (f (0)) = 0 + f (0) = f (0) si f este injectiva, urmeaza ca f (0) = 0

si atunci f (f (x)) = x, ∀x ∈ R. Pentru x = 0 în relatia din ipoteza obtinemf (f (y) + y) = f (ay), ∀y ∈ R, deci f (y) + y = ay, ∀y ∈ R, i.e. f (y) = (a− 1) y,∀y ∈ R. Din aceasta si din faptul ca ∀y ∈ R, f (f (y)) = y deducem ca (a− 1)2 = 1,adica a ∈ 0, 2. Prin urmare, f (x) = −x, daca a = 0; f (x) = x, daca a = 2 siecuatia functionala nu are solutii în rest.

IX.24. a) Fie x, y, z, t ∈µ−12,∞¶cu x + y + z + t = 1. Aratati ca x3 + y3+

+z3 + t3 ≥ 1

16.

b) Fie x1, x2, . . . , xn ∈µ− 2n,∞¶cu x1+x2+ . . .+xn = 1. Aratati ca x31+x32+

+ . . .+ x3n ≥1

n2.

Mihai Bogdan Ion, elev, CraiovaSolutie. Vom demonstra direct punctul b). Avem:

x1 > − 2n⇔ n

2x1 + 1 > 0⇔

³n2x1 + 1

´µx1 − 1

n

¶2≥ 0⇔

⇔ n

2x31 −

3

2nx1 +

1

n2≥ 0⇔ x31 ≥

3nx1 − 2n3

,

cu egalitate pentru x1 =1

n. Scriind inegalitatile analoage si adunând cele n relatii

membru cu membru, obtinem concluzia; egalitatea are loc pentru x1 = x2 = . . . =

= xn =1

n.

IX.25. Fie ABC un triunghi oarecare si punctele A1, B1, C1 pe laturile (BC),(CA) si respectiv (AB). Dreptele AA1, BB1 si CC1 intersecteaza cercul circumscristriunghiului ABC în punctele A2, B2 si respectiv C2. Sa se arate ca

AA1A1A2

+BB1B1B2

+CC1C1C2

≥ 23

p2

Rr.

Neculai Roman, Mircesti (Iasi)Solutia autorului. Utilizând relatia lui Stewart si puterea punctului, avem:AA1A1A2

=AA21 ·BC

AA1 ·A1A2 ·BC =AB2 ·A1C +AC2 ·A1B −A1B ·A1C ·BC

A1B ·A1C ·BC =

=c2

ax+

b2

a (a− x)−1, unde x = A1B. Considerând în inegalitatea Cauchy-Buniakovski-

Schwartz a1 =c√x, a2 =

b√a− x

, b1 =√x, b2 =

√a− x, obtinem:

AA1A1A2

=1

a

"µc√x

¶2+

µb√a− x

¶2#− 1 ≥ (b+ c)2

a2− 1 = 4p (p− a)

a2,

cu egalitate dacaa1b1=

a2b2, i.e. x =

ac

b+ c, adica A1 este piciorul bisectoarei din A.

62

Page 67: Revista (format .pdf, 1.6 MB)

Fara a restrânge generalitatea, presupunem a ≥ b ≥ c;

atunci p − a ≤ p − b ≤ p − c si1

a2≤ 1

b2≤ 1

c2. Aplicând

inegalitatea lui Cebâsev, obtinem:

AA1A1A2

+BB1B1B2

+CC1

C1C2≥ 4p

·p− a

a2+

p− b

b2+

p− c

c2

¸≥

≥ 4p3

hX(p− a)

i ·X 1

a2

¸=4p2

3

µ1

a2+1

b2+1

c2

¶≥

≥ 4p2

3

µ1

ab+1

bc+1

ac

¶=4p2

3· 2pabc

=8p3

3 · 4RS =2p2

3Rr,

cu egalitate pentru a = b = c.

Inegalitatea demonstrata este mai tare decât inegalitatea cunoscutaAA1A1A2

+

+BB1B1B2

+CC1C1C2

≥ 9, deoarece 2p2

3Rr=8p3

3abc≥ 8p3

3

µa+ b+ c

3

¶3 = 8p3 · 98p3 = 9.

Clasa a X-aX.21. Fie (an)n≥1 o progresie aritmetica cu a1 ∈ N∗ si ratia r = 19. Sa se

cerceteze daca, pornind de la sirul (bn)n≥1 (unde bn este suma cifrelor termenuluian), putem permuta termenii acestuia încât sa se obtina o progresie geometrica.

Lucian-Georges Ladunca, IasiSolutia I (data de Gheorghe Iurea, Iasi). Presupunem ca (bn)n≥1 este o

progresie geometrica. Deoarece dintre numerele a1, a1 +19, a1+38 unul singur estedivizibil cu 3, acestea conduc la trei numere din sirul (bn): bm, bs, bt, m < s < t,dintre care unul singur este divizibil cu 3.Cum (bn) este progresie geometrica avem: bt−ms = bs−mt ·bt−sm , egalitate imposibila,

deoarece un membru este divizibil cu 3, iar celalalt nu.Solutia II (data de Ciprian Baghiu, Iasi). Presupunem ca (bn) este o

progresie geometrica de ratie q. Evident q ∈ Q∗+, si fie q =a

b, a, b ∈ N∗, (a, b) = 1.

Deoarece b1, b1q, . . . , b1qn, . . . sunt numere naturale nenule si (a, b) = 1, rezulta cab1bn∈ N, pentru orice n ∈ N∗ si atunci b = 1 iar q = a ∈ N∗. Fie q 6= 1, deci q ≥ 2; cum

2n−1 ≥ n2 pentru n ≥ 7 deducem ca, pentru n suficient de mare, b1qn−1 ≥ b12n−1 ≥

≥ b1nr > a1 + (n− 1) r, deci bn > an. Contradictie, întrucât pentru orice numar

natural an, an depaseste suma cifrelor sale, adica an ≥ bn. Cazul q = 1 se analizeazaca în solutia III.Solutia III (în maniera autorului). Vom rezolva problema în cazul general în

care r ∈ N∗. Fie a1 = x1x2 . . . xp si r = y1y2 . . . yk. Pentru n suficient de mare (n >p), a10n+1 = a1+10

nr = y1y2 . . . yk0 . . . 0x1x2 . . . xp, deci b10n+1 = y1+. . .+yk+x1++ . . .+xp. Prin urmare, daca (bn)n≥1 este o progresie geometrica, atunci q = 1. Cumb1 = x1 + x2 + . . . + xp si b10n+1 6= b1 pentru n suficient de mare, rezulta ca q 6= 1.În consecinta, nu putem forma o progresie geometrica pornind de la sirul (bn)n≥1.

X.22. Sa se demonstreze ca orice sir (xn)n≥1 de numere reale care satisface

63

Page 68: Revista (format .pdf, 1.6 MB)

relatia

¯xm+1xn − xm+n

xmxn

¯≤ 1

m+ n, ∀m,n ∈ N∗, este o progresie geometrica.

Dan Popescu, SuceavaSolutie. Pentru m,n ∈ N∗ arbitrari, avem:¯

xm+1xm

− xn+1xn

¯=

¯xm+1xn − xm+n + xm+n + xn+1xm

xmxn

¯≤

≤¯xm+1xn − xm+n

xmxn

¯+

¯xn+1xm − xm+n

xmxn

¯≤ 2

m+ n.

Definim yn =xn+1xn

si sa dovedim ca yn = q, ∀n ∈ N∗; daca ar exista m,n ∈ N∗cu ym 6= yn, atunci am avea 0 < a = |ym − yn| = |ym − yp + yp − yn| ≤ |ym − yp|++ |yp − yn| ≤ 2

m+ p+

2

n+ p<4

p, adica p <

4

a, ∀p ∈ N∗, deci N∗ ar fi o multime

majorata, absurd.

Nota. Înlocuind xn = x1qn−1, ∀n ∈ N∗ în ipoteza, obtinem

¯q

µ1− 1

x1

¶¯≤

≤ 1

m+ n, ∀m,n ∈ N∗, de unde x1 = 1. Reciproc, daca (xn)n≥1 este dat prin

xn = qn−1, ∀n ∈ N∗ (q 6= 0), conditia din enunt este verificata. Prin urmare,proprietatea din ipoteza caracterizeaza progrsiile geometrice cu x1 = 1.

X.23. Sa se calculeze suma Sn =nX

k=0

(−1)k(k + 1) (k + 2)

Ckn.

Alina Craciun, Pascani

Solutie. Deoarece Ck+2n+2 =

(n+ 2) (n+ 1)

(k + 2) (k + 1)Ckn, avem:

0 = (1− 1)n+2 =n+2Xj=0

C jn+2 (−1)j = C0n+2 + C1n+2 (−1) +

n+2Xj=2

Cjn+2 (−1)j =

= 1− (n+ 2) +nX

k=0

Ck+2n+2 (−1)k+2 = − (n+ 1) +

nXk=0

(n+ 2) (n+ 1)

(k + 2) (k + 1)Ckn (−1)k ,

deci n+ 1 = (n+ 2) (n+ 1)Sn, de unde rezulta ca Sn =1

n+ 2.

X.24. Fie ecuatiaax

ln b+

bx

ln a= 0, 0 < a < 1, b > 1. Sa se arate ca:

(i) ecuatia are o singura solutie pe R,(ii) solutia este negativa daca si numai daca ab > 1.

Petru Asaftei, Iasi

Solutie. (i) Ecuatia data se scrie echivalent³ab

´x= − ln b

ln a. Cum a ∈ (0, 1) si

b > 1, atunci − ln bln a

> 0, deci ecuatia are o singura solutie pe R.

(ii) Deoarece 0 <a

b< 1, pentru unica solutie x, avem:

x < 0⇔ − ln bln a

> 1⇔ ln b+ lna > 0⇔ ln ab > 0⇔ ab > 1.

X.25. Fie ABCD un tetraedru în care AB = CD si BC = AD. Notam cu

64

Page 69: Revista (format .pdf, 1.6 MB)

A1 punctul de pe sfera circumscrisa tetraedrului, diametral opus vârfului A. Sa sedemonstreze ca A1C ⊥ BD.

Constantin Cocea, IasiSolutie (data de Mihaela Negrea, Brasov). Pentru fixarea ideilor, sa pre-

supunem ca BC ≤ CD; atunci m(\BDC) < 90. Din triunghiurile dreptunghiceADA1 siABA1 avemA1D =

√4R2 −AD2 =

√4R2 −BC2 siA1B =

√4R2 −AB2 =

=√4R2 − CD2; rezulta ca A1B ≤ A1D, deci m( \BDA1) < 90. Daca notam cu

T1, T2 proiectiile pe BD ale punctelor C, respectiv A1, urmeaza ca T1, T2 ∈ (DB.Din teorema lui Pitagora generalizata obtinem:

DT2 =BD2 +A1D

2 −A1B2

2BD=

BD2 + 4R2 −BC2 − ¡4R2 − CD2¢

2BD=

=BD2 + CD2 −BC2

2BD= DT1,

deci T1 = T2 = T si A1T ⊥ BD. Atunci (A1TC) ⊥ BD, de unde A1C ⊥ BD.

Clasa a XI-aXI.21. Sa se rezolve sistemul de ecuatii:

2x1+x2−x3 = 0, 2x2+x3−x4 = 0, . . . , 2x2000+x2001−x1 = 0, 2x2001+x1−x2 = 0.Gabriel Popa, Iasi

Solutia I. Notam y1 = 2x1− x2, y2 = 2x2− x3, . . . , y2001 = 2x2001− x1; sistemulse rescrie:

y1 + y2 = 0, y2 + y3 = 0, . . . , y2000 + y2001 = 0, y2001 + y1 = 0.

Scazând a doua ecuatie din prima, a patra ecuatie din a treia s.a.m.d., obtinem y1 == y3 = . . . = y2001 si folosind ultima ecuatie deducem ca y1 = 0. Înlocuind, obtinemyn = 0, ∀n ∈ 1, 2, . . . , 2001. De aici, x2 = 2x1, x3 = 2x1, . . . , x2001 = 22000x1 si dinultima ecuatie avem 22001x1−x1 = 0, deci x1 = 0 si atunci x2 = x3 = . . . = x2001 = 0.Solutia II. Consideram sirul (xn)n≥1 definit prin relatia de recurenta xn+2 =

= 2xn+xn+1, ∀n ≥ 1; termenul general al acestui sir este xn = A·(−1)n+B·2n, n ≥ 1unde A si B se exprima în functie de x1 si x2 prin A =

1

3(x2 − 2x1), B =

1

6(x1 + x2).

Înlocuind în ultimele doua ecuatii x2000 si x2001, se obtine un sistem în necunoscutelex1 si x2, omogen cu determinantul nenul. Deci, x1 = x2 = 0 si atunci xn = 0,∀n ∈ 1, 2, . . . , 2001.Nota. Daca am fi avut un numar par de necunoscute, sistemul obtinut în x1

si x2 ar fi fost nedeterminat: x2 = −x1. Se obtine x1 = x3 = . . . = x2k−1 = α,x2 = x4 = . . . = x2k = −α, ∀α ∈ R.

XI.22. Sirul (xn) se defineste prin relatiile x0 = 1, x1 =1

2si xn+2 = xn+1x

2n,

n ∈ N∗. Sa se afle expresia termenului general xn si limn→∞xn.

Adrian Corduneanu, IasiSolutie. Se verifica imediat prin inductie ca xn > 0, ∀n ∈ N. Notând yn = lnxn,

prin logaritmarea relatiei date obtinem pentru (yn)n≥0 recurenta yn+2 = yn+1+2yn,

cu y0 = ln 1 = 0, y1 = ln1

2= − ln 2. De aici, yn = [(−1)n − 2n] ln 3

√2 si deci

65

Page 70: Revista (format .pdf, 1.6 MB)

xn = eyn =¡3√2¢(−1)n−2n

; rezulta ca limn→∞xn = 0.

XI.23. Sa se arate ca nu exista functii f : R→ R continue si neconstante astfelîncât f (Q) ⊂ Z.

Constantin Cocea, IasiSolutie. Fie x0 ∈ R\Q si (xn)n ⊂ Q, xn → x0. Cum f continua, avem ca

f (xn) → f (x0) si deoarece f (xn) ∈ Z, obtinem ca f (x0) ∈ Z, deci f (R\Q) ⊆ Z.Rezulta ca f (R) ⊆ Z si cum f continua, urmeaza ca f (R) = c, cu c ∈ Z, adica feste constanta.XI.24. Fie f : R → R o functie continua cu proprietatea ca exista (an)n un sir

fara limita astfel încât (f (an))n este convergent. Aratati ca f nu este injectiva.Ovidiu Munteanu, student, Brasov

Solutie. Cum f este continua, f (R) este un interval. Sa presupunem prin absurdca f este injectiva. Folosind si continuitatea, rezulta ca f este strict monotona siatunci functia f−1 : f (R)→ R este strict monotona si continua.Fie l ∈ R astfel încât lim

n→∞ f (an) = l. Daca l ∈ f (R), din continuitatea lui f−1

urmeaza ca exista limn→∞ an = lim

n→∞ f−1 (f (an)) = f−1 (l), contrar ipotezei. Daca l /∈/∈ f (R), cum f−1 este strict monotona, rezulta ca exista lim

n→∞ an = limn→∞ f−1 (f (an))

în R, contrar ipotezei.XI.25. Fie f : R→ R o functie derivabila pe R si f 0 : R→ R derivata sa având

proprietatea ca ∀ (xn)n≥1 astfel încât (f 0 (xn))n≥1 este convergent urmeaza ca sirul(xn)n≥1 este convergent. Sa se studieze monotonia functiei g : R→ R definita pring = f 0 f 0.

Dumitru Gherman, PascaniSolutie. Aratam mai întâi ca functia f 0 este injectiva. În caz contrar, exista

a 6= b cu f 0 (a) = f 0 (b); definim sirul (xn)n prin xn =

½a, n imparb, n par

. Atunci,

exista limn→∞ f 0 (xn) = f 0 (a) = f 0 (b), însa nu exista limita sirului (xn)n, ceea ce ar

contrazice ipoteza. Ramâne, deci, ca f 0 este injectiva. Cum f 0 are proprietatea luiDarboux, rezulta ca este strict monotona. În consecinta functia g = f 0 f 0 este strictcrescatoare.

Clasa a XII-aXII.21. Sa se determine functiile continue si crescatoare f : [0, 1] → R care

satisfac conditiaZ 1

0

f (sinx) dx+

Z 1

0

f (ln (1 + x)) dx = 1 +

Z 1

0

f2 (x) dx, ∀x ∈ [0, 1] .Dumitru Gherman, Pascani

Solutie. Pentru x ∈ [0, 1] avem 0 ≤ sinx ≤ x ≤ 1 si 0 ≤ ln (1 + x) ≤ x ≤ 1.Cum f este crescatoare, rezulta ca f (sinx) ≤ f (x) si f (ln (1 + x)) ≤ f (x) deunde, prin integrare, obtinem

R 10f (sinx) dx +

R 10f (ln (1 + x)) dx ≤ 2

R 10f (x) dx,

sau 1 +R 10f2 (x) dx − 2 R 1

0f (x) dx ≤ 0, adica

R 10[f (x)− 1]2 dx ≤ 0. Cum f este

continua, rezulta ca f (x) = 1, ∀x ∈ [0, 1], functie care în mod evident satisfaceconditia din ipoteza.

XII.22. Fie f : [0, 1] → R o functie cu proprietatea ca exista L ≥ 0 astfel ca66

Page 71: Revista (format .pdf, 1.6 MB)

|f (x)− f (y)| ≤ L |x− y| , ∀x, y ∈ [0, 1]. Sa se arate caZ 1

0

f2 (x) dx−·Z 1

0

f (x) dx

¸2≤ L2

12.

Dan Stefan Marinescu, HunedoaraSolutie. Pentru ∀x, y ∈ [0, 1], avem:

|f (x)− f (y)| ≤ L |x− y|⇔ [f (x)− f (y)]2 ≤ L2 (x− y)2 ⇔⇔ f2 (x)− 2f (x) f (y) + f2 (y) ≤ L2

¡x2 − 2xy + y2

¢.

Fixând y ∈ [0, 1] si integrând în raport cu x pe intervalul [0, 1], obtinem:Z 1

0

f2 (x) dx− 2f (y)Z 1

0

f (x) dx+ f2 (y) ≤ L2µ1

3− y + y2

¶.

Integrând în raport cu y pe [0, 1], vom avea:Z 1

0

f2 (x) dx− 2Z 1

0

f (x) dx

Z 1

0

f (y) dy +

Z 1

0

f2 (y) dy ≤ L2µ1

3− 12+1

3

¶,

de undeR 10f2 (x) dx−

hR 10f (x) dx

i2≤ L2

12.

XII.23. a) Sa se arate ca x ∈ Zn este inversabil daca si numai daca ∃f ∈ Zn[X]astfel încât f(0) = 0 si f(x) = 1.

b) Sa se arate ca exista f ∈ Zn[X] astfel încât f(0) = 0 si f(x) = 1, ∀x ∈ Zn, xinversabil.

***Solutie. a) Fie bx ∈ U (Zn); definim f ∈ Zn [X], f (X) = bx−1X, polinom care

are proprietatile ca f(b0) = b0 si f(bx) = b1. Reciproc, fie bx ∈ Zn si f ∈ Zn [X],f (X) = ba0Xm + . . . + bam astfel încât f(b0) = b0 si f(bx) = b1. Atunci bam = b0 si cumf 6= b0, exista g ∈ Zn [X] cu f (X) = X · g (X). Pentru bx, obtinem ca bxg (bx) = b1,adica bx ∈ U (Zn) si bx−1 = g(bx).

b) Fie U (Zn) = bx1, bx2, . . . , bxk; se stie ca bx1bx2 · · · bxk = −b1. Atunci polinomulf = (−1)k

kQi=1

(X − bxi) + b1 verifica toate conditiile cerute.XII.24. Fie (G, ·) un grup, x, y ∈ G si m,n, k ∈ N∗ astfel încât xmn = e si

xmyx−m = yk, unde e este elementul neutru. Sa se arate ca ykn−1 = e (generalizare

a problemei XII.15 din nr. 2/2000).Mihaela Negrea si Florin Popovici, Brasov

Solutie. Avem:

yk2

=¡yk¢k=¡xmyx−m

¢. . .¡xmyx−m

¢= xmykx−m = xmxmyx−mx−m = x2myx−2m.

Prin inductie matematica se arata ca ykj

= xjmyx−jm, ∀j ∈ N∗. Urmeaza cayk

n

= xnmyx−nm si folosind ipoteza obtinem ykn

= y, de unde prin simplificare cuy rezulta concluzia.

XII.25. Fie (G, ·) un grup cu n2 − n + 1 elemente astfel încât f : G → G,f (x) = xn sa fie endomorfism. Sa se demonstreze ca grupul G este abelian.

Ovidiu Munteanu, student, BrasovSolutie. Avem xnyn = (xy)n = x (yx)n−1 y, deci xn−1yn−1 = (yx)n−1. Atunci

67

Page 72: Revista (format .pdf, 1.6 MB)

ynxn = (yx)n = yx (yx)n−1 = yxxn−1yn−1 = yxnyn−1, de unde xnyn−1 = yn−1xn.Astfel, rezulta ca xn(n−1)yn(n−1) =

¡xn−1

¢n(yn)

n−1= (yn)

n−1 ¡xn−1

¢n= yn(n−1)×

×xn(n−1). Notând m = ordG, avem ca m = n2−n+1, deci n (n− 1) = m−1, adicaxm−1ym−1 = ym−1xm−1, ceea ce arata ca x−1y−1 = y−1x−1, ∀x, y ∈ G, si atuncixy = yx, ∀x, y ∈ G.

Solutiile problemelor pentru pregatirea concursurilordin nr.2/2001

A. Nivel gimnazialG1. Trei elevi scriu câte un numar având 2001 cifre: A = a2001a2000 . . . a2a1,

B = b2001b2000 . . . b2b1, C = c2001c2000 . . . c2c1. Sa se arate ca în scrierea acestornumere exista trei pozitii m, n si p astfel încât am = an = ap, bm = bn = bp sicm = cn = cp.

Gabriel Popa, IasiSolutie. Sa gândim cele trei numere scrise unele sub altele; citind pe verticala,

obtinem astfel 2001 numere de câte trei cifre akbkck, k = 1, 2, . . . , 2001. Acestenumere iau valori între 000 si 999, asadar 1000 de posibilitati. Conform principiu-lui cutiei, între cele 2001 numere sunt macar trei egale: exista m,n, p astfel încâtambmcm = anbncn = apbpcp, de unde concluzia.

G2. Determinati m ∈ N∗ maxim astfel încât1

m(a+ 3) (a+ 5) (a+ 7) (a+ 9) sa

fie numar natural pentru orice a numar natural impar.Gheorghe Iurea, Iasi

Solutie. Cum a = 2k + 1, k ∈ N, trebuie determinat numarul natural maximm care divide 16 (k + 2) (k + 3) (k + 4) (k + 5), ∀k ∈ N. Cautam m de forma 16n,cu n | (k + 2) (k + 3) (k + 4) (k + 5), ∀k ∈ N. Pentru k = 0 si k = 4 obtinemca n este un divizor comun al numerelor 2 · 3 · 4 · 5 si 6 · 7 · 8 · 9, deci n | 23 · 3.Rezulta ca valoarea maxima posibila a lui n este 24. Pe de alta parte, produsul(k + 2) (k + 3) (k + 4) (k + 5) are patru factori consecutivi si atunci se divide cu 3 sicu 8, deci cu 24. Prin urmare, valoarea maxima cautata a lui m este 16 · 24 = 384.G3. Fie ABC un triunghi cu unghiul A ascutit. Pe laturile [AB] si [AC] se

construiesc în exterior triunghiurile ADB echilateral, respectiv AEC dreptunghic cum( bA) = 90, m( bE) = 60. Fie M si N mijloacele segmentelor [CE], respectiv [BC].

a) Aratati ca DM ≡ BE;b) Aflati masurile unghiurilor 4DMN .

Adrian Zanoschi, IasiSolutie. a) Triunghiul AMC este isoscel,

deci m(\MAC) = m(\ACM) = 30. Atuncim(\DAM) = 60 + m( bA) + 30 = 90++m( bA) = m(\BAE). Daca tinem seama caDA = BA si AM = AE, urmeaza congruentatriunghiurilor4ADM si4ABE (L.U.L), deciDM = BE. b) [MN ] este linie mijlocie în

4CBE, deci MN =BE

2=

DM

2(1)

68

Page 73: Revista (format .pdf, 1.6 MB)

si \NMC ≡ \BEC. Cum \AMD = \AEB din congruenta de triunghiuri doveditala a), avem ca m(\AMD) +m(\NMC) = m(\AEM) = 60, deci m(\DMN) = 120−−60 = 60. Tinând seama de (1), rezulta ca4DMN este dreptunghic, cu unghiurilerespectiv de 30, 60, 90.

G4. Fie ABC un triunghi oarecare si A1, B1, C1 proiectiile vârfurilor A, B sirespectiv C pe laturile opuse. Ce conditii trebuie impuse triunghiului ABC pentruca între 4A1B1C1 si 4ABC sa existe o asemanare?

Paraschiva Bîrsan, IasiSolutie. În cazul în care4ABC este ascutitunghic, între unghiurile sale si cele ale

triunghiului ortic exista legaturile: m( bA1) = 180− 2m( bA), m( bB1) = 180− 2m( bB),m( bC1) = 180 − 2m( bC). Între 4ABC si 4A1B1C1 exista sase posibile asemanari.Daca, de exemplu, 4ABC ∼ 4A1B1C1, atunci bA ≡ bA1, bB ≡ bB1, bC ≡ bC1, de undem( bA) = 180 − 2m( bA) si analoagele, deci m( bA) = m( bB) = m( bC) = 60. La aceeasiconcluzie se ajunge si în celelalte cazuri.Daca 4ABC este obtuzunghic cu m( bA) > 90, atunci m( bA1) = 2m( bA) − 180,

m( bB1) = 2m( bB), m( bC1) = 2m( bC). Întrcât A = A1 ar conduce la m( bA) = 2m( bA)−180 ⇔ m( bA) = 180 si analog B = B1 ⇔ m( bB) = 0, C = C1 ⇔ m( bC) = 0,situatii inacceptabile, ramâne în esenta de studiat subcazul: 4ABC ∼ 4B1C1A1.

Obtinemm( bA) = 2m( bB),m( bB) = 2m( bC),m( bC) = 2m( bA)−180, decim( bC) = 180

7,

m( bB) = 2m( bC), m( bA) = 4m( bC).În concluzie, numai triunghiurile echilaterale si cele cu unghiurile

π

7,2π

7si4π

7sunt asemenea cu propriile triunghiuri ortice.

G5. Fiind dat triunghiul ABC, sa se determine multimea punctelor P din spatiucare satisfac conditia PA2 + PB2 = 2PC2 + CA2 + CB2.

Dan Popescu, SuceavaSolutie. Evident, C este punct al multimii cautate. Fie P 6= C un punct

al locului geometric definit în enunt si fie α = m(\PCM), unde M este mijlocul

segmentului [AB]. Din teorema medianei, PM2 =1

2

¡PA2 + PB2

¢ − 1

4AB2 si

CM2 =1

2

¡CA2 + CB2

¢ − 1

4AB2. Însa conform teoremei cosinusului aplicata în

4PCM , avem PM2 = PC2 + CM2 − 2PC · CM · cosα. De aici, prin înlocuiresi folosind ipoteza, obtinem ca PC · CM · cosα = 0, deci P apartine planului π cecontine punctul C si este perpendicular pe CM . Se verifica faptul ca orice punct alplanului π satisface conditia din ipoteza.

B. Nivel licealL1. Sa se rezolve în R sistemul de ecuatii:

x2 + λy2 = 1, u2 + λv2 = 1, xu− λyv = 0

în care x, y, u, v sunt necunoscute si λ > 0 o constanta.Adrian Corduneanu, Iasi

Solutie. Prima ecuatie se poate scrie x2 +³√

λy´2= 1, deci exista ϕ ∈ [0, 2π)

astfel încât x = cosϕ,√λy = sinϕ. Analog, gasim u = cosψ,

√λv = sinψ. Prin

69

Page 74: Revista (format .pdf, 1.6 MB)

înlocuire în a treia ecuatie, obtinem cosϕ cosψ−sinϕ sinψ = 0, deci cos (ϕ+ ψ) = 0,

adica ψ =³π2+ kπ

´− ϕ, k ∈ Z. Atunci u = cosψ = (−1)k sinϕ, iar √λv = sinψ =

= (−1)k cosϕ. Se verifica acum imediat ca solutiile sistemului sunt date de

x = cosϕ, y =1√λsinϕ, u = (−1)k sinϕ, v = (−1)k√

λcosϕ, cu ϕ ∈ [0.2π)

L2. Sa se arate canX

k=0

Ckn

Fk≥ 4n

F2n, unde F0 = F1 = 1 si Fn+1 = Fn + Fn−1.

Mihail Bencze, BrasovSolutie. Se cunoaste faptul ca termenul general al sirului lui Fibonacci este dat

de Fn =1√5

¡αn+1 − βn+1

¢, unde α =

1 +√5

2, β =

1−√52

. Atunci

nXk=0

CknFk =

1√5

"nX

k=0

Cknα

k+1 −nX

k=0

Cknβ

k+1

#=

=1√5[α (α+ 1)

n − β (β + 1)n] =

1√5

£α2n+1 − β2n+1

¤= F2n.

Folosind inegalitatea dintre media aritmetica ponderata si media armonica ponder-ata, obtinemPn

k=0CknFkPn

k=0Ckn

≥Pn

k=0CknPn

k=0

Ckn

Fk

⇔ F2n2n≥ 2nPn

k=0

Ckn

Fk

⇔nX

k=0

Ckn

Fk≥ 4n

F2n.

L3. Fie ABC un triunghi ascutitunghic, A1, B1 si C1 picioarele înaltimilor, Ocentrul cercului circumscris si A2, B2, C2 punctele de intersectie a dreptelor OA,OB, OC cu dreptele B1C1, C1A1 si respectiv A1B1. Sa se arate ca dreptele A1A2,B1B2 si C1C2 sunt concurente.

Constantin Cocea, IasiSolutie. Se stie ca AA1, OA sunt simetri-

ce fata de bisectoarea unghiului A, deci [OAC ≡≡ \A1AB,\OAB ≡ \A1AC. Atunci

A2B1A2C1

=tg [OACtg\OAB

=tg \A1ABtg\A1AC

=ctgB

ctgC.

Rezulta de aici caA2B1A2C1

· B2C1B2A1

· C2A1C2B1

=ctgB

ctgC· ctgCctgA

· ctgActgB

= 1,

si atunci având în vedere reciproca teoremei lui Ceva aplicata în 4A1B1C1, amprobat concurenta dreptelor A1A2, B1B2 si C1C2.

L4. Fie f : R→ R o functie primitivabila astfel încât |f (x)| ≤ a < 1, ∀x ∈ R, Fo primitiva a sa si b ∈ R, a < b. Sa se arate ca:(i) exista o unica functie g : R → R astfel încât F (x+ g (x)) = x + bg (x) ,

x ∈ R;(ii) functia g este continua si derivabila pe R.

Dan Stefan Marinescu, Hunedoara si Ioan Serdean, Orastie

70

Page 75: Revista (format .pdf, 1.6 MB)

Solutie (data de Gheorghe Iurea, Iasi). Pentru x ∈ R fixat consideramfunctia ϕ : R → R, ϕ (t) = F (x+ t) − bt. Avem ϕ0 (t) = f (x+ t) − b si atunciϕ0 (t) ≤ a− b, ∀t ∈ R. Cum a < b rezulta ϕ0 (t) < 0, deci ϕ este strict descrescatoare(1). De asemenea pentru t > 0 rezulta

R t0ϕ0 (x) dt ≤ R t

0(a− b) dx, de unde

ϕ (t) ≤ ϕ (0) + (a− b) t, t > 0, (2)

iar pentru t < 0 rezultaR 0tϕ0 (x) dx ≤ R 0

t(a− b) dx, de unde

ϕ (t) ≥ (a− b) t+ ϕ (0) , t < 0. (3)

Din (2) si (3) rezulta limt→∞ϕ (t) = −∞ si lim

t→−∞ϕ (t) = ∞ si folosind (1) deducem

ca ϕ este bijectiva. Prin urmare ∀x ∈ R, ∃tx ∈ R, tx unic astfel ca: ϕ (tx) = x.Functia g : R → R, g (x) = tx este functia cautata. Cum ϕ0 (t) 6= 0 ∀t ∈ R,g (x) = tx = ϕ−1 (x) este derivabila pe R.L5. Fie f, g : R → R doua functii continue, f para si g impara. Daca graficul

lui f admite asimptota orizontala y = b ∈ R, calculati limitele:

limn→∞

1

n

nZ−n

f (x)

1 + eg(x)dx, si lim

n→∞1

n

nZ−n

ex2

sin e−x2

1 + ex3dx .

Dumitru Gherman, PascaniSolutie. Efectuând pe parcurs schimbarea de variabila x = −t, avem

nZ−n

f (x) dx

1 + eg(x)=

0Z−n

f (x) dx

1 + eg(x)+

nZ0

f (x) dx

1 + eg(x)=

nZ0

eg(t)f (t)

1 + eg(t)dt+

nZ0

f (x) dx

1 + eg(x)=

nZ0

f (x) dx.

Aplicând criteriul lui Stolz-Cesaro, obtinem:

limn→∞

1

n

nZ−n

f (x)

1 + eg(x)dx = lim

n→∞1

n

nZ0

f (x) dx = limn→∞

n+1Z0

f (x) dx−nZ0

f (x) dx

== lim

n→∞

n+1Zn

f (x) dx = limn→∞ f (cn) = b, (unde cn ∈ (n, n+ 1) , deci lim

n→∞ cn =∞).

Pentru a calcula a doua limita se utilizeaza rezultatul obtinut pentru f(x) =

= ex2

sin(e−x2

) si g (x) = x3 si se gaseste ca valoarea ei este 1.

71

Page 76: Revista (format .pdf, 1.6 MB)

Probleme propuse

Clasele primareP.33. Care este cel mai mare numar pe care îl spunem atunci când numaram

crescator din doi în doi, din trei în trei sau din cinci în cinci, pornind de la 1 si farasa depasim 100?(Clasa I ) Raluca Popa, eleva, Iasi

P.34. Numarul merelor de pe o farfurie este cu 3 mai mare decât cel mai marenumar natural scris cu o cifra. Numarul perelor de pe aceeasi farfurie nu depasestenumarul merelor, dar este mai mare decât jumatate din numarul acestora. Câte perepot fi pe farfurie?(Clasa I ) Înv. Maria Racu, Iasi

P.35. Care dintre numerele 3132, 8182, 3435, 3932, 2021, 5960 este intrusul?(Clasa a II-a) Matei Luca, elev, Iasi

P.36. Cum putem realiza egalitatile

4 4 4 4 = 28 si 4 4 4 4 4 4 = 28

înserând între cifrele 4 de mai sus semnele grafice +, −, ×, :, ( )?(Clasa a II-a) Alexandru-Gabriel Tudorache, elev, Iasi

P.37. Suma a doua numere naturale este 109. Daca îl dublam pe primul si îltriplam pe al doilea, suma devine 267. Care sunt numerele?(Clasa a II-a) Înv. Galia Paraschiva, Iasi

P.38. Câte înmultiri de tipul abc× 9 = 8d1e sunt posibile?(Clasa a III-a) Sergiu Diaconu, elev, Iasi

P.39. Scrieti cel mai mic numar natural de sase cifre care îndeplineste, în acelasitimp, conditiile: a) nu are cifre care se repeta; b) suma cifrelor sale este 30; c) estemai mare decât 900000.(Clasa a III-a) Înv. Maria Racu, Iasi

P.40. Emilia are de rezolvat un numar de probleme. A hotarât sa rezolve câte 4probleme pe zi. Ea lucreaza însa mai mult cu 2 probleme pe zi si termina de rezolvatcu 5 zile mai devreme. Câte probleme a avut de rezolvat si în câte zile le-a terminat?(Clasa a III-a) Înv. Doinita Spânu, Iasi

P.41. Stiind ca data de 1 Decembrie din anul 2001 a fost într-o zi de sâmbata,sa se afle care va fi urmatorul an în care ziua de 1 Decembrie se va sarbatori într-ozi de duminica.(Clasa a IV-a) Înv. Rodica Rotaru, Bârlad

P.42. Danila Prepeleac i-a propus dracului sa se întreaca la trânta, dar pentrua-l pune la încercare i-a spus ca are un unchi, mos Ursila, batrân de 999 ani si 52

saptamâni, si de-l va putea trânti pe dânsul, se vor întrece apoi amândoi . Daca1

4

din vârsta lui mos Ursila depaseste cu 220 ani5

8din vârsta nepotului, ce vârsta are

Danila?(Clasa a IV-a) Înv. Valerica Beldiman, Iasi

72

Page 77: Revista (format .pdf, 1.6 MB)

P.43. Primele douasprezece numere dintr-un sir de numere sunt: 1, 2, 0, 3, 4, 1,5, 6, 2, 7, 8, 0.

a) Scrieti urmatoarele 6 numere din sir;b) Calculati suma primelor 111 numere din sir.

(Clasa a IV-a) Alina Stan, eleva, Iasi

Clasa a V-aV.31. Sa se arate ca

1

101+

1

102+ . . .+

1

200>7

12.

Petru Asaftei, Iasi

V.32. Determinati numerele prime a, b, c pentru care 5a+ 4b+ 7c = 107.Mihai Craciun, Pascani

V.33. Un numar natural scris în baza 10 are suma cifrelor 603. Este posibil casuccesorul sau sa aiba suma cifrelor 1? Dar ca acesta sa aiba suma cifrelor 3?

Matei Luca, elev, Iasi

V.34. Aflati numarul abc, stiind ca abc = 2n · ab+ 3n · bc+ 5n · ca, unde n ∈ N.Nicolae Stanica, Braila

V.35. Se da numarul N = 77 . . . 7 cu 2002 cifre. Cercetati daca N se poate scrieca suma a doua sau trei patrate perfecte impare.

Tamara Culac, Iasi

Clasa a VI-aVI.31. Fie S =a1a1 . . . a1| z

k1 cifre

+ a2a2 . . . a2| z k2 cifre

+ . . .+ anan . . . an| z kn cifre

, unde k1, k2, . . . kn ≥ 2.

Aratati ca S se divide cu 4 daca si numai daca a1 + a2 + . . .+ an se divide cu 4.Dumitru Gherman, Pascani

VI.32. Aflati ab stiind ca ab = (a− b)!·¡ba− 3¢ (unde 0! = 1, iar n! = 1·2·3·. . .·n,∀n ≥ 1).

Nicolae Stanica, Braila

VI.33. Într-o urna sunt bile albe, rosii, negre si albastre. Numarul bilelor albe

este3

5din numarul celorlaltor bile; bilele rosii reprezinta jumatate din celelalte bile,

iar bilele negre a treia parte din numarul celorlaltor bile. Daca extragem o bila,calculati probabilitatea ca aceasta sa fie rosie sau albastra.

Marcel Rotaru, Bârlad

VI.34. În 4ABC, fie M mijlocul laturii [BC]. Daca d (M,AC) =AB

2, aratati

ca m( bA) = 90.N.N.Hârtan, Iasi

VI.35. În 4ABC, m( bA) = 60 si m( bC) = 45. Bisectoarea [AD si înaltimea

[BE] se intersecteaza în M (cu D ∈ [BC], E ∈ [AC]). Sa se arate ca AM

EC=2

3.

Romeo Cernat, Iasi

Clasa a VII-aVII.31. Sa se rezolve ecuatia 1+ a2x + b2x = ax+ bx+ axbx, cu a, b ∈ R∗, a 6= b.

Dumitru Neagu, Iasi

73

Page 78: Revista (format .pdf, 1.6 MB)

VII.32. Fie a si b, c lungimile ipotenuzei si respectiv catetelor unui triunghi

dreptunghic. Sa se arate ca¡a2 + b2 + c2

¢µ 1a2+1

b2+1

c2

¶≥ 10.

Claudiu - Stefan Popa, IasiVII.33. Triunghiul ABC are unghiul A obtuz si semiperimetrul p. Cercurile de

diametre [AB] si [AC] delimiteaza o suprafata comuna S. Aflati valoarea de adevara propozitiei: ”Exista P ∈ S astfel încât d1 + d2 + d3 = p”, unde di sunt distantelede la P la laturile triunghiului ABC.

Catalin Calistru, IasiVII.34. Fie ABC un triunghi, iar I un punct interior lui. Daca cercurile înscrise

în triunghiurile AIB, BIC si CIA sunt congruente si tangente doua câte doua, atunci4ABC este echilateral.

Ioan Sacaleanu, HârlauVII.35. Fie ABCD un paralelogram, O centrul cercului circumscris 4ABD, iar

H ortocentrul 4BCD. Sa se arate ca punctele A,O,H sunt coliniare.Constantin Cocea si Dumitru Neagu, Iasi

Clasa a VIII-aVIII.31. Sa se determine multimea A =

½(m,n) ∈ N∗ ×N∗ | 4n− 1

mn+ 1∈ N

¾A.V.Mihai, Bucuresti

VIII.32. Sa se rezolve ecuatia1

x2 − x+ 1+

2

x2 − x+ 2+ . . .+

2002

x2 − x+ 2002=

= 2002.Mihaela Predescu, Pitesti

VIII.33. Determinati functiile f : R → R pentru care 1 + f (x+ y) ≤ f (x)++f (y) ≤ x+ y + 2, ∀x, y ∈ R.

Gheorghe Iurea, IasiVIII.34. Fie AB dreapta solutiilor ecuatiei x − y = 5 si CD dreapta solutiilor

ecuatiei x + y = 3, cu A,C ∈ Ox, B,D ∈ Oy. a) Aratati ca AB ⊥ CD; b)Calculati aria si perimetrul triunghiului BCD; c) Aratati ca AD ⊥ BC.

Vasile Solcanu, Bogdanesti (Suceava)VIII.35. Fie ABCDA0B0C0D0 un cub de muchie a. Determinati pozitiile puncte-

lor M ∈ (BB0) si N ∈ (CC0) pentru care perimetrul patrulaterului strâmb AMND0

este minim; aflati aceasta valoare minima.Mihaela Bucataru, Iasi

Clasa a IX-aIX.31. Fie multimile A =

©x2 + x |x ∈ Zª, B =

©x3 + x |x ∈ Zª, C =

©x4+

+x3 + x2 + x |x ∈ Zª. D =©2x4 |x ∈ Zª. Determinati A∩C, B∩D, A∩D, A∩B.

Andrei Nedelcu, IasiIX.32. Fie fi, gi : R → R, i ∈ 1, 2, . . . , n, functii care pastreaza semnul vari-

abilei. Sa se rezolve sistemul:x1 + f1 (x1) + g1 (x1) = x2x2 + f2 (x2) + g2 (x1 + x2) = x3. . .xn + fn (xn) + gn (x1 + x2 + . . .+ xn) = x1.

74

Page 79: Revista (format .pdf, 1.6 MB)

Obtineti sisteme diverse prin particularizarea functiilor!Iuliana Georgescu si Paul Georgescu, Iasi

IX.33. Rezolvati în N ecuatia n−hn2

i−hn3

i+h n12

i+h n18

i= 11 ·

h n36

i.

Gheorghe Iurea, Iasi

IX.34. Fie n ∈ N, n ≥ 2. Sa se determine x1, x2, . . . , xn ∈ R stiind cax21 + x22 + . . .+ x2n +

n+ 1

2= 2

µx1 sin

π

n+ 1+ x2 sin

n+ 1+ . . .+ xn sin

n+ 1

¶.

Vladimir Martinusi, Iasi

IX.35. Aratati ca sin (cosx) + sin (cos y) < 2 cosx+ y

2, ∀x, y ∈

³0,π

2

´.

Dan Popescu, Suceava

Clasa a X-aX.31. Consideram sirurile (Fn)n≥0, (Ln)n≥0 definite prin F0 = 0, F1 = 1,

Fn+2 = Fn+1 + Fn, ∀n ≥ 0, respectiv L0 = 2, L1 = 1, Ln+2 = Ln+1 + Ln, ∀n ≥ 0.Sa se arate ca

pPnk=1 L

2k ≤

qPnk=1 F

2k−1 +

qPnk=1 F

2k+1.Mihail Bencze, Brasov

X.32. Fie (an)n≥0 un sir de numere reale si pozitive. Pentru orice n ∈ N senoteaza cu Pn graficul functiei fn (x) = anx

2 + an+1x+ an+2. Determinati legea dedefinire a sirului stiind ca parabolele Pn au vârfurile pe axa Ox.

Temistocle Bîrsan, Iasi

X.33. Fie functiile f, g : R → (0,∞), f (x) = ax, g (x) = bx, unde a ∈ (1,∞),b ∈ (0, 1), ab 6= 1. Sa se arate ca exista o infinitate de paralelograme cu vârfurile pereuniunea graficelor celor doua functii.

Petru Asaftei, Iasi

X.34. Sa se arate ca ecuatiile 4 · 9x + (4x− 45) · 3x + 11 − x = 0 si 4 · 18x−−3 · 6x − 128 · 3x + 2x + 32 = 0 sunt echivalente.

Marcel Chirita, Bucuresti

X.35. Fie n ∈ N, n ≥ 3 si z1, z2, . . . , zn ∈ C∗ de module egale astfel încâtzkzt∈ C\ ±1,±i ∀ 1 ≤ k 6= t ≤ n. Notam aj =

1

zj

Qk 6=j zk + z2j , ∀j = 1, n. Aratati

ca daca doua dintre numerele aj sunt reale, atunci toate sunt reale. În plus, dacan 6= 4, atunci Qn

k=1 zk = 1 (în legatura cu problema X.86 din R.M.T. nr. 3-4/2000).Daniel Jinga, Pitesti

Clasa a XI-aXI.31. Fie A,B ∈Mn (Q). Daca detA 6= detB, demonstrati ca det (A+ πB) 6=

6= det (B + πA).Paul Georgescu si Gabriel Popa, Iasi

XI.32. Fie sn =Pn

k=1

h¡k2 + k + 1

¢Pkp=0 (−1)p Cp

k (k − p)ki. Calculati

limn→∞

³n+1√1 + sn − n

p1 + sn−1

´.

Stefan Alexe, Pitesti

75

Page 80: Revista (format .pdf, 1.6 MB)

XI.33. Fie (xn)n≥1 un sir de numere reale pozitive cu proprietatea ca existap ∈ N∗ si numerele a1, a2, . . . , ap > 0 cu a1 + a2 + . . . + ap > 1 astfel încât xn == a1xn+1 + a2xn+2 + . . .+ apxn+p, ∀n ≥ 1. Sa se arate ca inf xn | n ∈ N∗ = 0.

Marian Tetiva, Bârlad

XI.34. Fie a ∈ (0, 1) ∪ (1,∞) si b un numar între −√2 ln a si √2 ln a. Stabilitisemnul functiei f : R→ R, f (x) =

1

2(ax + a−x)− bx.

Gheorghe Costovici, Iasi

XI.35. Fie f : [0,∞)→R o functie continua, iar (ak)nk=1, (bk)nk=1 progresii geome-trice astfel încât ak < bk, ∀k = 1, n. Daca f (a1a2 · · · an) < 1 si f (b1b2 · · · bn) > 1,aratati ca exista (ck)

nk=1 progresie geometrica, ck∈(ak, bk) astfel încât f(c1c2 · · · cn)=1.

Doru - Dumitru Buzac, Iasi

Clasa a XII-aXII.31. Fie M =

©A ∈M2 (C) | A2 = I2

ª. Determinati toate subgrupurile lui

(M2 (C) , ·) continute în M .Angela Tigaeru si Catalin Tigaeru, Suceava

XII.32. Fie (G, ·) grup, iar a ∈ G\e fixat. Aratati ca numarul morfismelorsurjective de la G la (Z3,+) cu proprietatea ca f (x) = b2 ⇔ x = a este egal cunumarul subgrupurilor H ale lui G care nu-l contin pe a si care au proprietatea cax3 ∈ H, ∀x ∈ G.

Dana Stan, eleva, Iasi

XII.33. Fie f : R → R o functie monotona al carei grafic are un centru desimetrie ce nu apartine graficului. Aratati ca f nu admite primitive.

Oana Marangoci, eleva, Pascani

XII.34. Fie f : [0, 1]→ (0,∞) o functie care admite primitive si fie F o primitivaa sa cu F (1) = 0. Aratati ca exista c ∈ (0, 1) astfel încât F (c) > − c

2ec

2

f (c).

Rodica Luca Tudorache, Iasi

XII.35. Fie f, g : R→ R, unde f este o functie injectiva si g (x) =Pn

k=0 a2k+1×× [f (x)]2k+1 + sin f (x), cu a1 ∈ [1,∞) si a2k+1 ∈ (0,∞), ∀k = 1, n. Aratati ca fadmite primitive daca si numai daca g admite primitive.

Lucian Georges Ladunca, Iasi

Probleme pentru pregatirea concursurilor

A. Nivel gimnazialG21. Sa se afle restul împartirii prin 43 a numarului 62002

n

, n ∈ N.Catalin - Cristian Budeanu, Iasi

G22. Sa se arate ca între n si n! exista cel putin un numar prim, oricare ar fin ∈ N\ 0, 1, 2.

***

G23. Sa se rezolve în Z2 ecuatia x2 + 10x+ y! = 2002.Adrian Zanoschi, Iasi

76

Page 81: Revista (format .pdf, 1.6 MB)

G24. Aflati câte numere de 4 cifre au proprietatea ca cifrele citite de la stângaspre dreapta sunt invers proportionale cu cifrele citite de la dreapta spre stânga.Aceeasi problema pentru numerele de 3, respectiv 5 cifre.

Gabriel Popa, IasiG25. Sa se arate ca pentru orice x, y, x ∈ R, exista a, b, c ∈ Z astfel încât

(x− a)2+ (y − b)

2+ (z − c)

2 ≤ 34. Generalizare.

Vladimir Martinusi, IasiG26. Daca suma, produsul si câtul a doua numere irationale sunt numere

rationale, calculati suma cuburilor celor doua numere.Claudiu - Stefan Popa, Iasi

G27. Determinati n ∈ N∗ minim pentru care 2n − 1 este divizibil cu 125.Gheorghe Iurea, Iasi

G28. Sa se arate ca ecuatia x4−(a+ b)x3+(a+ b+ ab− 2)x2−¡a2+ b2− a− b¢x

+(a− 1) (b− 1) = 0 are cel putin doua solutii reale pentru orice a, b ∈ R.Marian Teler, Costesti (Arges)

G29. Fie triunghiul ABC cum( bA) < m( bC). Pe bisectoarea interioara a unghiuluibB luam un punct E astfel încât \EAB ≡ \ACB. Se prelungeste latura [BC] cusegmentul [BD] ≡ [AB], B între C si D. Sa se arate ca mijlocul M al segmentului[AC] se afla pe dreapta DE.

Constantin Chirila, IasiG30. Fie triunghiul AB0B1 dreptunghic în B0 si triunghiurile ABiBi+1 cu

BiBi+1 ⊥ ABi, ∀i ∈ N∗, iar m( \BiABi+1) = 30, ∀i ∈ N.

a) Demonstrati ca punctele A, Bq si Br sunt coliniare, unde q = 2¡n3 − n+ 1

¢,

r = 32002 − 32000 + 31998 − 31996 + . . .+ 32 − 30.b) Aflati aria triunghiului AB2001B2002 functie de a = B0B1.

Romanta Ghita si Ioan Ghita, BlajG31. Se considera un triunghi isoscel ABC cu baza BC = 2

√2 cm. Fie punctele

variabile M ∈ (AB) si N ∈ (AC) astfel încât [AM ] ≡ [CN ]. Fie O mijlocul segmen-tului [MN ] si P intersectia dreptelor AO si BC. Aflati perimetrul 4ABC stiind caaria minima a reuniunii suprafetelor triunghiulare [MBP ] si [NCP ] este

√3 cm2.

Adriana Maxiniuc, BotosaniG32. Fie patrulaterul convex ABCD cu m( bA) = m( bD) = 90, M un punct pe

dreapta AD, iar N ∈ BC astfel încât MN ⊥ BC. Aratati ca SCMB ≥ SAND.Neculai Roman, Mirceasti (Iasi)

G33. În triunghiul ABC cu m( bA) = 2α, fie D ∈ (BC) piciorul bisectoareiunghiului bA, iar M , N puncte pe (AB) respectiv (AC). Daca P = AD ∩MN ,

demonstrati ca1

AM+

1

AN=2cosα

AP(în legatura cu C:2402 din G.M. 5-6/2001).

Mihaela Bucataru, Iasi

G34. În triunghiul ABC având m(\BAC) = 135 se înscrie patratul MNPQ cuM,N ∈ (BC), P ∈ (CA) si Q ∈ (AB). Aratati ca:1

AM

AN=

b+ c√2

c+ b√2; 2

BM

CN=

c

b· AMAN

.Temistocle Bîrsan, Iasi

77

Page 82: Revista (format .pdf, 1.6 MB)

G35. Fie [ABCD] un tetraedru. În planele (ABC), (ADC), (ADB) consideramtangentele în A la cercurile circumscrise triunghiurilor ABC, ADC respectiv ADB,

care intersecteaza dreptele BC, CD, DB în M , N respectiv P . Notam x =MB

MC,

y =NC

ND, z =

PD

PA. Sa se arate ca

1

1 + x+

1

1 + y+

1

1 + z≥ 1.Marian Ionescu, Pitesti

B. Nivel licealL21. Rezolvati în N2 ecuatia a2 + 3b2 = 2n, unde n ∈ N este fixat.

Gheorghe Iurea, IasiL22. Fie n ∈ N un numar scris în baza 10. Acestui numar îi adaugam la sfârsit

147, numarului obtinut îi adaugam din nou la sfârsit 147 si asa mai departe. Aratatica printre numerele astfel obtinute exista numere compuse.

Adrian Zanoschi, IasiL23. Fie f : R → R o functie periodica de perioada principala T astfel încât pe

[0, T ] f se anuleaza de un numar finit de ori si fie (xn)n≥1 un sir de numere realenenule. Aratati ca exista α ∈ R astfel încât f (αxn) · f (α+ xn) 6= 0 ∀n ∈ N∗.

Paul Georgescu si Iuliana Georgescu, IasiL24. Fie A,B ∈Mn (R) matrice nesingulare cu detA+detB = 0. Exista α > 0

astfel încât A2B −B2A = αA?Catalin Calistru, Iasi

L25. Fie A ∈Mn (R) o matrice cu proprietatea ca existam ∈ N, m ≥ 3 si α ∈ R,|α| ≤ 1, astfel încât Am+1 − αAm − αA+ In = On. Sa se arate ca |detA| = 1.

Lucian Georges Ladunca, IasiL26. Fie f : Sn → Sn endomorfism astfel încât exista τ ∈ Sn pentru care

(f f) (σ) = τστ−1, ∀σ ∈ Sn. Aratati ca f are un punct fix (i.e.∃ω ∈ Sn, f (ω) = ω).Ovidiu Munteanu, Brasov

L27. Fie (A,+, ·) un inel cu unitate si n, k ∈ N∗, k impar, astfel încât xn+k = xn,∀x ∈ A. Sa se arate ca xk+1 = x, ∀x ∈ A (în legatura cu C: 1896 din G.M. nr.1/1997).

Dragos Deliu si Marian Tetiva, Bârlad

L28. Fie ABC si A0B0C0 doua triunghiuri ascutitunghice. Daca³ aa0´2≥µb

b0

¶2≥

≥³ cc0´2≥ S

S0, aratati ca triunghiurile sunt asemenea.

Ioan Sacaleanu, HârlauL29. Fie ABC un triunghi, iar C un cerc tangent laturilor [AB] si [AC] în F ,

respectiv E si care intersecteaza latura [BC] în M si N . Fie X un punct interiortriunghiului astfel încât exista un cerc C1 tangent laturilor [XB] si [XC] în Z, re-spectiv Y si care taie [BC] tot în M si N . Demonstrati ca patrulaterul EFZY esteinscriptibil.

Neculai Roman, Mircesti, (Iasi)L30. Fie ABC un triunghi echilateral, iar P un punct în planul triunghiului.

Notam cu A1, B1, C1 simetricele lui P fata de BC, CA si respectiv AB. Sa se arateca se poate forma un triunghi având lungimile laturilor egale cu AA1, BB1, CC1.

Constantin Cocea, Iasi

78

Page 83: Revista (format .pdf, 1.6 MB)

L31. Fie (an)n≥1 un sir de numere reale astfel încât a1 = a > 1, an+1 =

= an+ppap−1n −1, ∀n ∈ N∗, unde p ∈ N, p ≥ 2. Sa se calculeze lim

n→∞annp(în legatura

cu C:1463 din G.M. nr.11/1993).Viorel Cornea si Dan Stefan Marinescu, Hunedoara

L32. Fie (xn)n≥1 un sir de numere naturale care satisface conditia·xn+1xn

¸= xn,

∀n ≥ 1, iar x1 = 2. Sa se arate ca exista α > 1 pentru care limn→∞

xnα2n

= 1.

Cristinel Mortici, TârgovisteL33. Fiem ∈ N, m ≥ 2 fixat. Aratati ca solutiile continue ale ecuatiei functionale

f

ÃmXi=1

xi

!+

X1≤<i<j≤m

f (xi − xj) = m

ÃmXi=1

f (xi)

!, ∀xi ∈ R, i = 1,m,

sunt functiile de forma f (x) = cx2, cu c ∈ R.Adrian Corduneanu, Iasi

L34. Se considera sirul de functii (fn)n∈N, fn : [−2,∞)→ R definit prin f0 (x) == x, fn (x) =

p2 + fn−1 (x), ∀n ≥ 1. Notam

f (x) =

1 + limn→∞

µfn (x)

2

¶4n, daca x ∈ [−2, 2]

limn→∞ [3− fn (x)]

4n , daca x ∈ (2,∞).

Aratati ca f (x) defineste o functie pe [−2,∞) si cercetati daca aceasta functie admiteprimitive.

Stefan Alexe, PitestiL35. Sa se arate ca exista si este unic α > 1 astfel încâtZ π/4

0

ex

ex + α sinxdx ·

Z π/4

0

sinx

ex + α sinxdx =

π2

64α

(se stie ca 0, 45 < e−π/4 < 0, 46).Gabriel Popa si Paul Georgescu, Iasi

IMPORTANT• Revista ”RECREATII MATEMATICE” poate fi consultata pe Internet laadresa www.RecreatiiMatematice.home.ro .

• În scopul unei legaturi rapide cu redactia revistei, pot fi utilizate urma-toarele adrese e-mail: [email protected], [email protected] .Pe aceasta cale colaboratorii pot purta cu redactia un dialog privitor lamaterialele trimise acesteia, procurarea numerelor revistei etc.

• La problemele de tip L se primesc solutii de la orice iubitor de matematicielementare (indiferent de preocupare profesionala sau vârsta). Fiecare din-tre solutiile acestor probleme - ce sunt publicate în revista dupa un an -va fi urmata de numele tuturor celor care au rezolvat-o.

• Adresam cu insistenta rugamintea ca materialele trimise revistei sa nu fie(sa nu fi fost) trimise si altor publicatii.

79

Page 84: Revista (format .pdf, 1.6 MB)

Filiala Iasi a SSM - repere ale activitatiiA fost cerut un raspuns succint la întrebarea ”Ce face filiala Iasi a Societatii de

Stiinte Matematice din România?”. Prin lansarea acestei întrebari se preconizeazasi raspunsul realmente concis: ”Nimic”.Acest raspuns ar circumscrie si absenta unor planuri de activitate, a unor sedinte

în care sa se voteze cu mâini pe sus, a unor actiuni de atragere de noi membri, a unorprelegeri de popularizare si poate multe alte absente. Zicem ca acesta ar fi raspunsulexact pentru cei ce-si imagineaza SSM si filialele sale ca o fabrica de suruburi ce nuproduce nici macar saibe. Zicem noi ca aceasta interpretare este mecanicista si, caurmare, acest raspuns este incomplet.Cu onoarea de a-l avea presedinte pe acad.Radu Miron si cu sansa de a avea un

secretar extrem de eficient, prof.Vasile Nechita, filiala Iasi se afirma ca un liant alunei familii de matematicieni.•De câtiva ani buni, profesorii ieseni se plaseaza în topul autorilor de carti matem-

atice scolare printre colegii lor din Bucuresti, Craiova, Constanta, Pitesti.• Cursurile anuale de perfectionare de la Busteni sunt mereu dominate de grupul

de profesori ieseni.• Suntem în calendarul national anual prin concursuri interjudetene: Adolf Haimo-

vici, Florica T.Câmpan, Recreatii Matematice.• Dupa ultima olimpiada, trei elevi de gimnaziu de la Liceul National s-au plasat

în lotul national de juniori, unul dintre ei, Pachitariu Marius câstigând recent simedalia de aur la cea de a 6-a Balcaniada Matematica pentru juniori.• Mai zicem ca Iasul a gazduit patru tabere de pregatire a juniorilor, o tabara

pentru elevii claselor a X-a, alte tabere nationale pentru premiantii olimpici.• Mai apreciem ca Gazeta Matematica, seriile A si B, are la Iasi suficienti cititori

si colaboratori.• Într-o aproximatie destul de buna si recunoscuta revista Recreatii Matematice ce

gazduieste aceste rânduri este o emanatie a familiei de matematicieni ieseni apropiatiîntre ei prin Filiala Iasi a SSM.Vinovat de abaterea de la conciziune prin extrapolari ce pot fi si subiective se face

presedintele filiale,Prof. dr. Dan Brânzei

Redactia revistei ”Recreatii Matematice” acorda câte o diploma siun premiu în carti urmatorilor elevi:

ANDRONICMihaela (Sc.nr.7 ”N.Tonitza”): 2/2001(10pb),1/2002(9pb),2/2002(9pb),DUMITRIU Alexandru (Sc.nr.7 ”N.Tonitza”): 2/2001(13pb),1/2002(6pb),2/2002(9pb),TIBA Marius (Sc.nr.11 ”O.Cazimir”): 2/2001(14pb),1/2002(9pb),2/2002(5pb),PRUTEANU Irina (Lic. ”Garabet Ibraileanu”): 2/2001(5pb),1/2002(5pb),2/2002(5pb).

Cartile au fost oferite de catre

80

Page 85: Revista (format .pdf, 1.6 MB)

Pagina rezolvitorilorBRASOV

Liceul ”N. Titulescu”.

Clasa a IX-a. AUGUSTIN Alexandru: VII (25,29), VIII (24,27), IX (26); BALAN Laurentiu Florin:

VII (26,29), VIII (24,27), IX (26); BOSCORNEA Ionut Bogdan: VII (25,26,29), VIII (27), IX (26);

BURSUCANU Lucian: VII (25,26,29), VIII (27), IX (26); CENTU Nicoleta Veronica: VII (25,26,28),

VIII (27), IX (26); EDU Filip Vladimir: VII (25,26,29), VIII (27), IX (26); GHEORGHE Liana Elena:

VII (25,26,29), VIII (27), IX (26); HALIPPA Andra: VII (25,26,29), VIII (27), IX (26); LITRA Anatolie:

VII (25,26,29), VIII (27), IX (26); MANOLE Bogdan: VII (26), VIII (24,25,27), IX (26); MOARCAS

Liviu: VII (25,26,29), VIII (27), IX (26); MOLDOVAN Ovidiu: VII (25,29), VIII (27), IX (23,26); NITA

Alina: VII (25,26,29), VIII (27), IX (26); PANA Andreea: VII (25,26,29), VIII (27), IX (26); PASCAL

Andreea: VII (25,26,29), VIII (27), IX (26); PLATON Alexandru Paul: VII (25,26,29), VIII (26,27), IX

(26); RACHITEANU Lavinia Elena: VII (25,26,29), VIII (27), IX (26); RUSEN Adrian Nicolae: VII

(25,26,29), VIII (27), IX (26); SOFLETEA Ancuta Corina: VII (25,26,29), VIII (27), IX (26); SOIMU

Adelina: VII (25,26,29), VIII (27), IX (26); TUDOR Ioana: VII (25,26,29), VIII (27), IX (26).

Clasa a X-a. BORODI Alexandru: VIII (26,27), IX (23,26), X (22,23); CIOBANU Alina: VIII

(24,27), IX (23,26), X (22,23); CIOLANEL Lucia: VIII (24,27), IX (23), X(22,23); COSTEA Adrian: VIII

(26,27), IX (23,26), X (24); DUMBRAVEANU Alexandru: VIII (26,27), IX (23,26), X (24); EFTIMIE

Madalin: VIII (24,27), IX (23,26), X (22,23); MURESANU Stefan: VIII (26,27), IX (23,26), X (24);

PASCAN Marius: VIII (24,27), IX (23,26), X (22,23); PADURET Alexandru: VIII (26,27), IX (23,26),

X (24); PATRUNJEL Alexandru: VIII (24,26,27), IX (23), X (22); POSEA Alexandra: VIII (26,27), IX

(23,26), X (24); PUIU Lavinia: VIII (24,27), IX (23,26), X (22,23).

Clasa a XII-a. ACUNUNEI Adina: X (24), XI (22,25,27), XII (27); AVRAM Elisabeta: X (24),

XI (22,25), XII (21,23); BOTA Marius: X (24), XI (22,25), XII (21,23); COSTACHE Adrian: X (24),

XI (22,25), XII (21,23); COSTACHE Lucian: X (24), XI (22,25), XII (21,23); DRAGAN Alexandru: XI

(22,25), XII (21,23,27); GEORGESCU Tudor: XI (22,23), XII (21,23,24); MIHAI Gabriela: X (24), XI

(22,25), XII (21,23); PURCARU Daniel: X (24), XI (22,25), XII (21,23); TODORACHE Adrian: X (24),

XI (22,25), XII (21,23).

CÂMPULUNG - MUSCEL (ARGES)Scoala ”Sf. Iacob”. Clasa a VI-a. BINICA Ioana: V (27,28), VI (27,29), G (6).

CRAIOVAColegiul National ”Fratii Buzesti”. Clasa a V-a. CONSTANTIN Anca: P (30-32), V (26,28);

COANDA Alina Mihaela: P (30-32), V (26,28); FOARFA Alexandra: P (30-32), V (26-30); LOLEA

Bianca - Lorena: P (30-32), V (26,28); NICHITA Raluca: P (31), V (26-30); OPREA Anda Raluca: P

(31), V (26-29); PUIU Denisa - Andreea: P (30-32), V (26,28); TUTESCU Anca Stefania: P (30-32), V

(26-30); VISANU Laura Stefania: P(31), V (26-29).

HÂRLAU (IASI)Scoala ”Petru Rares”. Clasa a IV-a (înv. CRETU Maria). SCRIPCARIU Gabriela: P (16,17,20-23);

SCRIPCARIU Ioana: P (16,17,20-23).

IASIScoala nr. 3 ”Al. Vlahuta”. Clasa a IV-a (înv. GALIA Gabriela). SOFICU Crina - Maria: P (24-

27,29, 30).

Scoala nr. 7 ”N. Tonitza”. Clasa a IV-a (înv. MELINTE Rodica). ANDRONIC Mihaela: P (24-32);

DUMITRIU Alexandru: P (24-32). Clasa a IV-a (înv. TUDOSE Elena). CIUBOTARIU Remus: P

81

Page 86: Revista (format .pdf, 1.6 MB)

(24,25,27-31); IONESCU Bogdan: P (24-32); ZORICI Iulian: P (24,27,28-32).

Scoala nr. 11 ”O. Cazimir”. Clasa a III-a (înv. PÂRÂIALA Dumitru). TIBA Marius: P (26-30).

Scoala nr. 22 ”B. P. Hasdeu”. Clasa a IV-a (înv. TÂRZIORU Iuliana). BALAUTA Bogdan - Alexan-

dru: P (24-32); CHIHAIA Mihai: P (24-32); PRODAN Andreea: P (24,25,27-32); RAITA Bogdan: P

(24-32); SILION Catalina: P (24-32); SPÂNU Dragos - Andrei: P (24-32).

Scoala nr. 23 ”T. Maiorescu”. Clasa a II-a (înv. CHIRILA Beatrice) TUDORACHE Alexandru -

Gabriel: P (24-29).

Colegiul National Iasi. Clasa a VI-a. PRELIPCEAN Cristina: V (28-30), VI (26,29,30).

Liceul de Informatica ”Gr. Moisil”. Clasa a VII-a. COSTIN Ciprian: V (28,30), VI (28,29), VII

(27,29), G (6-8,11); MIHUL Andrei: V (28), VI (29), VII (26,27,29).

Liceul Teoretic ”Garabet Ibraileanu”. Clasa a V-a. BUDEANU Stefana: P (31), V (22-25); STOIAN

Radu - Gabriel: V (21-23,25), VI (23); UNGUREANU Dragos: P (31), V (22-24, 26,28), G (7). Clasa

a VII-a. ANDRIESCU Alina: V (23,28), VI (25), VII (22,29); BRANISTEANU Stefana: V (21-23), VI

(21), VII (22,23,27), VIII (23,27), G (3,7); BUDEANU Ioana: VI (26-29), VII (26,29); JUVERDEANU

George: V (28), VI (28,29), VII (27,29), PRUTEANU Irina: V (27,28), VI (27,29), VII (27). Clasa a

IX-a. TONU Constantin: VII (22,25,29), VIII (29), G (4), IX (22); VULCU Vlad: VII (21-23,29), VIII

(23,29), IX (22).

ERRATApagina si rândul în loc de se va considera

p. 4, r. 13 Tartalia Tartagliap. 33, r. 6 n

√x1 + x2 + . . .+ xn n

√x1x2 · . . . · xn

p. 56, r. 19√a2m2 + n2

√a2m2 + b2

p. 57, r. 81+εR−1+ε

1−εR−1+ε

p. 62, r. 7 Alexandri Alecsandrip. 64, r. 7 (1 + a+ b)

0(1 + a+ b)

a

p. 72, r. 17 α2 + β2 + γ2 + αβ + αγ + βγ α2 + β2 + γ2 − αβ − αγ − βγp. 72, r. 17 x2 + y2 + z2 + xy + xz + yz x2 + y2 + z2 − xy − xz − yzp. 77, r. 16 3x− 1 = 1 3x− y = 1p. 77, r. 19 1 · 2 + 2 · 3 + n (n+ 1) 1 · 2 + 2 · 3 + . . .+ n (n+ 1)

p. 77, r. 22 înaltimea din A în Eînaltimea din [AD] ,D ∈ (BC) , în E

p. 79, r. 5BM2

MC2BN2

NC2

p. 79, r. 6BMn

MCn

BNn

NCn

82

Page 87: Revista (format .pdf, 1.6 MB)

Revista RECREAŢII MATEMATICE apare de două ori pe an (la datele de 1 martie şi 1 septembrie) şi se adresează elevilor, profesorilor, studenţilor şi tuturor celor pasionaţi de matematicile elementare. În atenţia tuturor colaboratorilor Materialele trimise redacţiei spre publicare (note şi articole, chestiuni de metodică, probleme propuse etc.) trebuie prezentate îngrijit, clar şi concis; ele trebuie să prezinte interes pentru un cerc cât mai larg de cititori. Se recomandă ca textele să nu depăşească patru pagini. Evident, ele trebuie să fie originale şi să nu fi apărut sau să fi fost trimise spre publicare altor reviste. Problemele originale destinate rubricii Probleme propuse vor fi redactate pe foi separate câte una pe fiecare foaie, cu enunţ şi demonstraţie/rezolvare, fiind însoţite de numele autorului, şcoala şi localitatea unde lucrează/învaţă. Redacţia revistei va decide asupra oportunităţii publicării materialelor primite. În atenţia elevilor Numele elevilor care vor trimite redacţiei soluţii corecte la exerciţiile şi problemele din rubrica Probleme propuse vor fi menţionate în Pagina rezolvitorilor. Elevii vor ţine seama de următoarele reguli: 1. Pot trimite soluţii la minimum cinci probleme propuse în numărul prezent şi cel anterior al revistei; pe o foaie va fi redactată soluţia unei singure probleme. 2. Elevii din clasele VI-XII au dreptul să trimită soluţii la problemele propuse pentru clasa lor, pentru orice clasă mai mare, din două clase mai mici şi imediat anterioare. Elevii din clasa a V-a pot trimite soluţii la problemele propuse pentru clasela a IV-a, a V-a şi orice clasă mai mare, iar elevii claselor I-IV pot trimite soluţii la problemele propuse pentru oricare din clasele primare şi orice clasă mai mare. Orice elev poate trimite soluţii la problemele de concurs (de tip G şi L). 3. Vor fi menţionate următoarele date personale: numele şi prenumele, clasa, şcoala şi localitatea. 4. Plicul cu probleme rezolvate se va trimite prin poştă (sau va fi adus direct) la adresa Redacţiei:

Prof. dr. Temistocle Bîrsan Catedra de Matematică

Universitatea Tehnică “Gh. Asachi” Iaşi Bulevardul Carol I nr. 11, 6600, Iaşi

(pentru “Recreaţii Matematice”) E-mail: [email protected]

Page 88: Revista (format .pdf, 1.6 MB)

CUPRINS

ARTICOLE ŞI NOTE MATEMATICE ŞT. FRUNZĂ – Fractali..............................................................................................1 G. POPA, P. GEORGESCU – Dreapta lui Euler privită ca loc geometric ................6 I. GEORGESCU, P. GEORGESCU – Un mod de calcul al numărului lexicografic de ordine asociat unei permutări .................................................8 T. BÎRSAN – Generalizări ale teoremei lui Ceva şi aplicaţii .....................................10 M. TETIVA – O proprietate a sistemelor de generatori ale unor grupuri ...................15 D. POPESCU – Şiruri recurente de forma ),(1 nn xnfx =+ ....................................17

NOTA ELEVULUI C. ANTON – Proprietăţi ale punctului lui Nagel stabilite cu ajutorul numerelor complexe....................................................................18 O. CÂRJĂ – Aplicaţii ale rotaţiei planului complex ..................................................21

CHESTIUNI METODICE GH. IUREA – Funcţii trigonometrice inverse – aplicaţii............................................24 R. GHIŢĂ, I. GHIŢĂ – O metodă de calcul al destanţei dintre două drepte neparalele în spaţiu...........................................................27 G. MÎRŞANU – Observaţii metodice privind punctele de inflexiune ........................29 M. GÂRTAN – Un procedeu de rezolvare a unei ecuaţii diofantice de gradul al doilea .......................................................................31

CHESTIUNI COMPLEMENTARE MANUALELOR M. CRĂCIUN – Numărul divizorilor şi suma divizorilor unui număr natural ...........32 C. CHIRILĂ – Aplicaţii ale criteriului de integrabilitate al lui Lebesgue ..................34

MATEMATICA ÎN CLASELE PRIMARE I. ROŞCAN, V. BELDIMAN – O provocare – problemele nonstandard ..................37

CONCURSURI ŞI EXAMENE CONCURSUL “ADOLF HAIMOVICI”, ed. a VI-a, Iaşi, 2002.................................39 CONCURSUL “FLORICA CÂMPAN”, ed. a II-a, Iaşi, 2002 ...................................42 CONCURSUL “TRAIAN LALESCU”, ed. a III-a, Iaşi, 2002...................................46 OLIMPIADA DE MATEMATICĂ, faza judeţeană, Iaşi, Baraj, 2002.......................47 OLIMPIADA DE MATEMATICĂ – clasele V - VI, Iaşi, 2002 ................................47 OBM – JUNIORI, 2002 (probleme propuse, soluţii)..................................................50

PROBLEME ŞI SOLUŢII Soluţiile problemelor propuse în nr. 2/2001 .................................................................54 Soluţiile problemelor pentru pregătirea concursurilor din nr. 2/2001...........................68 Probleme propuse .........................................................................................................72 Probleme pentru pregătirea concursurilor .....................................................................76 FILIALA IAŞI A SSM – REPERE ALE ACTIVITĂŢII .............................................80 Pagina rezolvitorilor......................................................................................................81